EPPP Exam Questions

¡Supera tus tareas y exámenes ahora con Quizwiz!

T/F A 13 year old could be administered the WAIS to test for MR.

F WAIS age range is 16-89. They might be given the SB5 (range 2-85), or another appropriate test.

____ is a rare, idiosyncratic reaction to neuroleptic drugs that may be caused by dopamine blockage in the basal ganglia. Muscle rigidity, hyperthermia, and stupor are the core symptoms of this. Other symptoms include painful joints, mutism, tachycardia, and urinary retention. a) tricyclic overdose b)Neuroleptic malignant syndrome (NMS) c)5-HT toxicity syndrome d)lithium toxicity

Neuroleptic malignant syndrome (NMS)

____ is characterized by muscle rigidity, hyperthermia, and stupor

Neuroleptic malignant syndrome (NMS)

______ is a rare, idiosyncratic reaction to neuroleptic drugs that may be caused by dopamine blockage in the basal ganglia.

Neuroleptic malignant syndrome (NMS)

__ transformations preserve the rank order and relative size of the distance between scores.

Linear

____ variables indicate interaction.

Moderating variables

_____investigated age-related changes in cognitive abilities. In contrast to earlier cross-sectional studies which indicated that increasing age in adulthood is associated with declines in many cognitive abilities, Schaie's longitudinal study found substantial age-related declines for only two abilities - perceptual speed and numerical ability.

Schaie's Seattle Longitudinal Study

Internal models of the self & the world that develop over the course of experiences beginning in early life. Can be adaptive: facilitate more efficient information processing. if maladaptive can dominate person's perception of a situation & create or maintain a maladaptive emotional state.

Schemas (Core Beliefs or Underlying Assumptions)

Of the following, which is the best way to control the halo effect. a. train the raters. b. use peer ratings. c. make the results of assessments available to employees. d. involve raters in the development of the rating scale.

a

Parasuicide (i.e., attempted, but not completed, suicide) is least associated with. a. middle to upper SES. b. female gender. c. younger age (under 30). d. divorce.

a

Recent research suggests that people who watch a lot of television. a. fear their environment more than people who report being less frequent TV viewers. b. fear their environment somewhat less than people who report being less frequent TV viewers. c. fear their environment much less than people who report being less frequent TV viewers. d. are about equal in terms of fear of their environment to people who report being less frequent TV viewers.

a

Regression analysis is used to: a) predict the mean value of a dependent variable on the basis of a known value on an independent variable b) predict the true value of a dependent variable on the basis of a known value on an independent variable c) estimate the true value of a variable on the basis of an obtained value on that variable d) estimate the true value of a variable on the basis of a linear combination of two or more variables

a

Research comparing heterogeneous to homogeneous work groups has found that heterogeneous groups. a. are more creative and better at decision-making. b. are more creative but less productive overall. c. make better decisions but are less productive overall. d. are more creative but worse at decision-making.

a

Scores on standardized IQ tests are usually considered to represent an _____ (scale of measurement): a)interval b)ordinal c)ratio d) nominal e)ratio

a

turning inward of the libido: a)extroversion b)introversion c)ego d) id

b) introversion

Which age group of Native Americans has the highest suicide rate? a) 15-34 b) 34-45 c) 45-56 d) 65+

15-34

Age of onsete for ADHD is prior to age___ a) 5 b) 6 c) 7 d) 8

7

_____ is generally considered the treatment-of-choice for Panic Disorder.

Panic Control Treatment (PCT)

Who: Client Centered

Rogers

Who: reality therapy

William Glasser

On a histogram, scores are plotted on the horizontal axis or ___-axis or ___________. a) x, abscissa b) x, ordinate c) y, abscissa d) ordinate

a

finding pleasure in external stimuli. a)extroversion b)introversion c)ego d) id

a) extroversion

damage to the medial temporal lobes along with certain adjacent structures can produce _____

amnesia.

your achievement measure is unable to distinguish between people with high levels of achievement

ceiling effect

health belief model

community psychology strategy--preventative health care

Multiplication and division can be performed on: a) nominal scales b) ordinal scales c) interval scales d) ratio scales

d

clinical trials (experimental)

efficacy studies

Joining is a technique used by ____

structural family therapists

A short coming of the single group time series design is that its internal validity can be threatened by:

history

20 - 22%

how many older adults meet criteria for a mental d/o according to APA (2004)

1952 by Eysenck. Results of 24 outcome studies published between 1920 & 1950 were summarized. Eysenck concluded that effects of therapy were "small or non-existent" & that any positive effects may reflect nothing more than spontaneous remission.

psychotherapy efficacy debate started when & by whom?

According to Millon, when fantasy and repression fail to help a person with Narcissistic PD deal with personal failure ansocial humiliation, the person trypically resorts to: a) denial & confabulation b) rationalisation & projection c) displacement & reaction formation d) hypochondriasis

rationalisation & projection

OCD and OCPD share which symptom? a) prominent obsessions b) repetitive rituals c) risk aversion d) emotional rigidity

repetitive rituals

Which of the following has the strongest evidence of success for reducing teen pregnancy rates? a) abstinence only programs b) service learning programs c) school condom distribution d) community wide programs

service learning programs

Clomipramine exerts its beneficial effects on OCD sympotoms through its action on: a) epinephrine b) norepinephrine c) dopamine d) serotonin

serotonin

________ (McGuire, 1969) to be a useful method for helping people resist persuasive messages. It involves providing the individual with arguments against his/her initial position along with counterarguments prior to hearing the persuasive message.

attitude inoculation

Helm's white racial identity model distinguishes btwn 6 statuses, with each status being associated with a differen info processing strategy. For exampl, the "flexibility and complexity" strategy is characteristic of the _______ status a) internalisation b) integrative awareness c) reintegration d) autonomy

autonomy

The _________ heuristic refers to the tendency to judge the frequency of an event based on how easy it is to think of examples of that event.

availability

The_______ was defined as the belief that change, rather than stability, is the normal mode of life and constitutes motivating opportunities for personal growth rather than threats to security.

challenge disposition

Which of the following disorders is known to be caused by brain lesions. a. myasthenia gravis. b. cerebral palsy. c. cystic fibrosis. d. Parkinson's disease.

cerebral palsy.

Preoperational development spans from ages __ to __

2 to 7

A GAF score over ____ indicates that there is no need for therapy a) 50 b) 60 c) 70 d) 80

70

A prognosis of _______ is worse than that of Sx a) Autism b) MDD c) BPD d) ADHD

Autism

Although relaxation training has not been found to be particularly effective for _____, neither has biofeedback.

asthma

The single subject designs do not include the: a) "AB" design b) "ABB" design c) "ABAB" design d) "ABA" design

b

attributing unacceptable wishes to another: a)displacement b)projection c)fixation d)sublimation

b) Projection

Which of the following is likely to be one of a child's first words. a. ball. b. give. c. cold. d. doggy.

c

pleasure principle--source of sexual & aggressive drives. a)superego b)id c)ego d)oral

c) id

Holland: Agricultural work is consistent with the _____ theme.

realistic

standard error of mean decreases

when population standard deviation is decreased and sample size is increased

The behavior described in this question is referred to as the _____ At about ____ months of age, children first provide evidence that they know that objects exist when they are out of view. However, they commit the A-not-B error, which continues to about ____ months of age.

"A-not-B error."eight, 12

What % of fraternal twins both have Sx? a) 10% b) 13% c) 17% d) 25%

17%

aka signal anxiety b/c an impulse is seeking expression

anxiety

In Dementia (rather than pseudodementia) symptoms have a __ onset a) gradual b) abrupt c) unexpected

gradual

Bem's self-perception theory; a) our attitudes and behavs are unrelated b) emotions shape our attitudes and behavs c) we infer attitudes from our behavs d) we rely on our attitudes to choose a behav

we infer our attitudes from our behavs

credibility, the client's perception that the therapist is expert & trustworthy. The 2nd process is giving the client's perception that he/she has received something from therapy.

what 2 processes are critical when working with culturally diverse populations according to Sue & Zane (1987)

In order of incidence: Anxiety, severe cognitive impairments, & depression

what are the most common mental health problems in older adults?

calculating effect size which converts data from different studies to a common metric so results can be quantitatively combined & compared

what is meta-analysis

475 studies between 1941 & 1976; mean effect size .85; indicates that the average client at the end of therapy "is better off than 80% of those who need therapy but remain untreated."

what was the result of the smith, glass, & miller (1980) meta-analysis

no one type consistently superior, but CBT superior for panic, phobias, & compulsions

which form of therapy is most efficacious?

1) behavioral & environmental interventions for dementia; 2) memory & cognitive retraining "probably efficacious" for patients with dementia; 3) cognitive, behavioral, & brief psychodynamic therapies have been shown to be "probably efficacious" as treatments for depression

which treatments are especially effective for which disorders with older adults (Gatz et al., 1998)?

overall--members of ethnic & cultural minority groups. African Americans higher than Anglo Americans (whites); Asians lower drop-out rate; Hispanics drop-out rates similar to Anglo Americans (Sue et al., 1991)

who is likely to prematurely terminate therapy?

all levels of the independent variable are administered sequentially to all subjects. a

within subjects design

___ is a precursor of serotonin, and a rapid drop in plasma levels of this is known to precede a decrease in brain levels of ___ and serotonin.

Tryptophan, Tryptophan

Females with ________do not develop Females with this disorder have an XO chromosome pattern (i.e., one X chromosome with no other functioning sex chromosome). In females with ____________, part or all off one X chromosome is missing. secondary sex characteristics during puberty. They are typically short with a thick, webbed neck and are usually infertile due to a lack of normal ovaries. Many have mild learning problems

Turner syndrome

A person with damage to which area of the frontal lobes is most likely tobe associated with apathy and paucity of speech and movement? a) medial frontal area b) anterior cingulate area c) dorsolateral prefronal area d) orbitofrontal area

anterior cingulate

Korsakoff's syndrome is characterised by: a) dysgraphia, finger agnosia, acalculia, and right-left disorientation b) responding to simple questions with absurd, approximate, or inappropriate answers c) strong oral tendencies, hypersexuality, lack of concentration, and alterations in appetite d) anterograde amnesia, confusion, and confabulation

anterograde amnesia, confusion, and confabulation

Which medications are most commonly considered second-line medications for ADHD? a) neuroleptics b) anxiolytics c) antidepressants d) anticonvulsants

antidepressants

After psychostimulants, what type of medication is a second-line tx for ADHD?

antidepressants (e.g., bupropriate imipramine)

Which type of PD is characterised by being least disrupted by affect? a) schizoid b) schizotypal c) antisocial d) histrionic

antisocial

Which would be least useful for treating alcoholism? a) social-skills training b) stress management training c) antidepressant meds d) anxiolytic meds

anxiolytic meds

in a ___, power is delegated throughout the organization. In contrast, in _____, power is primarily in the hands of one person or a few people.

decentralized organization, centralized organizations

The MMPI's K scale is treated as which of the following when scoring? a) a suppressor variable b) an extraneous variable c) a measure of response consistency d) a measure of random responding

suppressor variable

____is irrelevant to the criterion, but affects criterion scores and thus acts as a source of error and reduces the correlation between the predictor and the criterion.

suppressor variable

The symptoms that would be expected from the abrupt discont of beta-blockers include: a) fever, sore throat, mouth ulcers and lethargy b) nausea, diarrhea, vomiting c) sweating, palpitations, headache, termulousness d) constricted pupils, decreased visual acuity, sweating, constipation

sweating, palpitations, headache, termulousness

The _____________ is responsible for the body's fight/flight reaction: a) limbic system b) thalamus c) parasympathtic nervous system d) sympathetic nervous system

sympathetic nervous system

Dementia Complex, a form of dementia that begins in the ___ of HIV infection.

symptomatic HIV infection (third stage)

Numerous opportunistic symptoms develop during the ____ stage of HIV

symptomatic HIV infection (third stage)

Visual imagery is linked to which: a) putamen b) hippocampus c) reticular formation d) basal ganglia

hippocampus

Validity can be no greater than _______ reliability

the square root of

According to DSM-IV-TR, the estimated prevalence of Schizophrenia ranges between ____ and ____

0.5% and 1.5%.

____ is characterized by headache, tremor, and confusion

5-HT toxicity syndrome

Avoid blaming women for their current problems. emphasize woman's strengths.

Avoiding Revictimization

T/F Bipolar I is more functioning than Bipolar II

F

Data in research studies can be quantified accorded to __ scales of measurement: a) 1 b) 2 c) 3 d) 4

d

Combines between groups and within subjects methodologies.

mixed design

For a test with a mean of 100 and SD of 15, the upper and lower limits of the SE of measurement are: a) 0 to +1 b) -1 to +1 c) 0 to 100 d) 0 to 15

0 to 15

A Cohen's d of .50 indicates that one group obtained a mean that is ______ standard deviation(s) higher than the mean obtained by the other group.

0.5 (1/2 SD)

An IQ score ____ SD(s) below the mean is required for a diagnosis of borderline intellectual functioning a) 0.5 b) 1 c) 1.5 d) 2

1 =85

a brief psychotic disorder is diagnosed with symptoms are present for how long? a) less than 3 months b) 1 day-1month c) less than 6months d) 1-3months

1 day-1 month

A diagnosis of Tourette's is made in the presence of both motor and vocal tics for a minimum of how long? a) 3 months b) 6 months c) 1 year d) 2 years

1 year-if not that long, it is diagnosed as Tic Disorder NOS transient

Sx is evident in what percentage of the total population? a) 0.5% b) 1% c) 2% d) 5%

1%

.01 alpha level

1% chance researcher has incorrectly rejected the null-making type one error

Schizophreniform disorder is diagnosed when symptoms are present for how long? a) less than one month b) 1-6months c) 3-9months d) less than 1 year

1-6months

According to Bandura's social learning theory, what 4 processes are involved in observational learning?

1. Attention (attending to and accurately perceiving behavior) 2. Retention (symbolic processing of modeled behavior) 3. Reproduction 4. Motivation

Children's understanding of race as a physical, social and biological category is usually first evident by age: a) 4 b) 6 c) 10 d) 13

10

Schizoaffective disorder is diagnosed in the presence of psychotic and mood disorder features. There must be a period of _________ days in which psychotic but not mood smptoms were evident: a) 7 b) 14 c) 21 d) 28

14

What is the concordance rate in fraternal twins with Sx? a) 17% b) 25% c) 46% d) 48%

17%

________ as many males commit suicide as females a) 2 b) 3 c) 4 d) 6

4-5

Kochanska and Knaack (2003) conclude that effortful (inhibitory) control is an important contributor to the early development off conscience. Their research found that effortful control becomes a stable, coherent trait by ________ months off age. a) 8 to 12 b) 12 to 18 c) 18 to 30 d) 33 to 45

33-45

Outpatient psychotherapy use in the US is hing women, whites, and individuals _ years of age a)18 to 24 b) 25 to 34 c) 35 to 44 d) 65 and older

35-44

Teratogens are most likely to cause severe structural damage during the: a) first 8wks of dvpt b) 9-18 wks of dvpt c) 2nd trimester d) 3rd trimester

1st 8 weeks of dvpt

From adolescence onwards, girls are ______ times more likely to be depressed than boys a) 2 b) 3 c) 4 d) 6

2

Acute stress disorder lasts between: a) 1 day-2 weeks b) 2 days-3 weeks c) 2 days-4 weeks d) 2 days-6 weeks

2 days-4weeks

For a DSM diagnosis of acute stress disorder, the indiviudal must deveop symptoms withing 4 weeks of exposure to a trauma and symptoms myst persist for a min of _________ and a max of __________ a) 2 days; 4 weeks b) 4 days; 2 weeks c) 2 days; 6 weeks d) 4 days; 8 weeks

2 days; 4 weeks

In a normal distribution, a percentile rank of approx ________ is equivalent to a score that is 2 SDs below the mean a) 16 b) 4.5 c) 2.5 d) 1

2.5

In kids with LD, the most frequent co-diagnosis is ADHD with approx _______ & of LD kids also receiving this diagnosis a) 5-10 b) 20-25 c) 50-55 d) 65-70

20-25

Females attempt suicide about ___ times more often than males a) 2 b) 3 c) 4 d) 6

3

A GAF score of _______ indicates suicidal ideation a) 35 b) 45 c) 55 d) 65

45

This highest lifetime risk for MDD is among individuals aged: a) 18-29 b) 30-44 c) 45-64 d) 65+

45-64

What % of children where both parents have Sx are diagnosed with Sx? a) 30% b) 36% c) 42% d) 46%

46

What is the concordance rate in children with both parents with Sx? a) 17% b) 25% c) 46% d) 48%

46%

What % of monozygotic twins both have Sx? a) 46% b) 48% c) 50% d) 55%

48%

What is the concordance rate in monozygotic twins with Sx? a) 17% b) 25% c) 46% d) 48%

48%

A better prognosis for individuals with autism is associated with communication skills acquired by age: a) 3 b) 5 c) 7 d) 10

5

What percentage of children diagnosed with ADHD has some problems in adulthood? a) 40 b) 50 c) 60 d) 70

60

An IQ equal to or below __________ is required for diagnosis of MR a) 60 b) 65 c) 70 d) 75

70

Suicide rate is highest for males aged: a) 15-19 b) 20-24 c) 40-44 d) 75-79

75-79

An IQ score of _______ suggests broderline intellectual functioning a) 70 b) 75 c) 80 d) 85

85 (1 SD below the mean)

Frequent conflicts, especially those involving open expressions of hostility, between husbands and wives. a. is more likely to lead to increases in under controlled behaviors in male children than in female children. b. is more likely to lead to increases in under controlled behaviors in female children than in male children. c. has similar negative behavioral consequences for male and female children. d. has not been linked to particular behavioral problems in either male or female children.

A

This is the simplest single-subject design, which includes a single baseline

ABA

Refers to a therapist's ability to see the world as the client does & to convey that understanding to the client

Accurate Empathetic Understanding

The highest rates of emergency room visits occurs for what racial group?

African American

Which PD is most similar to social phobia? a) Avoidant PD b) Schizoid PD c) schizotypal PD d) Dependent PD

Avoidant PD

when using hte DSM, _______ is used ot record physical or biol factors a) Axis II b) Axis III c) Axis IV d) Axis V

Axis III

The most recent edition of the Stanford-Binet represents an extensive revision of this test. Among other things, when using the new fourth edition of the Stanford-Binet. 1. it is no longer necessary to score items as they are administered. 2. performance on the vocabulary sub test is used to help determine "entry level" on all other subtests. 3. fifteen different subtests are administered to each examinee. 4. raw subtest scores for each subtest are converted to standard scores that have a mean of 50 and a standard deviation of 8. a. 1 and 4 only. b. 2 and 4 only. c. 1,2, and 3 only. d. 1, 2, 3, and 4.

B

Which is most useful for explaining why so many people think that their horoscopes accurately describe them? a) Barnum effect b) false consensus bias c) Zeigarnik effect d) illusory correlation

Barnum effect -people accept vague or general descrips of themselves as accurate (guilibility, wishful thinking, and confirmation bias)

how 1 thinks greatly influences how one feels & behaves. therapy is a "collaborative process of empirical investigation, reality testing, & problem-solving" b/w client & therapist.

Beck's Cognitive Therapy

__________ occurs when two different conditioned stimuli are paired with the unconditioned stimulus.

Blocking Note: In this case the second CS rarely successfully become associated with the US.

___________ is the term for speech that is delayed or indirect because of inclusion of parenthetical or unneccessary details and parenthetical remarks.

Circumstantiality note: in contrast to loose associations, it always comes back to main point

the ____ is a measure of cognitive functioning that is useful for assessing individuals with Alzheimer's dementia or other type of primary degenerative dementia

Global Deterioration Scale

____is a disturbance of the patient's ability to respond to stimuli on the side of the body opposite to the side of brain lesion, in the absence of simple sensory or motor deficits.

Contralateral neglect

The assumption that changes in regional cerebral blood flow correspond to changes in neural activitiy underlies the use of all of the following brain imaging techniques except: a) PET b) fMRI c) CT d) SPECT

CT

_____ is the tendency for genetic endowment to restrict the development of certain characteristics to one or a few outcomes.

Canalization

Field: humanistic; Contributions: founded person-centered therapy, theory that emphasizes the unique quality of humans especially their freedom and potential for personal growth, unconditional positive regard, fully functioning person

Carl Rogers

_________ means making up things to compensate for memory impairment.

Confabulation

__________ involves administering the treatments (different levels of the IV) in different orders to different groups of participants, and is used to control multiple treatment interference (also known as carry-over or order effects) that may result when multiple levels of the independent variable(s) are administered to the same participants.

Counterbalancing

____ refers to the degree to which a criterion measure fails to measure all aspects of the ultimate (conceptual) criterion.

Criterion deficiency

is due to malfunctioning of the adrenal cortex.

Cushing syndrome

planning post-consultation matters; reducing involvement & follow-up; & termination

Disengagement

Racial/Culturall Raciall Identity Development model: ____results when the individual realizes that there are inconsistencies in his/her cultural attitudes, values, and beliefs.

Dissonance (ambivalence)

Summarize the research on leaders conducted at Ohio State in the 1950s.

Distinguished task-oriented (initiating structure) and person-oriented leaders (consideration) approaches

1st generation antipsychotics exert their therapeutic effects by acting as _________ receptor antagonists a) D1 b) D2 c) 5-HT d) H1

D2

Researchers interested in work-related communication distinguish between two types of communication networks - centralized and decentralized. __________ networks are best for complex, unstructured tasks, while _______ networks are better for simple, structured tasks.

Decentralized centralized

_______ occurs in the _______ stage and contributes to the ability to conserve.

Decentration, concrete operational

______, response B, refers to reasoning from the general to the specific.

Deduction

place greater emphasis on the ego's role in personality development. two ego-functions: ego-defensive (resolution of conflicts) & ego-autonomous functions (adaptive, non-conflict laden functions)

Ego-analysts

Normal/existential anxiety--proportionate to the cause, constructive, does not require repression. Neurotic anxiety--person tries to evade normal anxiety. typically manifests as a loss of subjective sense of free will & an inability to take responsibility fro one's own life.

Existential therapists identify 2 types of anxiety

T/F Sleep terrors occur in REM stage of sleep

F

emphasis on the power differences between men & women & how that difference impacts on both men's & women's behaviors; intrapsychic events always occur & must be interpreted within an oppressive social context

Feminist Therapy characteristics

six bases of social power identified by ____

French and Raven

a family is conceptualized as a system--more than just the sum of its parts due to relationships & transactional patterns. Open system receives energy by interacting with the environment. Closed system is resistant to change b/c of rigid or impermeable boundaries.

General Systems Theory

therapist must be genuine & authentic in therapy. honest communication of therapist's feelings whenever it is appropriate

Genuineness (Congruence)

perceptions of parts as wholes

Gestalt

psychoanalysis, phenomenology, & existentialism

Gestalt Therapy incorporates principles from...

subjective experiential change

Hypnosis essential feature (Orne & Dinges, 1989)

Sue et al (1991)-Which ethnic group show best outcomes at end of therapy? a) African-Americans b) Anglo-Americans c) Asian-Americans d) Hispanic-Americans

Hispanic-Am

Levels off at about 26 session. approx 75% of patients show measurable improvements at 26; the number only rises to 85% at 52 session. This is called dose-dependent effect

Howard et al. (1986) indicates that the relationship b/w treatment & outcome...what?

Both internal and external validity are concerns in the design of research. ___refers to the extent to which it can be assumed that the changes in the dependent variable are caused by the independent variable. External validity refers to the extent which the results of research can be generalized.

Internal validity, External validity

person psychologically swallows whole concepts without fully understanding or assimilating them

Introjection (Gestalt)

____involves blending with the family by, for example, adopting its communication and affective style.

Joining

What theory: The ego is an important part of this theory, and, considered the self to be the archetype for the ego.

Jung's analytic psychology

What theory: described people as being more creative and goal-directed. promoted a more active interchange between analyst and patient placed a great deal of emphasis on changes that occur during adulthood, especially in midlife.

Jung's analytic psychology

Language and social interactions are the key elements in ________ theory of cognitive dvpt a) E Gibson's b) L Vygotsky's c) W Perry d) M Rutter

L Vygotsky

According to _____ behavior is a function of the person and the environment. He expressed this relationship with the following formula: B = f(P, E).

Lewin's field theory,

Who: rigid triangles, detouring, stable coalition, and triangulation.

Minuchin, Structural Family Therapy

_____ transformations alter the rank order and relative size of the distance between scores.

Nonlinear

is an extrapyramidal system disease that affects the control of voluntary movements, is believed to be due to a lack of dopamine in the basal ganglia. a. myasthenia gravis. b. cerebral palsy. c. cystic fibrosis. d. Parkinson's disease.

Parkinson's disease

Name three types of nonlinear transformations?

Percentile ranks (which always have a flat distribution regardless of the distribution of the raw scores) are an example of a nonlinear transformation. Logarithms and square roots are other types of nonlinear transformations.

Although ____ views irrational thoughts as determiners of dysfunctional behaviors, the notion of "automatic thoughts" is more characteristic of ____.

REBT, Beck's theory of and treatment for depression

Transform a moderator variable into another IV

Randomized block design

What therapy: the result of incongruence between self and experience.

Rogers' client-centered therapy.

disorganization of self as the result of incongruence between self & experience which can occur when the individual experiences conditions of worth

Rogers' view of maladaptive behavior

The best medication treatment for OCD is: a) tricyclics b) SSRIs c) MAOIs d) axiolytics

SSRIs

the organized, consistent conceptual gestalt composed of perceptions of the characteristics of the "I" or "me" & the perceptions of the relationships of the "I" or "me" to others & to various aspects of life, together with the values attached to these perceptions

Self (Rogers)

Awareness-aware of their assumptions, values, & beliefs; 2) Knowledge-attempt to understand the worldview of culturally diverse clients; 3) skill-use therapeutic modalities & interventions that are appropriate for culturally diverse clients.

Sue & Sue (2002) define cultural competence as having 3 competencies. what are they?

involves symbols that have shared meaning, emerges around end of 1st year of life. involves logical, sequential, & consistent thinking & underlies language development.

Syntaxic mode of cognitive development

T/F A problem with Holland's theory is that it doesn't take into account the impact of discrimination, which may limit the availability of certain types of jobs for members of certain groups.

T

T/F A subpoena is a legal document that requires a psychologist to appear at a designated time at a legal proceeding. If the client invokes the privilege (says she does not want the therapist to release confidential information to the court), the subpoena is still valid and the psychologist must appear unless he/she has been released from it.

T

T/F Apraxia, aphasia and agnosia are not part of Huntington's disorder.

T

T/F On the WAIS-III,‭ ‬a Verbal IQ that is substantially‭ ‬higher than the Performance IQ is most suggestive of depression.

T

genuine care about client, affirm client's worth as a person, & accept the client without evaluation.

Unconditional Positive Regard (respect)

The ____is a measure of motor speed, visual attention, and cognitive flexibility.

Trail Making Test

For Minuchin _____ occurs when both parents attempt to get the child to side with him or her.

Triangulation

72% of the neurotic adults in his no-therapy group showed improvement w/in 2 years of symptom onset. Only 66% of patients receiving eclectic psychotherapy & 44% receiving psychoanalytic psychotherapy showed a substantial decrease in symptoms

What did Eysenck base his conclusions on?

positive change in therapy is not due to any unique or specific techniques, but to the common factors found in all treatment modalities, e.g. catharsis, positive relationship, advice, behavioral regulation, & cognitive learning & mastery.

What does Lambert & Bergin (1994) say about the Lipsey & Wilson conclusion?

The tendency to remember interrupted or uncompleted tasks better than completed ones is: a) Barnum effect b) false consensus bias c) Zeigarnik effect d) illusory correlation

Zeigarnik effect

A behavior therapist working with a child who bites her nails sets a timer so that it rings every 5 minutes. For every 5 minute period during which the child doesn't bite her nails, she is given a token. This treatment is an example of. a. D.R.O. b. response cost. c. overcorrection. d. Premack Principle.

a

A new client of yours, who is homosexual, tells you that he overheard his previous therapist make derogatory remarks about his sexual orientation to a co-worker. Your best course of action would be to. a. discuss his option of filing a complaint against the psychologist with the Ethics Committee. b. file a complaint against the psychologist with the Ethics Committee yourself. c. call the psychologist and tell him that your client overheard his remarks. d. provide the client with support and remind him that there are many prejudiced people in the world.

a

A person with a temporal lobe tumor is most likely to exhibit which of the following? a. memory impairment, depression or anxiety, and hallucinations. b. poor judgment, impulsivity, and inappropriate jocularity. c. confabulation, irritability and suspiciousness, and prosopagnosia. d. dysgraphia, depression and apathy, and denial of his or her problems.

a

A scale in which the units of measurement between the numbers on the scale are all equal in size. a)interval b)ordinal c)ratio d) nominal e)ratio

a

According to the path-goal theory of leadership, a primary task of a leader is to help subordinates find the best path for attaining goals. In terms of leadership style, this theory proposes that. a. the best style (directive, supportive, participative, or achievement-oriented) depends on certain characteristics of the task and the subordinates. b. the best style (directive, supportive, participative, or achievement-oriented) depends on the leader's experience, personality, and so on. c. the best style is a participative one that allows employees to help set their own goals. d. the best style is an achievement-oriented one that focuses on enhancing the skills and knowledge of subordinates so that they are better able to achieve their goals.

a

After a training program has been completed, an industrial psychologist wants to determine if the instruction was effective. What type of evaluation will he use. a. summative. b. formative. c. primary. d. secondary.

a

An achievement test is administered to one hundred people. The test's mean is 50 and its standard deviation is 5 and the test scores are normally distributed. If you want to use test scores to select the top 16 people, you would set the cutoff score at. a. 55. b. 60. c. 65. d. 70.

a

An adolescent's adoption of a "ready-made" identity that was formulated by a parent or other authority figure is referred to as. a. foreclosure. b. moratorium. c. fusion. d. emersion.

a

An advocate of a Japanese management philosophy is most likely to advocate which of the following. a. encouraging employees to try out more than one job within the company. b. encouraging each employee to come up with ways for increasing opportunities for individual responsibility and autonomy. c. closely supervising employees . d. rapidly promoting employees who demonstrate superiority.

a

An extraneous variable is sometimes referred to as: a) a confound b) equivalence c) a criterion variable d) an output variable

a

An industrial psychologist suggests that a factory manager play background music for lineworkers. The manager decides to try this out and begins to play the radio in the factory. The music is most likely to have which of the following effects. a. it will improve performance on routine tasks. b. it will have positive long-term effects on satisfaction and motivation. c. it will decrease worker irritability and frustration. d. it will increase on-the-job accidents.

a

Approximately 40% of women report experiencing some degree of distress just prior to menstruation. The symptoms of this "premenstrual syndrome" include temporary weight gain, irritability, restlessness, and headache, and appear to be due to which of the following. a. decreased hormonal levels. b. increased blood flow. c. elevated blood sugar levels. d. excessive fluid loss.

a

Dividing subjects by age (young adults, middle aged adults, elderly adults) and assessing the abilities of each group at different points in time. a) cross-sequential design b) cross-sectional design c) time-series design d) longitudinal design

a

During your first session with a 38 year old man, he tells you that he is homosexual and that he found out, several weeks ago, that his partner of six years is having an affair and is planning on moving out of their house. The man says that he is very nervous and anxious, that he wishes he wasn't a homosexual, and that he lays in bed at night obsessing about his partner. His anxiety has affected his work: He is having trouble concentrating, is not getting along well with his co-workers, and has "called in sick" several times in the past two weeks. Based on these symptoms, the best diagnosis is. a. Adjustment Disorder with Anxious Mood. b. Bereavement. c. Ego-Dystonic Homosexuality. d. P.T.S.D..

a

Elaine considers her friend, Tom, to be quiet and reserved. She says that when she is alone with Tom or when they are in a small group, Tom listens more than he talks and is never one to "make a scene." One day, however, Elaine observes Tom in a large crowd that is waiting for the arrival of a "special celebrity guest" at the opening of a new shopping mall. The guest is nearly 30 minutes late, and the crowd is beginning to show signs of impatience. Elaine is surprised when she sees Tom initiate loud "booing" and other derogatory comments while waving his clenched fist in the air in anger. Which of the following theories or models best explains Tom's unusual behavior. a. deindividuation. b. social facilitation. c. catharsis. d. paradoxical intention.

a

Freud argued that the "work of the mental apparatus is directed toward keeping the quantity of excitation low." If the mental apparatus is unsuccessful in doing so, the result is likely to be. a. anxiety. b. ego decompensation. c. psychosis. d. object-cathexis.

a

If Student A originally scored near the mean, Student B originally scored at the high end of the distribution and Student C originally scored at the low end of the distribution and there was a raw score change, what students percentile rank would be affected the most? a) Student A b) Student B c) Student C d) They will all be affected equally

a

If the raters' assessment abilities improved over time then what threat to internal validity would this be? a) instrumentation b) testing c) statistical regression d) differential mortality

a

In group therapy, members establish a "taking turns"pattern in which each group session is devoted, sequentially, to each group member. In response to this situation, Yalom recommends: a. mass group interpretation. b. here-and-now activation. c. paradoxical prescription. d. reframing

a

In their theories of development, both Piaget and Kohlberg emphasize which of the following. a. cognitive maturation. b. individual differences. c. imprinting. d. unconscious conflicts.

a

Kohlberg argued that there is a monotonic relationship between moral judgment and moral action and proposed that, as one moves from a lower to a higher stage of moral development. a. the range of possible moral actions becomes narrower and the individual assumes greater responsibility for relating his or her judgments to actions. b. the range of possible moral actions becomes broader and the individual assumes greater responsibility for relating his or her judgments to actions. c. the range of possible moral actions remains the same but the individual assumes greater responsibility for relating his or her judgments to actions. d. the range of possible moral actions becomes narrower but the individual's sense of responsibility for relating his or her judgments to actions remains about the same.

a

Margaret Mahler, who is associated with object relations tlieory, views separation-individuation as a reflection of. a. development of a permanent sense of self and a permanent mental representation of the other. b. the formation of strong bonds with additional "significant others". c. splitting of the ego and internalization of the "good mother" as the ideal object. d. development of the conflict-free ego sphere.

a

Methods of random sampling include all of the following except: a) random arrangement sampling b) stratified random sampling c) cluster sampling d) multistage cluster sampling

a

Most interpretations of performance on the Bender-Gestalt are directed toward. a. screening for brain damage. b. assessing personality. c. estimating intelligence. d. identifying learning disabilities.

a

Stimulus control training is most effective when. a. the target behavior is consistently reinforced when it is performed in the presence of the discriminative stimulus. b. a continuous reinforcement schedule is replaced by an intermittent one once the target behavior has reached its desired level or intensity. c. the latency between presentation of the discriminative stimulus and the performance of the target behavior is maximized. d. the reinforcement is consistently withheld following the target behavior.

a

Subjects learn a task while under the influence of a CNS stimulant. Three days later, half of the subjects are tested while under the influence of the drug; the other half are tested while drug-free. Subjects who have been given the drug perform better on the task during testing. This result is predicted by which of the following? a. state dependent memory. b. proactive facilitation. c. drug-dependence effect. d. priming.

a

T-scores are based on ___ point intervals with T= ___ being the distribution's mean. a) 10, 50 b) 10, 60 c) 100, 500 d) 100, 600

a

Test-retest reliability would be the most appropriate method of determining the reliability of a test if the test is designed to assess which of the following. a. general intelligence. b. mood. c. state anxiety. d. reading level.

a

The F-ratio represents a comparison between two estimates of: a) variance b) standard deviation c) means d) t-scores

a

The Strong Interest Inventory would probably be least useful. a. as a job selection test. b. as a guide for choosing college major. c. for making career decisions. d. for predicting turnover.

a

The _________ is the average of the squared differences of each observation from the mean. It tells us about the distribution of data and is equal to the square of the standard deviation. a) variance b) z-score c) t-score d) normal distribution

a

The concept of triangulation is central to general systems theory and can be found in the work of Bowen, Minuchin, Satir, and others. Although these theorists differ somewhat in their definition of triangulation, in general, it is said to be occurring when. a. to reduce tension between them, a two-person system draws in a third party. b. to reduce tension between them, the members of a two-person system focus their attention on a third party. c. to avoid dealing with the conflict between them, a two-person system forms an alliance against a third person. d. to reduce conflict, two members of a three-person system assume a submissive role when interacting with the third dominant member.

a

The independent variable may also be known as the: a) predictor or input variable b) predictor or output variable c) input or criterion variable d) output or criterion variable

a

The original purpose of the classic Hawthorne studies conducted at the Western Electric plant in Chicago in the 1920s and 1930s was to determine. a. the effects of certain environmental factors on work performance. b. the impact of information work group norms on worker productivity. c. the effects of innovative management styles on job satisfaction. d. the relationship between job responsibility and job satisfaction.

a

The owner of several fast-food restaurants would be best advised to use which of the following intervention strategies if his goal is to help his employees stop smoking cigarettes. a. supplement a behaviorally-oriented program with inter-restaurant competition. b. give employees individual bonuses for reduced cigarette smoking. c. provide employees with information about the negative side-effects of cigarette smoking that is designed to arouse a moderately high level of fear. d. institute a self-control program that is monitored by a physician and a behavioral psychologist.

a

The presence of which of the following symptoms would suggest a diagnosis of Conduct Disorder rather than a diagnosis of Oppositional Defiant Disorder. a. frequent lying and running away from home. b. low frustration tolerance and temper outbursts. c. drug use. d. onset of symptoms after age 11.

a

The primary purpose of the state licensing boards is to. a. protect the public from poorly trained and incompetent psychologists. b. supervise the training of psychologists. c. keep a limit on the number of licenses granted yearly. d. uphold APA's ethical practices guidelines.

a

The word HEART is projected onto a screen so that HE is projected to the subject's right visual field only while ART is projected to the subject's left visual field only. If the subject in the experiment is a "split brain"patient, you would expect him or her to report seeing. a. HE only. b. ART only. c. ART HE. d. HEART.

a

Undernutrition during the prenatal period and early infancy. a. affects both the size and structure of the brain's cells. b. primarily affects the peripheral (versus central) nervous system. c. increases the body's susceptibility to disease, especially during childhood. d. impairs later cognitive functioning but has little impact on social or emotional development.

a

Unilateral ECT to the right hemisphere would most likely produce. a. predominantly anterograde amnesia involving nonverbal material. b. predominantly anterograde amnesia involving verbal material. c. predominantly retrograde amnesia involving nonverbal material. d. predominantly retrograde amnesia involving verbal material.

a

Various predictors are used in industry for the purpose of personnel selection. When validating these predictors, which of the following is most commonly used as the criterion measure. a. supervisory ratings of job proficiency. b. peer ratings of job proficiency. c. objective measures of production quality and quantity. d. salary level

a

What kind of study helps experimental psychologists develop or refine their hypotheses before conducting an experimental study? a. qualitative pilot study b. quantitative review c. protocol analysis d. descriptive summary

a

When conducting an experiment, a researcher sets alpha at .01. On the basis of the results of the t-test she uses to analyze her data, the researcher concludes that her results are "significant at the .01 level." This means that. a. there is a 1% probability that she has falsely rejected the null hypothesis. b. there is a 1% probability that she has falsely retained the null hypothesis. c. there is a 99% probability that she has correctly rejected the null hypothesis. d. there is a 99% probability that she has correctly retained the null hypothesis.

a

When using the Halstead-Reitan, an examinee's performance on seven measures (e.g., the Category Test, the Seashore Rhythm Test, and the Finger Oscillation Test) is used to derive a or an. a. Impairment Index, which can be used to determine if an examinee has brain damage. b. Impairment Index, which can be used to determine if an examinee is suffering from a functional disorder. c. Pathology Profile, which allows the examiner to compare the examinee's score profile to those of people with and without brain damage. d. Pathology Profile, which helps the examiner determine the appropriate DSM-IV diagnosis.

a

When working with a Japanese client in therapy, during the first session, you decide that your primary goals are to normalize the client's problems and instill a sense of hope. According to Sue and Zane (1987), these goals are. a. examples of "giving". b. ways of establishing ascribed credibility. c. ways of fostering "amae". d. expressions of cultural knowledge.

a

Which of the following is the most commonly used criterion-measure in organizational settings? a. supervisor ratings. b. standardized performance tests. c. quantitative measures of production. d. assessment centers.

a

You administer a test to a sample of young adults with PTSD and, from their scores, calculate a kappa coefficient of .95. This indicates that the test is. a. reliable. b. valid. c. reliable and valid. d. reliable but not valid.

a

You notice that a friend of yours gets upset every time someone is standing less than 12 feet away from him or her. Which of the following characteristics is your friend most likely to possess. a. low self-esteem. b. female gender. c. a disdain of violence. d. low levels of authoritarianism.

a

In distinguishing social phobia from agoraphobia, keep in mind that: a) the presence of panic attacks is more indicative of Agoraphobia than Social Phobia b) Agoraphobia involves anxiety in multiple social settings, while Social Phobia is always restricted to one or two settings c) a diagnosis of Social Phobia in adults is made only when the individual is aware that his/her anxiety is unreasonable d) a diagnosis of Agoraphobia in individuals over 18 requires a duration of symptoms for at least six months

a diagnosis of Social Phobia in adults is made only when the individual is aware that his/her anxiety is unreasonable

The DSM requires all EXCEPT for a diag of Transient Tic Disorder a) the presence of one or more motor and/or vocal tics b) a duration of symptoms for no more than six consecutive months c) an onset of symptoms prior to 18 years of age d) the diagnostic criteria for Tourette's Disorder have never been met

a duration of symptoms for no more than six consecutive months Duration must be no less than 4 weeks and no more than 12 months

Combining MAOI with certain cold, cough, sinus or allergy meds can produce: a) neuroleptic malignant syndrome b) a hypertensive crisis c) agranulocytosis d) tardive dyskinesia

a hypertensive crisis

An assessment of an employee's job performance is called....

a performance appraisal

Maternal employment during the 1st 3yrs of child's life is most likely to have: a) a significant and long-term negative impact on the child's emotional, cognitive, and behavioral functioning b) a small negative impact on the child's cognitive functioning but no consistent impact on emotional or behavioral development c) a small negative impact on the child's emotional and behavioral functioning but no consistent impact on cognitive development d) a significant negative impact on the child's behavioral functioning but no consistent impact on cognitive and emotional development

a small negative impact on the child's cognitive functioning but no consistent impact on emotional or behavioral development

telological approach is: a) beh as being largely motivated by a person's future goals. b) all behavior is purposeful c)ndividuals have a moral obligation to help, serve, or benefit others d) that nothing exists beyond the natural universe

a)

Studies have found that by ______ months of age, over 60% of infants recognise their own image in a mirror a) 21 b) 18 c) 15 d) 12

a) 21

According to_________ a supervisor can adopt one of three roles and in each role can focus on one of three skill areas a) Bernard and Goodyear's Discrimination Supervision Model b) Stoltenberg and Delworth's Integrated Development Model c) Roger's Person-Centered Supervision Model d) Ellis's Rational Emotive Behavioral Supervision Model

a) Bernard and Goodyear's Discrimination Supervision Model

Perls was a co-founder of: a) Gestalt therapy b) reality therapy. c) solution-focused brief therapy. d) transactional analysis.

a) Gestalt therapy

The goodness-of-fit model proposed by Thomas implies: a) adjustment in children is related to the degree of congruence between a child's basic temperament and his/her parents' caregiving b) life satisfaction is related to the degree of congruence between a person's basic personality and his/her chosen lifestyle c) adjustment to marriage is maximized when partners have similar values and goals d) cognitive development is maximized when environmental demands are consistent with the child's biological maturation

a) adjustment in children is related to the degree of congruence between a child's basic temperament and his/her parents' caregiving

Bandura's social learning theory predicts that: a) cognitive events act as intervening variables in the acquisition of a new behavior b) social approval is a powerful and primary reinforcement c) reinforcement has a greater impact on learning than on performance d) the acquisition of new behaviors is the result of internalized S-R connections

a) cognitive events act as intervening variables in the acquisition of a new behavior -obvs learning involves 4 processes: attention, retention, reproduction and motivation. Attention and retention are cog processes

Marlatt & Gordon's relapse prevention therapy focuses on teaching individuals to: a) cope with situations that trigger relapse b) open up about their feelings c) manage their feelings of anger d) avoid situations that trigger relapse

a) cope with situations that trigger relapse

Which type of hallucinations occur when waking up and are characteristic of narcolepsy? a) hypnopompic b) hypnagogic c) illusions

a) hypnopompic

Freud's structural theory is composed of these parts: a) id ego & superego b) oral, anal, phallic c)repression, reaction, projection d) separation, individuation, exploration

a) id ego & superego

Which is likely to be lease effective when working with American Indian or Alaskan Native therapy clients? a) promoting a‭ "‬value free‭"‬ environment in therapy. b) adopting a holistic perspective that regards illness as a result of multiple factors. c) incorporating traditional healers and practices into the treatment process. d) adopting a client-centered approach that avoids highly directive techniques.

a) promoting a‭ "‬value free‭"‬ environment in therapy.--this is similar to adopting a color-blind perspective

Which type of PD is characterised by an indifference to interpersonal relationships? a) schizoid b) schizotypal c) antisocial d) histrionic

a) schizoid

conscious/morality--integration of societal & parental restrictions. consequence of successful Oedipal development. a)superego b)id c)ego d)oral

a) superego

job satisfaction is inversely related to

absenteeism and turnover

In goal-setting theory, __________ of goals is considered to be paramount.

acceptance (and intent to acheive, which may be influenced by whether or not they helped set the goals)

The ____ of a predictor can be described in terms of its sensitivity, specificity, positive predictive value, and negative predictive value. a) positive predictive value b) negative predictive value c) sensitivity d) specificity

accuracy

In LD assessment, what does a "substantial discrepancy" mean?

acheivement 2 SDs below IQ

MMPI -an examinee whose highest clinical scale scores are on Scales 4 & 9 is most likely to exhibit : a) dysphoria and anhedonia b) egocentrism and grandiosity c) denial of homosexual impulses or behaviors d) acting out behaviors

acting out behavs; 4= Psychopathic, 9= hypomania

The largest increase in the rates of suicide and suicide attempts in recent years is among ______

adolescents

As we age, we get tired earlier and wake up earlier. This is called...

advanced sleep phase

Job relatedness and business necessity are assoc with : a) adverse impact b) truth in testing c) comparable worth d) personnel training

adverse impact -if a selection procedure has an adverse impact, the employer can still using it if he can demo that it is job related and a business necessity

When a client seeks hypnosis to retrieve forgotten memories, the therapist should: a) provide such services only if he/she has adequate training and experience b) provide such services only if he/she has adequate training and experience and seeks supervision in order to ensure objectivity c) advise the client of the potential risks of hypnosis in this context before using it d) advise the client that hypnosis is not an appropriate procedure for retrieving forgotten memories

advise the client that hypnosis is not an appropriate procedure for retrieving forgotten memories

requires the consultant to adopt an explicit value orientation in order to best foster the goals of a disenfranchised group

advocacy consulting

Shedler and J. Block conclude that problem drug users exhibit a coherent syndrome, often as early as age 7, that is characterized by __, ___, & ___ and that these symptoms clearly precede the onset of drug use.

alienation, impulsivity, and subjective distress

acknowledge their American heritage but are more strongly influenced by their ethnic background, is referred to, by Phinney and Devich-Navarro, as ____.a) blended biculturalism, b) alternating biculturalism, c) separated. d)marginalization

alternating biculturalism

Among individuals aged 15-34, which ethnic groups has highest rate of completed suicide? a) whites b) african am c) am indian d) hispanic

am indian

According to Sue, an African-American who feels there is little she can do to overcome racial oppression and so is chronically anxious has: a) an external locus of control and an external locus of responsibility b) an external locus of control and an internal locus of responsibility c) an internal locus of control and an external locus of responsibility d) an internal locus of control and an internal locus of responsibility

an external locus of control and an external locus of responsibility -they are likely to exhibit learned helplessness

Berkowitz's (1971) research found that frustration leads to a "readiness for aggression" but that the actual expression of aggression requires ____________ plus the presence of ____________.

anger arousal environmental (aggressive) cues

inability to make purposeful movements

apraxia

In training programmes, utility analysis would be used to: a) identify organizational needs with regard to training. b) identify the effective components of a training program. c) assess the return-on-investment of the training program. d) assess the satisfaction of participants with the training program.

assess the return-on-investment of the training program

The key feature that distinguishes true and quasi-experimental research is random _________.

assignment

A diagnosis of undifferentiated somatoform disorder requires which? a) at least eight different physical complaints b) at least one pseudoneurological symptom c) at least one physical complaint d) preoccupation with physical symptoms

at least one physical complaint

In response to questions on the AAI-a mother describes her childhood relationships with her own parents are very positive but is unable to give specific examples of pleasant experiences. Altthough she says this inabilitiy is due to poor memory, she is able to recall other events that occuring in childhood. Most likely, her 13mo old baby will exhibit which attachment style? a) secure b) avoidant c) resistant d) disorganized/disoriented

avoidant

Which PD is characterised by social inhibition and hypersensitivity? a) avoidant b) antisocial c) schizoid d) schizotypal

avoidant

Kohlberg describes reasoning in his sixth stage of moral dvpt as involving consideration of universal ethical principles. In contrast, the last stage of Gilligan's model of moral dvpt emphasizes: a) avoiding harm to oneself and others b) scarficing one's own desires to those of others c) a desire to uphold social contracts d) ability to take prespective of others

avoiding harm to oneself and others

one technique which separates feminist therapy from others

avoiding labels

"Assessment centers" are most commonly used to. a. hire and promote clerical workers. b. hire and promote managers. c. train semi-skilled and skilled workers. d. evaluate the performance of current employees.

b

10. You are hired by a small company to provide evaluation and counseling services to employees. The company owner tells you that employees routinely sign a waiver of confidentiality when they are hired. In this situation, you should. a. tell the employer that you can give her only very limited and relevant information about an employee even though the employee has signed a waiver. b. clarify the company policy with the employees that you see and determine if they want to continue in counseling. c. provide the employer with information only after you have confirmed that the employee has actually signed the waiver. d. refuse to give the employer any information about the employees that you see.

b

A Jungian "archetype" is best described as. a. an organized constellation of feelings, thoughts, and perceptions. b. a structural component of the collective unconscious. c. the "mask" adopted by a person in response to social demands. d. a significant historical event.

b

A ___________ is referenced to other scores in the distribution whereas a __________ is referenced to items on the test. a) percentage score, percentage rank b) percentage rank, percentage score c) percentage score, percentage estimate d) percentage rank, percentage mean

b

A child living in the United States is exposed to both English and a second language between the ages of six months and three years and, as result, becomes fluent in both languages. If the child had not been exposed to the second language until after the age of 4 or 5, she would have had more trouble acquiring the second language. This provides evidence for. a. the concept of critical periods. b. the concept of sensitive periods. c. the Whorfian hypothesis. d. the notion of imprinting.

b

A child with Oppositional Defiant Disorder is least likely to exhibit which of the following. a. frequent arguments and conflicts with his or her parents. b. high degree of physical aggression toward his or her peers. c. consistently blames others for his or her mistakes. d. views his or her problems as due to the unrealistic demands of others.

b

A child would be most likely to do which of the following first. a. walk alone. b. show separation anxiety. c. pretend that a wood cylinder is a cup. d. say her first word.

b

A client presenting memory loss must be carefully evaluated to determine if the loss is due to dissociative amnesia, a medical or substance-induced disorder, or malingering. Clare R., a 46-year-old housewife, is brought to treatment by her husband. Among other symptoms, Clare displays a memory loss for both recent and distant events. Under hypnosis, however, Clare is able to remember incidents she was unable to recall before being hypnotized. Therefore, the therapist can rule out which of the following disorders. a. dissociative amnesia only. b. amnestic disorder only. c. dissociative amnesia and malingering only. d. amnestic disorder and dissociative amnesia only.

b

A leader who can recognize the need for change and create a vision that motivates followers to effectively achieve that change is referred to as a. a. transactional leader. b. transformational leader. c. superleader. d. dreamer.

b

A listener is least likely to change her attitude as the result of a communicator's message if. a. the listener accidentally overhears the message. b. the listener has prior knowledge about the content of the message. c. the communicator is arguing against his or her own best interests. d. there is a moderate discrepancy between the listener's initial position and the communicator's position.

b

A psychologist administers the MMPI-2 to a 36-year old man who has exhibited a number of neurotic symptoms. The man receives a very low K Scale score. On the basis of this score, the psychologist can tentatively conclude that the man. a. tries to present himself in a favorable light. b. is very self-critical. c. is psychopathic. d. may have an organic disorder.

b

A psychologist who obtained a Ph.D. in industrial psychology wants to change her specialty to clinical psychology. To meet the requirements of APA's guidelines regarding a change in specialty, the psychologist must. a. complete an internship in clinical psychology under the supervision of a licensed clinical psychologist. b. complete appropriate doctoral-level classes and supervised post-doctoral training. c. obtain a second Ph.D. in clinical psychology from an accredited university or professional school. d. obtain appropriate supervision of her clinical practice.

b

A psychotherapeutic treatment of smoking is investigated, using a group of 15 heavy smokers as subjects. First, the researcher obtains the subjects' mean number of cigarettes smoked per day. The, the subjects undergo the therapy. Finally, the mean number of cigarettes one week after therapy is obtained. The researcher will use the: a) t-test for independent samples b) t-test for correlated samples c) one sample t-test d) Wilcoxon Matched-Pairs test

b

A researcher wants to compare the effectiveness of different interventions in improving the language skills of autistic adults. She assigns 60 subjects from a sample of autistics into either one of three groups of 20 each: one group that will receive a controversial new form of training; one group that will receive traditional behavioral modification training; and a control group that will receive no training. Afterwards, the subjects are administered a test to assess their language skills. If a researcher wanted to know if there was a difference between group means, she would use the: a) t-test for independent samples b) ANOVA c) MANOVA d) chi-square

b

A scale of measurement that is used for rank ordering of observations. Example: Classification of freshman, sophomore, junior, senior.a)interval b)ordinal c)ratio d) nominal e) ratio

b

A woman is told that she has a serious illness for which there is no cure and that it is likely that she will die within six months. From the perspective of Kubler-Ross' stages that dying people pass through, you would expect the woman's first reaction to finding out about her prognosis to be. a. "Why me?" b."The test results must be inaccurate." c. "I must be being punished for something I did." d. "I hate my life anyway, so what difference does it make?"

b

According to DSM-FV,for approximately 30 to 40% of the individuals with Mental Retardation seen in clinical settings, no clear cause can be identified. For those individuals for whom the etiology of the disorder is known, heredity is responsible in what percentage of cases? a. less than 1%. b. about 5%. c. about 20%. d. more than 50%.

b

According to Kohlberg, moral development is directly related to. a. changes in cognitive skills. b. changes in cognitive skills and social perspective-taking. c. changes in cognitive skills and socioemotional status. d. sociocultural influences and changes.

b

According to Patterson and his colleagues, aggression in children is linked to the use of coercive discipline by their parents. In turn, the parents' use of coercive discipline is directly related to. a. the parents' misinterpretations of the child's intentions when he or she misbehaves. b. the parents' personality characteristics and the child's temperament. c. the nature of the attachment between the child and the parent. d. the effectiveness of the discipline in stopping the child's misbehavior.

b

According to the James-Lange theory. a. I think, therefore I am. b. I'm trembling, so I must be scared. c. It's noon so I must be hungry. d. That was a great birthday surprise, so I must be happy.

b

According to the logic of the ANOVA, if the independent variable has an effect, between-group variance should be _______ ________ ______ within-group variance. a) significantly smaller than b) significantly greater than c) the same as d) none of the above

b

After playing the role of a child abuser in a feature film, a well-known and formerly well-liked actor begins receiving a lot of "hate mail." The writers of these letters are exhibiting. a. psychological reactance. b. the fundamental attribution error. c. the availability bias. d. defensive attribution.

b

Among adolescents, suicide is most associated with. a. affective disorders, anxiety disorders, and histrionic personality traits. b. affective disorders, conduct disorder or antisocial personality disorder, and substance abuse. c. substance abuse, conduct disorder, and attention deficit disorder. d. depression, adjustment disorder, and anxiety disorders.

b

An effective intervention for hyperactive children would probably include which of the following. a. the child will decide on appropriate tasks and rewards for completing tasks. b. the parents will set-up a behavioral routine and enforce it consistently. c. both parents and child will determine appropriate punishments for misbehaviors. d. the parents will determine a behavioral routine but be flexible in its enforcement.

b

Androgens and estrogens are secreted by the. a. gonads only. b. gonads and adrenal glands. c. gonads and pituitary gland. d. gonads and pineal gland.

b

As sample size increases, what decreases? a) the mean b) the standard error of the mean c) the standard deviation d) the range

b

Because they share a number of symptoms, it is often difficult to distinguish between Dementia and pseudodementia of Major Depressive Disorder in elderly patients. The presence of which of the following symptoms, however, is more suggestive of Major Depressive Disorder. a. patient gives confabulated answers to questions. b. patient complains about and is distressed by memory loss. c. patient exhibits apathy and loss of interest in usual activities. d. patient exhibits frequent incontinence.

b

Conduct Disorder frequently co-occurs with ADHD. Which of the following is not true about children with both disorders? a. They have higher rates of social maladjustment than children with ADHD only. b. They have more extensive cognitive difficulties and poorer school achievement than children with ADHD only. c. They have poorer long-term outcomes and a higher risk of adult psychopathology than children with ADHD only. d. They often have a less favorable response to stimulant medication than children with ADHD only.

b

Dr. Prach, a licensed psychologist, is consulting on staff at a teaching hospital. Several psychology interns come to her to complain that they are being excluded from doing the duties they were promised at the beginning of their internships. Dr. Prach would be most helpful if she. a. reports the administrators of the internship program to the ethics committee. b. explores options the interns have in making their needs known to the administrators. c. suggests that she "go to bat" for the interns and try to convince the administrators to change their minds. d. tells the interns that she should not get involved in their dispute.

b

During the first few sessions of family therapy, a structural family therapist makes use of "tracking" and "mimesis" in order to. a. keep from becoming emotionally triangulated into the family system. b. join with the family system. c. make initial changes in the family structure. d. reduce stress in the family system.

b

During the first session with a client, you learn that he has a history of offenses that you find difficult to deal with and feel, therefore, that you would not like working with him. As an ethical psychologist you should. a. disregard your personal preferences and accept the young man into therapy. b. refer the young man to another professional. c. accept the young man into therapy but seek supervision. d. tell the young man that you cannot accept him into therapy.

b

During the third therapy session, a client reveals to you that he has been seeing another psychologist for the past six months. The client claims that this therapist doesn't seem to be "doing any good," and that he feels you'll be better able to help him. As an ethical psychologist, you should. a. call the psychologist and inform her of the situation but only with the client's consent. b. tell the client you will continue therapy with him but discuss the implications of the situation. c. tell the client you cannot continue therapy with him unless he terminates therapy with the other psychologist. d. continue seeing the client but suggest that it would be best if he terminates therapy with the other psychologist.

b

Fatigue, boredom, hunger, and physical or intellectual development are examples of this threat to internal validity: a) differential mortality b) maturation c) experimenter bias d) selection

b

Following the assessment of a child and a meeting with the child's parents, a school psychologist working for a public school district concludes that child abuse has occurred. The psychologist should. a. make a report to the school principal and let the principal handle the situation. b. make a report to the appropriate child protective agency. c. confront the parents with his suspicions and insist that they seek counseling. d. do nothing as it is not within the realm of his responsibilities and would represent a breach of confidentiality.

b

For a family systems (Bowenian) therapist, when afamily exhibits a high degree of fusion but one member is more differentiated than the others, the likely approach would be to: a. work with the least differentiated family members. b. work with the most differentiated family member. c. work with all family members as a group. d. work with all family members in individual therapy until they reach a similar level of differentiation.

b

For score reporting purposes, the most commonly used measure of variability is the: a) variance b) standard deviation c) range d) mean

b

From the perspective of Gestalt therapy, an optimal "contact boundary" is semipermeable and coterminous with the actual self. Introjection is one type of boundary disturbance. It occurs when. a. the contact boundary is located far in the environment. b. the contact boundary is located deep within the self. c. the contact boundary is turned back against the self. d. the contact boundary is extremely rigid.

b

In a normal distribution, the z-score of +1 is equivalent to a percentile rank of ___ and is therefore the cutoff point for the top 16%. a) 50 b) 84 c) 95 d) 99

b

In a research study in which it is necessary to use deception, subjects are not told the exact nature of the study prior to their participation. According to APA's Ethics Code, in this situation, the investigator is obligated to. a. explain why subjects were not informed in the presentation of his research findings. b. explain the true nature of the study to all subjects as soon as possible. c. file a waiver of informed consent with the Ethics Committee prior to beginning the study. d. do none of the above as long as the study is determined to be of significant scientific value.

b

In discussing a depressed client, a therapist says the client's problems are due to the fact that she did not experience normal grief following the death of her mother. He says that the focus of treatment will be on helping the client go through the grieving process and restore her interests and friendships so that she can overcome her loss. Apparently, this therapist is a practitioner of. a. rational-emotive therapy. b. interpersonal therapy. c. object-relations therapy. d. reality therapy.

b

When a program to reduce aggression in children is based on the work of Patterson and his colleagues, it will include which of the following? a. providing the child with opportunities for catharsis. b. training parents in child management skills. c. increasing the child's perspective-taking skills. d. individual insight-oriented therapy for each parent.

b

In family therapy, the adolescent daughter complains that she doesn't think she can change and she feels that doing what the therapist has requested is pointless. The therapist responds by saying, "Well, in that case, I think you shouldn't even try." The therapist's response is an example of. a. reactance. b. restraining. c. positioning. d. prescription.

b

Individuals with which of the following disorders cross-dress for the purposes of sexual arousal. a. transsexualism. b. transvestism. c. gender identity disorder. d. all of the above.

b

Mental health professionals are sometimes called upon to assist the court in assessing a person's potential for violent behavior. In general, the predictions made by psychologists and psychiatrists yield an abundance of. a. false negatives. b. false positives. c. true negatives. d. true positives.

b

Of all the post-hoc tests, which one is the most conservative? a) Neuman-Keuls test b) Scheffe test c) Tukey Honestly Significant Difference (HSD) test d) Fisher's Least Significant Difference Test

b

Parametric tests are based on all of the following assumptions except: a) normal distribution b) mutual exclusivity c) homogeneity of variance d) independence of observations

b

Parkinson's Disease is associated primarily with a loss of cells in the. a. reticular formation. b. basal ganglia. c. amygdala. d. temporal and occipital lobes.

b

Ranks and likert scale scores are examples of this measurement type: a)interval b)ordinal c)ratio d) nominal e) ratio

b

Rehm's self-control therapy begins with self-monitoring, which involves maintaining a daily log of. a. positive activities. b. automatic thoughts. c. distressful thoughts. d. antecedent events.

b

Research on Maslow's need hierarchy theory has demonstrated all of the following except. a. line managers perceive greater fulfillment of needs than staff managers, especially esteem and self-actualization needs. b. older workers have greater deficiencies in fulfilling esteem and self-actualization needs than younger workers. c. race seems to be related to fulfillment of needs with minority managers experiencing less fulfillment than non-minority managers. d. managers who work for small companies are less deficient in meeting their needs than managers at large companies, especially at lower levels of management.

b

Subtracting the mean from each score in a distribution, squaring each result, and then summing the squared results. a. yields the distribution's standard deviation. b. provides a measure of dispersion. c. yields a coefficient of determination d. provides a measure of central tendency

b

Systematic desensitization, a behavioral technique originally developed by Wolpe, is based on which of the following. a. aversive counterconditioning. b. counterconditioning. c. habituation. d. operant extinction.

b

The College Entrance Examination Board has conducted several well-controlled studies to assess the effects of commercial coaching courses on SAT (Scholastic Aptitude Test) performance. Results of these studies have generally found that coaching. a. has no effect on SAT performance b. has a slight (but insignificant) positive effect on SAT performance c. has a significant positive effect on the SAT performance of lower-achieving students only d. has a significant positive effect on the SAT performance of students at all levels of achievement

b

The best way to reduce the likelihood of the development of PTSD following a crisis or other unusual event is. a. stress-inoculation training prior to the event. b. immediate debriefing with opportunities to reprocess the event. c. in vivo or imaginal exposure with response prevention. d. systematic desensitization.

b

The difference between the one-way ANOVA and the factorial ANOVA is that: a) the one-way ANOVA permits the assessment of both main effects and interaction effects b) the factorial ANOVA permits the assessment of both main effects and interaction effects c) the one-way ANOVA permits the assessment of interaction effects only d) the factorial ANOVA permits the assessment of main effects only

b

The hypotheses that states that a mean (or means) is either greater than or less than another mean (or other means) is: a) the null hypothesis b) one-tail hypothesis c) two-tail hypothesis d) the alternative hypothesis

b

The less control of the external environment a researcher has, the ___________ __________ validity the study will have. a) more internal b) less internal c) greater coefficient d) less external

b

The primary limitation of the ___ is that they do not lend themselves to determining just how much difference there is between scores: a)interval b)ordinal c)ratio d) nominal e) ratio

b

When taking a practice test in preparation for the licensing exam, your performance on the test is least likely to be adversely affected by. a. a loudly ticking clock. b. the stereo that you're too lazy to turn off. c. neighbors arguing in the apartment next door. d. a thunderstorm.

b

When using covert sensitization to treat a sexual fetish, the fetish object acts as a or an. a. unconditioned stimulus. b. conditioned stimulus. c. primary reinforcer. d. secondary reinforcer.

b

Which of the following individuals is least likely to view maladaptive behavior as the inhibition of the natural capacity for growth. a. Rogers. b. Wolpe. c. Goldstein. d. Maslow.

b

Which of the following is most relevant to equity theory. a. Rosenthal effect. b. social comparison theory. c. drive theory. d. the buffering hypothesis.

b

Which of the following is not a type of descriptive statistic? a) frequency distribution b) standard error of the mean c) measure of central tendency d) measure of variability

b

Which of the following would be least likely to increase the power of a statistical test. a. increasing the reliability of the dependent variable measure by decreasing its susceptibility to random error. b. changing the level of significance (alpha) from .05 to .01. c. using a one-tailed test (assuming it is appropriate to do so). d. increasing experimental variance by increasing the magnitude of the independent variable effect.

b

Which of the following would help confirm a diagnosis of Alzheimer's Dementia while ruling out a diagnosis of Dementia due to Head Trauma? a. anterograde and retrograde amnesia appear at about the same time. b. anterograde amnesia appears first; after a period of time, retrograde amnesia appears. c. retrograde amnesia appears first and gradually worsens; after a period of time, anterograde amnesia appears. d. anterograde and retrograde amnesia appear at about the same time but the retrograde amnesia begins with impairments in recent memory and then progresses to impairments in remote memory.

b

Which of the following would probably be least useful when a therapist is working with an Asian-American client. a. identifying therapy goals during the initial session. b. establishing an egalitarian relationship with the client early in therapy. c. making use of "relationship questions" that elicit information about the client's interactions with others. d. asking the client to try to identify exceptions to the presenting problem.

b

You are hired by a community mental health center as an outside consultant to help one of the clinicians develop a treatment plan for a difficult client. This type of consultation is referred to as. a. consultee-centered case consultation. b. client-centered case consultation. c. client-administrative case consultation. d. consultee-administrative case consultation.

b

An employee who feels that salaries and other benefits where he works are not fairly decided is concerned with: a) procedural justice b) distributive justice c) retributive justice d) interactional justice

b) distributive justice

You conduct a study to compare the effects of treatment type and treatment duration on cigarette smoking. Following administration of the treatments to subjects for the appropriate length of time, you determine the average number of cigarettes smoked each day by subjects in each group and obtain the following data. After two weeks, treatment one subject smoked 10 cigarettes per day, treatment subject two smoked ten per day and treatment three subject smoked 15. After four weeks, subject one smoked 6 per day, subject two smoked 12 and subject under treatment three smoked 9. After eight weeks subject one smoked 3 per day, subject two smoked 6 and subject three smoked three per day. Based on the above information, you can conclude that there are. a. main effects of treatment type and duration only. b. main effects of treatment type and duration and interaction effects. c. main effects of treatment type only. d. interaction effects only.

b

Zelda obtains a score of 41 on a test that has a mean of 50 and a standard deviation of 6. If the raw scores are changed so that the test now has a mean of 100 and a standard deviation of 12, Zelda's raw score would be equal to. a. 91. b. 82. c. 41. d. none of the above.

b

divides observations into categories and provides info on the order of those categories.a)interval b)ordinal c)ratio d) nominal e)ratio

b

At __months, infants begin to search for a hidden obj but reach for it in the last place they found it even when they've seen it has moved to another location a) 4 to 8 b) 8 to 12 c) 12 to 18 d) 18 to 24

b) 8 to 12

Restlessness, psychomotor agitation, flushed face, diuresis, rambling speech, and muscle twitching are most suggestive of: a) Alcohol Withdrawal b) Caffeine Intoxication c) Cocaine Intoxication d) Hyperthyroidism

b) Caffeine Intoxication

In mental health services, the research has found: a) Hispanic- and Asian-Americans overutilize them, while African-Americans underutilize them. b) Hispanic- and Asian-Americans underutilize them, while African-Americans overutilize them. c) Hispanic-, Asian-, and African-Americans overutilize them. d) Hispanic-, Asian-, and African-Americans underutilize them.

b) Hispanic- and Asian-Americans underutilize them, while African-Americans overutilize them.

internal representations of objects & object relations a)self image b)introjects c)archetypes d) interjects

b) Introjects

Attention deficits in ADHD are conceptualised as the consequences of: a) an inability to focus attention for an extended period of time. b) a marked inability to regulate attention. c) inadequate cognitive flexibility. d) lower-than-normal levels of task-related motivation.

b) a marked inability to regulate attention.

As a treatment for Tourette's, the SSRIs : a) are more effective than haloperidol or pimozide for alleviating tics b) are less effective than haloperidol or pimozide for alleviating tics but are useful for reducing obsessive-compulsive symptoms that often accompany this disorder c) cannot be used in conjunction with haloperidol or pimozide because of the risk for a toxicity syndrome d) are contraindicated because of their exacerbating effects on tics

b) are less effective than haloperidol or pimozide for alleviating tics but are useful for reducing obsessive-compulsive symptoms that often accompany this disorder

Classic migraine: a) involves temporary loss of movement of the eyes b) begins with an "aura" c) is the most common type of migraine d) is relatively brief in duration but recurs frequently

b) begins with an "aura"

To determine the degree of assoc btwn 2 variables when one is continuous and the other is an artifical dichotomy? a) contingency b) biserial c) phi d) Spearman

b) biserial

Structural equation modeling is used to: a) assess the length of time to the occurrence of a critical event b) evaluate the causal (predictive) influences of multiple latent factors c) determine, for a sample of people, how many types the sample represents d) identify natural grouping among a collection of observations

b) evaluate the causal (predictive) influences of multiple latent factors --identify underlying (latent) factors that relate to a set of measured variables and the nature of the causal relationships btwn them

An inverse agonist works by: a) blocking a neurotransmitter's access to the site b) exerting the opposite effect of a neurotransmitter. c) facilitating the action of a neurotransmitter at the site. d) mimicking the effect of a neurotransmitter.

b) exerting the opposite effect of a neurotransmitter.

reflects optimism, confidence & contributes to the welfare of others a)mistaken life style b)healthy life style c)open life style d) style of life

b) healthy life style

A test has high validity coeff for whites, Af-Ams, and Asian-Ams, but low validity for Native Ams. The race is a: a) suppressor variable b) moderator variable c) distractor variable d) uncontrolled variable

b) moderator variable --moderates relationship btwn predictor and criterion

Dissociative amnesia is associated with ______ amnesia which occurs when the individual is able to recall only some of the events that occured during a defined period of time a) localized b) selective c) systematized d) transient

b) selective -inability to recall some but not all events during that time

____ (Heider, 1958) is based on the principle of cognitive consistency, which proposes that having inconsistent attitudes produces a state of discomfort that leads to attitude change.

balance theory

ADHD has been linked to lower-than-normal activity in the __________ a) SCN b) somatosensory cortex c) basal ganglia d) hippocampus

basal ganglia

Repeated exposure to alcohol during prenatal dvpt is most likely to negatively impact the: a) hypothalamus, thalamus, and parietal lobes. b) medulla, thalamus, and temporal lobes. c) basal ganglia, hippocampus, and frontal lobes. d) brainstem, amygdala, and frontal lobes.

basal ganglia, hippocampus, and frontal lobes.

the _________ refers to the tendency to ignore relevant statistical data when making a probability judgment and to rely, instead on irrelevant info: a) Von Restorff effect b) Forer effect c) availability heuristic d) base rate fallacy

base rate fallacy

Some people maintain beliefs about themselves and others that cause them emotional distress. Alfred Adler referred to these beliefs as: a) fictional goals b) basic mistakes c) activating events d) logical fallacies

basic mistakes

According to Ajzen's theory of planned behav, a person's ___________ is directly fluenced by 3 factors-attitude toward the behav, subjective norms and perceived behav control a) behavioral consistency b) behavioral intention c) locus of control d) self-efficacy

behavioural intention

A therapist with an etic perspective a) believes that mental disorders may be manifested differently by people from different cultural groups. b) believes that mental disorders are manifested in similar ways by all people, regardless of their cultural group. c) adopts a psychodynamic model to describe the causes of mental disorders. d) adopts a behavioral model to describe the causes of mental disorders.

believes that mental disorders are manifested in similar ways by all people, regardless of their cultural group; etic=universal

Tourette's disorder has an age of onset of_______ a) over 14 b) 15 c) 16 d) below 18

below 18

pattern of viewing themselves as equally American and ethnic or as more American, represents _____. a) blended biculturalism, b) alternating biculturalism, c) separated. d)marginalization

blended biculturalism

Biculturalism among African-American and Mexican-American adolescents was recently examined by J. S. Phinney and M. Devich-Navarro, 1997. These investigators found that the adolescents in their sample exhibited one of three patterns:

blended biculturalism, alternating biculturalism, or separated.

the most likely side effects of propranolol are: a) bradycardia and depression b) tachycardia and mania c) tachycardia and tremor d) bradycardia and hypersomnia

bradycardia and depression (propranolol is a beta-blocker)

"Privilege" refers to a client's legal right to have confidential communications protected from disclosure. The situations that legally allow a waiver of privilege vary from state to state. However, generally speaking, which of the following is true about privilege. 1. Privilege is waived when the information is requested at a proceeding initiated by the client in a criminal action to determine his or her sanity. 2. Privilege is waived when the therapist suspects that a minor (e.g., the client or child of the client) has been the victim of child abuse. 3. Privilege is waived when the therapist has reasonable suspicion to believe that the client is a danger to another person. a. 3 only. b. 2 and 3 only. c 1,2 and 3. d. none of the above.

c

42. A patient with which of the following Personality Disorders is most likely to exhibit symptoms that could be erroneously interpreted as signs of depression a. Avoidant Personality Disorder. b.Obsessive-Compulsive Personality Disorder. c. Dependent Personality Disorder. d. Narcissistic Personality Disorder.

c

A 19-year old male who grew up in the inner-city receives a WAIS-R Verbal IQ score that is 20 points higher than his Performance IQ score. This discrepancy suggests. a. a learning disability. b. an educational deficit. c. right hemisphere damage. d. an invalid test.

c

A client has missed the last three appointments and has not paid for the past five. You receive a letter from a state in-patient facility requesting information about the client who is now a patient there. The request includes a valid release of information. According to the Ethics Code, you are acting ethically in this situation if you. a. notify the client that you will release the information only after an acceptable arrangement has been made for his unpaid bill. b. agree to release only a summary of your records until an acceptable arrangement has been made for his unpaid bill. c. provide the hospital with the information it has requested. d. use your own discretion as to the best course of action.

c

A client you have been seeing for two months suddenly informs you that this will be his last session. You believe this is not a good time for the client to stop therapy and are concerned about the potential negative consequences of doing so. You should. a. allow the client to quit since he has the right to do so. b. provide the client with appropriate referrals. c. discuss with him the reasons you believe he should continue and recommend that he re-think his decision. d. inform him that you are not responsible for any negative consequence of terminating prematurely.

c

A correlation between X and Y of .60 represents BLANK as much shared variability asa correlation of .30. a. twice. b. three times. c. four times. d. six times.

c

A factorial experimental design is used when: a) there is low external validity b) there is only one independent variable c) there is more than one level of independent variable d) there is more than one dependent variable measured

c

A feminist therapist is least likely to advocate the use of which of the following as a therapeutic technique. a. self-disclosure. b. sex-role analysis. c. diagnosis. d. role-modeling.

c

A mother of a newborn is most likely to say that, based on the nature of her baby's cry, she can tell if her baby is. a. hungry, tired, or uncomfortable. b. hungry, thirsty, or tired. c. hungry, angry, or in pain. d. hungry, lonely, or irritable.

c

A person receives a score of 90 on a test with a mean of 100 and a standard deviation of 5. The corresponding z-score is ____, the T-score is ___ and the stanine score is about ___. a) 1, 30, 9 b) -1, 300 c) -2, 30, 1 d) -2, 30, 3

c

A psychologist is consulted by a medical doctor who is concerned about a patient in a pain clinic. The patient constantly asks for his pain medication and the nurses, who get tired of his barrage of requests, give him too much medication. To get the man to cut down on his requests, the psychologist is most likely to recommend. a. giving him a sedative to keep him quiet. b. giving him his medication after every twenty requests. c. giving him his medication once every four hours. d. giving him higher but less frequent doses of medication.

c

A researcher designs a study in which subjects will be hospitalized mental patients. The hospital research committee has approved the project, which involves several treatments, some of which include experimental drugs that may cause harmful side effects. Before beginning the study, the researcher should. a. obtain informed consents from the patients if possible. b. obtain informed consents from the patients' guardians. c. obtain informed consents from the patients' guardians and assent from the patients. d. none of the above are necessary since the study has been approved by the hospital's research committee.

c

A researcher investigating the chemical correlates of memory would be most likely to find that administration of which of the following would improve the memory of elderly adults. a. L.G.N. b. T.H.E. c. R.N.A. d. G.A.B.A.

c

A researcher predicts that a six-month course of play-oriented psychotherapy will change the mean reading achievement scores of eight-year-old children. This would be a: a) null hypothesis b) one-tail hypothesis c) two-tail hypothesis d) alternative hypothesis

c

A researcher wants to test the hypothesis that female attorneys earn less per year than the national average for attorneys. From a government study, the researcher learns that the national average income for attorneys is $85,000. The researcher obtains a sample of 25 female attorneys and determines that their mean income is $72,000. The researchers will then use the: a) t-test for independent samples b) MANOVA c) one sample t-test d) Mann-Whitney U

c

A strategic family therapist is most likely to use which of the following therapeutic techniques in dealing with a husband and wife who complain that they are constantly arguing with each other. a. obtain a detailed family history from each partner. b. ask the couple to identify the benefits they obtain from arguing. c. tell the couple to argue for at least two hours each evening. d. work individually with the partner who is most "differentiated".

c

A test developer assesses the reliability of an achievement test using the test-retest method. She obtains a reliability coefficient of .58. The researcher should consider this coefficient to be. a. high for this type of test. b. about average for this type of test. c. low for this type of test. d. high for any type of test.

c

A test is developed to evaluate students' comprehension of information presented in a high school history class. When the test is administered to three classes of students at the end of the semester, all of the students perform very poorly on the test. This suggests that the test may have poor. a. concurrent validity. b. incremental validity. c. content validity. d. discriminant validity.

c

According to Erikson's theory of psychosocial development, middle-aged adults are in which stage of development? a. ego integrity versus despair. b. initiative versus guilt. c. generativity versus stagnation. d. industry versus inferiority.

c

According to Sue and Zane (1987), in terms of ensuring good treatment outcomes, the most important factor in culturally-sensitive therapy is. a. demonstrating cultural knowledge. b. using culture-specific techniques. c. using techniques that establish credibility. d. being authentic and empathic.

c

According to the interference hypothesis, normal aging is associated with a diminishing ability to screen out irrelevant distractions. Assuming that this hypothesis is true, this has the greatest implications for which aspect of attention? a. divided. b. sustained. c. selective. d. episodic.

c

Al A., a lonely, single, middle-aged psychologist and college professor, is invited to a play by a graduate student, Betty B., who Al finds sexually attractive. Betty has returned to school after a 15-year absence and has been divorced for three years. If Al accepts Betty's invitation, he will. a. be acting unethically because the Ethics Code prohibits professors from dating students. b. not be acting unethically as long as he doesn't become sexually involved with Betty. c. not be acting unethically as long as Betty is a graduate student in a department other than the one Al teaches in. d. not be acting unethically because this situation is not covered by the Ethics Code.

c

Although a number of longitudinal studies suggest that assessment centers have a high degree of predictive validity, some critics have argued that the high validities reported by these studies are actually the result of. a. a "contrast effect". b. the Hawthorne effect. c. criterion contamination. d. sample size.

c

Among bilingual speakers, those who exhibit "code-switching". a. tend to obtain lower scores on measures of reading comprehension. b. have less trouble switching between the grammatical rules of the majority and minority languages. c. go back and forth between languages during the course of a conversation as a way to better express themselves. d. unconsciously confuse the majority and minority languages while speaking, especially in high-stress situations.

c

An OD change intervention is most likely to be successful when. a. an internal (rather than an external) change agent is used. b. an external change agent is used but he/she communicates directly only with the company's managers. c. an external change agent is used and he/she communicates directly with both managers and subordinates. d. an external change agent is hired to act as a representative of subordinates only.

c

An adolescent is brought to therapy by her mother who says the girl has recently become "another person." She says her daughter used to be easy-going and easy to get along with and was well-liked at school by her peers and teachers. However, now her daughter is constantly irritable, is smoking and drinking, and has started having trouble at school. When the therapist interviews the girl, he learns that she is engaging in high-risk sexual behavior, but the girl feels that nothing bad can ever happen to her. Based on these symptoms, the best diagnosis is. a. ADHD. b. Conduct Disorder. c. Bipolar 1 Disorder. d. Bipolar II Disorder.

c

An elderly client displays paranoid ideation. In addition to providing a safe, stable environment, treatment for this client would most likely include which of the following. a. ECT. b. peer counseling. c. a phenothiazine. d. methylphenidate.

c

As a treatment for Obsessive-Compulsive Disorder, hypnosis. a. is contraindicated because it often paradoxically exacerbates symptoms. b. has been found to be more effective for compulsions than obsessions. c. may have clinical utility, especially when combined with other behavioral techniques. d. is as effective or, in some cases, more effective than other behavioral techniques.

c

IQ scores, the Fahrenheit or Celcius scale and scores on standardized tests are examples of: a) nominal data b) ordinal data c) interval data d) ratio data

c

Bill B., a 34-year old small business owner, has trouble completing tasks at home and work, frequently "blows his fuse" at family members and employees, and has his wife do all the paperwork at the office because he doesn't have the patience for it. His wife says Bill never listens to her and often does things without thinking about the consequences. When asked about his past, Bill says that he often got in trouble while he was in school. He frequently got into fights and, in high school, was arrested several times for drugs and drunk driving. All through school, his teachers said he wasn't living up to his potential. Although Bill no longer uses illegal drugs, he often drinks heavily on weekends. Based on these symptoms, the most likely diagnosis for Bill is. a. Antisocial Personality Disorder. b. Borderline Personality Disorder. c. ADHD. d. Bipolar Disorder.

c

Dr. Colombu, a licensed psychologist, is working in a community mental health center. The parents of a 15-year old girl bring their daughter for counseling. The parents will, of course, be paying Dr. Colombu's fee. As an ethical psychologist, Dr. Colombu should. a. caution the girl that, since she is a minor, anything she says may be told to her parents. b. advise the parents that he is required to maintain confidentiality and that he will not be able to advise them of anything the girl says during therapy sessions without her consent. c. clearly establish with the girl and her parents prior to therapy what the policy will be with regard to sharing information with the parents. d. use his discretion with regard to what information to reveal to the girl's parents and what information to keep confidential.

c

Due to ____________, cross-sectional studies tend to _____________ true age-related declines in performance. a. practice effects and underestimate b. intergenerational effects and underestimate c. cohort effects and overestimate d. order effects and overestimate

c

Electroconvulsive shock therapy. a. is as or more effective than drug therapy for acute Schizophrenia. b. has been found to be more effective in treating Mania than Depression. c. is considered useful for treating Depression, especially when the Depression involves suicidal preoccupation. d. is no longer used as a treatment for Depression because of its substantial impact on memory processes.

c

Failure to initiate a sequence of responses (a "behavioral chain") is often a problem because. a. the contingencies (reinforcers) have not been consistently applied. b. habituation has made the reward less desirable. c. the final (primary) reinforcer is remote. d. the final consequence is unknown.

c

For women suffering from PMS, the most severe physical and emotional symptoms usually occur. a. during the last days of menses. b. four to ten days prior to menses. c. during the week prior to and the first few days of menses. d. during the first three to four days of menses.

c

From the perspective of Rational Emotive Therapy. a. irrational beliefs are acquired primarily through social learning processes. b. irrational beliefs are acquired largely through the process of selective reinforcement. c. people are biologically prone to the acquisition of irrational beliefs. d. people adopt irrational beliefs as the result of early traumatic events that are still unresolved.

c

If a researcher's hypothesis were counter-intuitive and contradicted previous research or theory (ie. "cold fusion"): a) a Type I error is more serious than a Type II error and one would want to set the alpha high b) a Type I error is less serious than a Type II error and one would want to set the alpha low c) a Type II error is less serious than a Type I error and one would want to set the alpha low d) a Type II error is more serious than a Type I error and one would want to set the alpha high

c

If you are most concerned about the reliability of an achievement test you will be administering to high school freshmen, which one of the following kinds of tests would you be least likely to use. a. fill-in-the-blank. b. four-alternative multiple-choice. c. true-false. d. essay.

c

In a normal distribution, the: a) mean and the mode are equal only b) mean and the median are not equal c) mean, median and mode are equal d) mean, median and mode are not equal

c

In a positively skewed distribution, the median is: a) greater than the mean and the mode b) greater than the mean but less then the mode c) greater than the mode but less than the mean d) equal to the mean and the mode

c

In contrast to traditional psychodynamic psychotherapy, the brief psychotherapies. a. focus on current phenomenon only. b. discourage the development of transference. c. focus on specific conflicts. d. all of the above.

c

In the assessment of cognitive abilities with a standardized test, you would "test the limits". a. before administering the test. b. before administering each subtest. c. after administering the test. d. whenever it seems appropriate to do so.

c

In vivo flooding has been found to be an effective treatment for Agoraphobia. Studies investigating the effects of in vivo flooding for this disorder suggest that. a. frequent, brief exposures are more effective than less frequent, longer exposures to the feared stimuli. b. people with high arousability are more responsive to in vivo treatments than people with low arousability. c. high anxiety provocation is not the key factor in the effectiveness of in vivo flooding. d. courtterconditioning is a necessary component of in vivo flooding as a treatment for agoraphobia.

c

Increasing age is least likely to have which of the following effects on a man's sexual response cycle. a. the time to achieve an erection will increase. b. complete penile erection may not occur until just prior to orgasm. c. the resolution phase and refractory period will both increase. d. all of the above.

c

Increasing alpha has the effect of ________ power and the probability of making a ____________ error increases. a) decreasing, Type I b) decreasing, Type II c) increasing, Type I d) increasing Type II

c

Kohlberg's theory of moral development views it as being. a. the result of parental expectations. b. biologically innate and universal. c. promoted by peer interactions. d. the result of school and church teachings.

c

Multiple-baseline designs are used when the use of a ___________ design is not possible. a) "AB" b) longitudinal c) reversal d) correlational

c

Of 100 students surveyed on a particular college campus, 50 voted Republican and 50 voted Democratic in the last election. On another college campus, 60 voted Republican and 40 voted Democratic. The correct statistical test to use to determine if the difference in voting preferences at the two colleges is significant is which of the following. a. two-way ANOVA. b. t-test. c. chi-square test. d. Mann-Whitney U test.

c

Of the following, which is most true about adolescents who have a history of repeated suicide attempts. a. the failure of their attempts is usually due to a lack of access to lethal means. b. their continued threats to commit suicide are of minimal concern because very few adolescent repeat attempters actually complete suicide. c. the real purpose of their suicide attempts is often to exert power in situations in which they feel powerless. d. their suicide attempts are often a self-imposed extension of the physical abuse they have experienced since early childhood.

c

One problem associated with subjective measures of performance is that they are susceptible to certain biases or errors. For example, the leniency bias occurs when an evaluator consistently overrates all of the employees being appraised. Which of the following is the most effective method for reducing such appraisal errors. a. graphic checklists. b. Likert scales. c. forced-choice scales. d. Guttman scales.

c

Order Effects, which are a threat to external validity, are also known as: I. Carryover Effects II. Multiple Treatment Interference III. Demand Characteristics a. I only b. II only c. I and II d. I, II and III

c

Parametric and nonparametric tests share which one assumption? a) normal distribution b) homogeneity of variance c) unbiased sample d) use of nominal data

c

Presence of which of the following symptoms suggests that the appropriate diagnosis is Alcohol Withdrawal as opposed to Cocaine Withdrawal? a. insomnia. b. dysphoria. c. seizures. d. psychomotor agitation.

c

Research on mnemonic devices suggests that they are. a. superior to rote rehearsal in terms of both short-and long-term retention. b. superior to rote rehearsal and elaboration in terms of both short-and long-term retention. c. superior to rote rehearsal in terms of short-term but not long-term retention. d. superior to rote rehearsal and elaboration in terms of short-term but not long-term retention.

c

Research on working women suggests that combining work and family roles is most associated with. a. stress-related illness. b. reduced involvement in family roles. c. enhanced self-esteem. d. low job commitment.

c

Research suggests that, in comparison to popular children, less popular children tend to be all of the following except. a. less intelligent. b. less physically attractive. c. less friendly and sociable. d. less cooperative.

c

The ___________ the standard deviation, the _________ that scores in a distribution are likely to deviate from the mean. a) lower, more b) higher, less c) higher, more d) none of the above

c

The most powerful method for controlling extraneous variables is via random ________________ : a) representation b) selection c) assignment d) sampling

c

To be consistent with the Federal Educational Rights and Privacy Act (aka the Buckley Amendment), a school psychologist must keep in mind that. a. it is necessary to obtain the signed consent of a student's parents before releasing any information from the student's file to officials of another school where the student is planning to enroll. b. student records, including identifying information, must be released to designated federal and state educational authorities if needed in connection with the evaluation of federally-sponsored educational programs. c. parents have the right not only to inspect and review their children's school records but also to challenge the contents of records. d. all of the above.

c

To ensure that a consent to treatment form is truly an "informed" consent, a therapist should. a. make sure that the client reads the entire form before signing it. b. warn the client of the potential risks involved in signing the form. c. make sure the client understands the information contained in the form before he or she signs it. d. warn the client of the potential limits of confidentiality.

c

To evaluate an intervention designed to increase the time a hyperactive child stays on-task when doing schoolwork, the best technique would be. a. situational sampling. b. event recording. c. interval recording. d. sequential analysis.

c

What happens to the shape of the distribution when raw scores are transformed into percentile ranks? a) It is positively skewed b) It is negatively skewed c) It is a nonlinear transformation d) It is a linear transformation

c

When a psychologist is requested by a court to provide psychological assessment of a defendant for a competency hearing, the psychologist is primarily asked to evaluate. a. the defendant's psychic functioning as it relates to his or her psychological development. b. the defendant's vocational history and probability of rehabilitation. c. the defendant's ability to understand and participate in a court proceeding. d. the psychological status of the defendant at the time of the alleged crime.

c

When administering a standardized 60-item test that has a time limit of 60 minutes, a test administrator accidentally gives the examinees only 50 minutes to complete the test. To get accurate test results, the test administrator should. a. add 10 points to each examinee's score. b. administer the last 10 items to examinees on another day. c. administer an alternate form of the test to examinees. d. do any one of the above.

c

When deciding on an intervention plan for a client suffering from agoraphobia, it is important to keep in mind that which of the following seems to be the key element in treating this disorder. a. counterconditioning. b. progressive relaxation. c. in vivo exposure. d. cognitive restructuring.

c

When faced with an "undifferentiated family ego mass," a family therapist who has adopted a Bowenian approach would be most likely to do which of the following. a. help differentiate family members by becoming emotionally triangulated into family dyads. b. help differentiate family members by using behavioral and cognitive-behavioral techniques that promote individuation. c. work first with the most differentiated family member to help him or her achieve greater individuation. d. work first with the "identified patient" to help him or her become more differentiated.

c

When using Beck's cognitive approach to therapy with a client with Obsessive-Compulsive Disorder. a exposure in vivo or in imagination is not considered beneficial because it produces strong emotions that interfere with cognitive work. b. exposure in vivo or in imagination is conducted only when the symptoms are so severe that they are interfering with cognitive work and must be alleviated quickly. c. exposure in vivo or in imagination is considered useful for eliciting "hot cognitions" related to the disorder so that they are available for cognitive work. d. exposure in imagination is considered more useful than exposure in vivo because it elicits target cognitions without the strong emotions that may interfere with cognitive work.

c

When using the technique known as the Premack Principle, the reinforcer is. a. applied intermittently. b. a generalized conditioned reinforcer. c. a behavior that occurs frequently. d. a stimulus that naturally elicits the desired behavior.

c

Which of the developmental changes is most likely to occur in males between the ages of 40-45. a. a decrease in work satisfaction. b. significant personality change and emotional disturbance due to the "mid-life" crisis. c. a shift in perspective from "time-since-birth" to "time-left-to-live". d. peaceful acceptance of the impending black doom of the grim reaper.

c

Which of the following is not a t-test? a) one sample t-test b) t-test for independent samples c) t-ratio test d) t-test for correlated samples

c

Which of the following medical conditions is least likely to be a cause of an anxiety disorder due to a general medical condition. a. thyroid dysfunction. b. epilepsy. c. diabetes mellitus. d. malignancy.

c

__________________ research does not have any internal validity. a) Developmental b) Quantitative c) Correlational d) Qualitative

c

Which of the following statements is least true of individuals who have been diagnosed with Autistic Disorder. a. only a small percentage of them go on as adults to live and work independently. b. signs of the disorder may be present in early infancy. c. a subset of individuals with the disorder, "those who do have the cognitive ability to eventually function independently", "grow out" of the disorder in adulthood. d. they are sometimes excellent in tasks involving long-term memory.

c

Withdrawal from which of the following substances is least likely to involve hallucinations. a. a sedative. b. an anxiolytic. c. cocaine. d. alcohol.

c

You are conducting a study to investigate the phenomenon known as "blocking." Consequently, you will. a. first present the CS prior to the US; then, once the CR is established, present the US prior to the CS. b. first present the CS prior to the US; then, once the CR is established, present the CS prior to a sec and US. c. first present the CS prior to the US; then, once the CR is established, present the original CS simultaneously with a second CS prior to the US. d. first present the CS prior to the US; then, once the CR is established, present a second CS prior to the original CS.

c

You are using in vivo aversive conditioning to reduce alcohol consumption in chronic alcoholics. To maximize the effectiveness of the treatment, the aversive stimulus (electric shock) should be applied. a. while the client is thinking about taking a drink. b. right before the client takes a drink. c. immediately after the client begins drinking. d. after the client has finished a drink

c

You are using neuroleptics to treat a schizophrenic patient. If you were to rely on research into their use in the treatment of schizophrenia, you would discover which of the following. a. thorazine tends to be the least effective drug for treating schizophrenia. b. the thioxanthenes tend to be the most effective drugs for treating schizophrenia. c. the selection of an appropriate neuroleptic depends largely on the patient's responsivity. d. no neuroleptic has been found to be particularly effective for the positive symptoms of this disorder.

c

You have just started seeing a couple in conjoint therapy and believe that the husband is physically abusing his wife. Most likely you will. a. wait for the husband or wife to bring up the issue of abuse. b. say that you want to see the wife in individual therapy. c. ask the couple how they deal with conflict and disagreement. d. ask the couple to sign a "no-violence" contract.

c

____________ refers to pre-existing subject factors that account for scores on a dependent variable. a) history b) testing c) selection d) maturation

c

A student obtains a % rank on the verbal test of the McCathy Scales of Children's Abilities. If the student's score had been one SD higher, her percentile rank would have been approx: a) 90.0 b) 95.0 c) 98.0 d) 99.9

c) 98.0 (97.5=2SD above mean)

In what way do Asperger's and Autism differ? a) Asperger's do not display repetitive behavs b) Aspeger's do not display impaired social interactions c) Asperger's do not significant impairments in language

c) Asperger's do not significant impairments in language-they are able to speak earlier-by age 2.5yrs

Which of these has an onset at 2 years after a period of normal dvpt? a) autism b) Rhett's c) Childhood disintegrative disorder d) ADHD

c) Childhood disintegrative disorder

Which best addresses the issue of interactional justice in an organisation? a) Did employees consider the outcomes of an organizational decision reasonable and just? b) Was the organizational procedure used to make the decision fair? c) Did employees feel they were treated fairly when informed about the decision? d) Were the employer's reasons for making the decision reasonable and valid?

c) Did employees feel they were treated fairly when informed about the decision?

In item response theory, item difficulty is defined: a) by the number of examinees in the tryout sample who answered the item correctly b) by the ability of the item to discriminate between examinees who fail or succeed on an external criterion c) by the probability that an examinee with a given level of the ability measured by the test will answer the item correctly d) by the probability that an examinee will pass the item given his/her predicted score on the test

c) by the probability that an examinee with a given level of the ability measured by the test will answer the item correctly

Which ability would be least useful for assessing the percep abilities of a 3-4month old infant? a) sucking b) heart rate c) head turning d) reaching

c) head turning-this is used in infants aged 5.5-12months

Which is least likely to be an initial symptom of multiple sclerosis? a) blurred or double vision b) fatigue that worsens in the afternoon c) impaired memory and attention d) heaviness, weakness, or loss of sensation in the legs

c) impaired memory and attention --these develop later

When requested to act as a fact witness in a court trial involving a former therapy client, a psychologist: a) is not required to obtain a waiver of confidentiality from the client b) is required to obtain a waiver only if the request comes from the court rather than from the client's attorney c) may or may not be required to obtain a waiver of confidentiality from the client d) should provide information about the client only with a court order

c) may or may not be required to obtain a waiver of confidentiality from the client -only provide info without the client's waiver or a court order

When a client belongs to a different ethnic or cultural group comes to therapy, a therapist is best advised to: a) treat the client like any other person unless there is reason to do otherwise b) alter the diagnostic and treatment processes to coincide with the client's ethnic or cultural background c) recognize that ethnic and cultural forces can have an impact on both the therapist's and the client's psychological processes d) refer the client to another therapist from the same ethnic or cultural background whenever it is feasible to do so

c) recognize that ethnic and cultural forces can have an impact on both the therapist's and the client's psychological processes

when using split-half method to estimate reliab of a speed test: a) the split of the test into halves should be done in a random way b) the split of the test into halves should be done on the basis of odd-versus even-numbered items c) the resulting reliability coefficient will overestimate the reliability of the test d) the resulting reliability coefficient will underestimate the reliability of the test

c) the resulting reliability coefficient will overestimate the reliability of the test ----measures of internal consistency overestimate reliab of a speed test

You receive a subpoena from the court requesting that you supply it with copies of your records on a former client. Apparently, the client has been charged with a crime and he is using "insanity" as hisdefense. You should: a) refuse to turn over the records to the court because to do so would be a breach of the client's confidentiality b) turn over the records only if you believe they will help the client's case c) turn over the records to the court unless the client changes his plea d) turn over the records to the court regardless of the client's plea

c) turn over the records to the court unless the client changes his plea

A DSM diagnosis of MR requires an IQ test thatis at least: a) one-half standard deviation below the mean b) one standard deviation below the mean c) two standard deviations below the mean d) three standard deviations below the mean

c) two standard deviations below the mean

A person with apperceptive visual agnosia: a) can name a familiar object she sees but does not know how to use it. b) can name a familiar object when she does so spontaneously but not when she is asked to do so. c) cannot name a familiar object she sees but may recognize it when it is placed in her hand and cannot copy or match a drawing of the object. d) cannot name a familiar object she sees but may know what it is used for and can copy or match a drawing of the object.

cannot name a familiar object she sees but may recognize it when it is placed in her hand and cannot copy or match a drawing of the object.

Persuasion that occurs via the ______route is the result of careful consideration of the content of the message and any resulting attitude change is likely to be enduring and predictive of future behavior.

central

elaboration likelihood model identifies two routes to persuasion which are

central and peripheral

Soon after staring to take an antidepressant, a young woman develops several troubling symptoms including dry mouth, urinary retention, constipation and blurred vision. Which of the following would be most useful in alleviating these side effects? a) caffeine b) antihistamine c) dopamine agonist d) cholinergic agonist

cholinergic agonist

Which accounts for the largest proportion of severe mental retardation a) chromosomal abnormalities b) injury to the dvping brain c) prenatal exposure to teratogens d) endocrine and metabolic factors

chromo abnorms

In all states, the __ is the holder of the privilege, which means that it is up to the client to decide whether to exercise or waive the privilege.

client

the state in which a person's thoughts and behaviors match his or her beliefs and the expectations of others

cognitive consistency

systematic errors in thinking/reasoning forms the link b/w dysfunctional schemas & automatic thoughts.

cognitive distortions

Research into GAD treatment suggests that the most effective intervention is: a) flooding b) systematic desensitization c) stress inoculation d) cognitive therapy

cognitive therapy

hardiness as a combination of three attitudes ___, ___ & ____ that together provide the courage and motivation needed to turn stressful circumstances from potential calamities into opportunities for personal growth.

commitment, control, and challenge

A(n) ____________ seizure begins in a localised area of hte brain, spreads to other areas, produces clouding or loss of consciousness, and ordinarily lasts from 1-2mins a) absence b) atonic c) simple partial d) complex partial

complex partial

Achieving a predetermined level of measurement accuracy with as few test items as possible is the goal of: a) computer adaptive testing b) dynamic assessment c) performance based assessment d) curriculum-based assessment

computer adaptive testing

A 50 year old man with dorsal convexity dysexecutive syndrome is likely to exhibit: a) concreteness and perseveration. b) distractibility and emotional lability. c) sparse verbal output and lower extremity weakness. d) occasional hallucinations and a "dazed" feeling

concereteness and perservation

when a person discovers that positive regard is conditional not unconditional

conditions of worth

in supervision the supervisor is the same profession as the supervisee & has administrative responsibility & is in a position of power/authority over the supervisee. Consultants can advise, but consultees have to decide to take the information.

consultation v. supervision

The law of ___________ states that learning depends on the proximity of stimuli in space and time.

contiguity

The analysis that compaires the relative costs and outcomes of 2 or more interventions is referred to as a) cost offset analysis b) cost effectiveness analysis c) cost utility analysis d) cost benefit analysis

cost effectiveness analysis

The analysis of cost savings in medical care resulting form the provision of effective mental health services is referred to as a) cost offset analysis b) cost effectiveness analysis c) cost utility analysis d) cost benefit analysis

cost offset analysis

Someone taking an MAOI shouldn't eat all EXCEPT a) yogurt. b) soy sauce. c) cottage cheese. d) over-ripe avocados

cottage cheese

Dr Bob is using a repeated measures design to compare the effects of different techniques for reducing free-floating anxiety. To control potential carryover effects Dr Bob should use which of the following?

counterbalancing

___ helps control carryover effects by presenting the levels of the IV to different participants in a different order:

counterbalancing

160. Recent research on goal-setting in organizations has shown that all of the following are true except. a. providing feedback on goal achievement is a necessary but not sufficient condition for improved job performance. b. overall, participation in goal setting is not as important as acceptance of goals. c. employees who are told to "do their best" will usually exhibit lower levels of productivity than employees who are given difficult goals. d. participation in goal-setting is more important for educationally-advantaged workers.

d

A 34-year old woman is often very irritable, has trouble controlling her anger, has a history of frequent job changes, and often complains that she is bored. Based on these symptoms, the best diagnosis is. a. Schizoid Personality Disorder. b. Narcissistic Personality Disorder. c. Histrionic Personality Disorder. d. Borderline Personality Disorder.

d

A __________ skewed distribution can be thought of as a "difficult test" where most scores fall at the _____ end of the distribution. a) negatively and high b) negatively and low c) positively and high d) positively and low

d

A distinctive feature of the MMPI-2 is its inclusion of several "validity scales." The purpose of one of these scales, the L (Lie) Scale, is to. a. obtain information about the examinee needed to derive a specific psychiatric diagnosis. b. provide information on the test itself for use in validation studies. c. check the examinee's propensity to view him or herself in an unfavorable manner so that a "correction factor" can be applied to the examinee's scores. d. measure the examinee's tendency to depict him/herself favorably so that the validity of the test results can be determined.

d

A feminist therapist is least likely to view which of the following as an important aspect or goal of therapy. a. identifying the sociopolitical and interpersonal forces that underlie the woman's "psychopathology". b. reconstructing the therapeutic process so that the traditional pattern of dependence is not recreated in the therapeutic relationship. c. using the therapeutic relationship as an opportunity to model appropriate behavior. d. identifying and integrating the masculine and feminine aspects of the woman's personality.

d

A girl has a punishing father who is in the army reserves. Eventually, the girl becomes extremely anxious whenever she sees a man in uniform. The girl's reaction to uniformed men is an example of. a. higher-order conditioning. b. negative transfer. c. pseudoconditioning. d. stimulus generalization.

d

A person with apraxia has. a. deficits in memory. b. trouble recognizing objects by touch. c. impaired procedural memory. d. difficulty carrying out intentional movements.

d

A split-brain patient would be able to do which of the following? a. say the word "spoon" after it is projected to his left visual field. b. repeat a series of letters that have been whispered in his left ear. c. identify an odor that has been presented to his right nostril. d. identify a spoon with his left hand after an image of a spoon is projected to his left visual field.

d

Sex of salespeople is an example of what type of measurement: a)interval b)ordinal c)ratio d) nominal e)ratio

d

A type of measurement that divides variables into unordered categories. a)interval b)ordinal c)ratio d) nominal e)ratio

d

A variable is measured on a ___ scale when it is divided into categories and the frequency of individuals in each category will be compared: a)interval b)ordinal c)ratio d) nominal e)ratio

d

A worker whose job consists primarily of handling customer complaints will be happier if she blames the customer's problems on. a. external, stable, and global factors. b. internal, stable, and specific factors. c. external, unstable, and global factors. d. external, unstable, and specific factors.

d

According to Erikson, at age two, the primary task of psychosocial development is to develop BLANK, and failure to successfully accomplish this task leads to BLANK. a. an ego identity; identity diffusion. b. a capacity for industry; a sense of inferiority. c. a sense of basic trust; suspicion and mistrust. d. a sense of autonomy; doubt and shame.

d

According to Freud's psycho dynamic theory, the origin of anxiety is. a. an inability to free oneself from symbiotic relationships. b. reactions to strangers during infancy. c. an inability to repress id impulses. d. an inability to resolve conflicts between the id, the superego, and reality.

d

After speaking at a workshop at a large university, Dr. A., a renowned expert on eating disorders, is approached by a woman who asks him about her daughter who she believes is beginning to exhibit symptoms of anorexia nervosa. Dr. A. gives the woman the phone numbers of three therapists in the area who have experience with this disorder. Six months later, Dr. A. finds out that the woman is planning to file a malpractice suit against him. During that time, her daughter received treatment from one of the therapists on Dr. A.'s list but became much worse and is currently hospitalized. The woman feels that Dr. A. is responsible for not adequately warning her of the potential course of her daughter's disorder. In terms of malpractice. a. Dr. A. is guilty because he had a "duty to warn" the woman about possible complications of anorexia nervosa. b. Dr. A. is guilty because he apparently did not give the woman adequate referrals. c. Dr. A. is not guilty because he gave the woman the names of several therapists. d. Dr. A. is not guilty because he didn't have a professional relationship with the woman or her daughter.

d

An 12-year old who has received a diagnosis of Oppositional Defiant Disorder is LEAST likely to be exhibiting which of the following symptoms. a. swearing. b. low frustration tolerance. c. alcohol use. d. lying and cheating in games.

d

An advocate of client-centered therapy would most likely view diagnosis as. a. the starting point of therapy. b. an ongoing process. c. a joint task of the therapist and client. d. unnecessary and possibly detrimental.

d

An industrial psychologist is hired by a company to assist in the development of a selection program for assembly-line workers. Based on her knowledge of the literature in this area, the psychologist is most likely to advise the company to use which of the following selection techniques. a. interviews and personal recommendations. b. interviews and selection tests. c. selection tests and assessment centers. d. biographical information and work samples.

d

Consider a study where one is interested in the association between gender and leadership style. If this were to be a true experiment, the researcher would have to randomly: a) assign subjects to specific leadership groups b) conduct assessments to determine what type of leadership style each subject has c) sample the business community d) assign subjects to either the "male or the "female" group

d

Dr. Able, a psychologist, has been seeing a client for five months and feels that the client may not be benefitting from treatment. When she mentions this to him, the client says he is quite satisfied with the progress of therapy. Based on this information, Dr. Abie's best course of action would be to. a. continue seeing the client in therapy since the client is satisfied. b. tell the client that she is ethically required to refer him to another therapist. c. obtain supervision to ensure that her treatment is effective. d. consult with another psychologist to discuss the situation.

d

Dr. Dither is hired by the court to conduct an evaluation of a family as part of a child protection matter. Based on Dr. Dither's evaluation and other evidence, the court determines that the child can remain at home with his family who will be provided with supervision and counseling. Six months after the case is heard in court. Dr. Dither is contacted by the mother who says she would like to begin therapy with him for issues unrelated to the child protection matter. According to APA's Guidelines for Psychological Evaluations in Child Protection Matters, Dr. Dither should. a. agree to see the woman in therapy since the child protection matter has been resolved. b. agree to see the woman in therapy only if her problem is, in fact, unrelated to the child protection matter. c. agree to see the women in therapy since his familiarity with her situation will be an advantage. d. refer the woman to a colleague.

d

Dr. Lane, a psychologist, is conducting a research study on peer pressure. Her subjects will be 120 fifth grade students. The study will involve a 30-minute interview with each student and observation of the students during their lunch hour. Before beginning the study, the psychologist should. a. obtain the consent of the students. b. obtain the consent of the students' parents. c. obtain the consent of appropriate school personnel. d. obtain the assent of the students and the consent of their parents.

d

Dr. Werner, a psychologist, is conducting a research study and obtains subjects for his study by placing advertisements in a local newspaper. Dr. Weiner informs potential subjects about all aspects of the study and tells them that they should not agree to participate unless they are willing to stay for its duration (six sessions over a six-week period), since once they sign up, they will not be permitted to withdraw from the study. According to APA's Ethics Code, the proposed arrangement is. a. clearly ethical. b. ethical as long as the withdrawal restriction is included in the informed consents that all subjects must sign. c. ethical as long as the withdrawal restriction has been determined to be crucial to the study. d. unethical.

d

If a researcher wanted to separate the effects of therapy from the effects of intelligence, as well as identify the interaction between the variables then what procedure would she use? a. matching b. ANCOVA c. holding the extraneous variable constant d. blocking

d

If one devised a new treatment for a deadly disease, making: a) a Type I error is more serious than a Type II error and one would want to set the alpha high b) a Type I error is less serious than a Type II error and one would want to set the alpha low c) a Type II error is less serious than a Type I error and one would want to set the alpha low d) a Type II error is more serious than a Type I error and one would want to set the alpha high

d

In a normal distribution of scores, about ___ of all scores will fall between the range set by the s.d. points -1 and +1. About ___ of all scores will fall between s.d. -2 and +2, and over ___ of scores will fall between s.d. -3 and +3. a) 16%, 48%, 99% b) 16%, 68%, 95% c) 48%, 68%, 95% d) 68%, 95%, 99%

d

In a one-group interrupted time-series design, the major threat to internal validity is _______________. a) maturation b) testing c) regression d) history

d

In avoidance conditioning, the initial fear that causes the avoidance response is the result of which of the following. a. negative reinforcement. b. punishment. c. stimulus generalization. d. classical conditioning.

d

In which of the following situations would an exception to the requirement for an informed consent to treatment not be an issue. a. medical or psychological emergency. b. incompetency. c. client waiver. d. court-ordered treatment.

d

Jung's theory differs from Freud's in all of the following ways except. a. Jung's perspective reflects a more optimistic view of human nature. b. Jung promoted a more active interchange between analyst and patient. c. Jung conceptualized the psyche as continuing to evolve throughout the lifespan. d. Jung replaced the concept of the ego with the concepts of the self and the ideal-self.

d

Parametric tests are not robust with regard to what? a) normal distribution b) mutual exclusivity c) homogeneity of variance d) independence of observations

d

Parametric tests are used for what kind of data? I. ordinal data II. nominal data III. interval data IV. ratio data a) I only b) I and II c) II and III d) III and IV

d

Research investigating outcome following a traumatic head injury suggests that within 12 to 18 months following the trauma BLANK percentage of total recovery of functions directly related to the activity of the brain can usually be expected. a. 20 to 30%. b. 40 to 50%. c. 60 to 75%. d. 80 to 95%.

d

Shortly after a school psychologist begins working for a small rural school district, she finds that there are a number of children in a class for retarded children who are emotionally disturbed but who appear to have an average or above average level of intelligence. The county has no class for emotionally disturbed children. The psychologist should. a. report the situation immediately to the psychology ethics committee in her state. b. report the situation immediately to the teacher's standards and practices commission in her state. c. threaten to quit if the children are not placed in a regular classroom. d. evaluate each student and recommend an appropriate action regardless of available options.

d

The Ethics Committee is investigating a complaint against a psychologist and requests, by letter, that you provide the Committee with information about the complainant who is a former client of yours. When you check your records, you realize that you stopped seeing this client nine years ago. As an ethical psychologist, you should. a. respond immediately to the Committee's request by supplying them with the information they have asked for. b. respond immediately to the Committee's request but provide them only with the information you feel is relevant to the complaint. c. respond to the Committee's request by informing them that the information you have about the client is obsolete. d. respond immediately to the Committee's request only after verifying that the client has signed a release of information.

d

The ________ only indicates if there is a significant difference somewhere among the means in the study. It does not indicate exactly which means significantly differ from one another. a) one sample t-test b) beta c) chi-square d) f-ratio

d

The mother of a 35-year-old man says that, for the past five weeks, he has been acting very uncharacteristically. He seems perplexed and nervous much of the time, his tone of voice varies between flat and shrill, and his speech is sometimes like "word salad." The mother says that sometimes it seems like he is listening to voices or sounds that she cannot hear. Based on this information, the best diagnosis is. a. schizophrenia, disorganized type. b. schizoid personality disorder. c. schizoaffective disorder. d. schizophreniform disorder.

d

The order in which one would use a statistical test is as follows: I. Data are collected II The obtained statistical value is compared to a tabled critical value III. The null and alternative hypotheses are stated IV. The data are analyzed through the use of the appropriate statistical test a) I, III, II, IV b) I, II, III, IV c) III, I, II, IV d) III, I, IV, II

d

The primary limitation of the __ is that the only mathematical operation that can be performed on the obtained data is to count the frequency of cases in each class.a)interval b)ordinal c)ratio d) nominal e)ratio

d

The research suggests that which of the following would be least useful for the treatment of alcoholism? a. social-skills training. b. stress management training. c. antidepressant medication. d. anxiolytic medication.

d

The target of an organizational development (OD) intervention depends, of course, on the nature of the organization's problems. However, most OD interventions share in common an overarching concern with. a. structure. b. function. c. task and technology. d. human/social relationships.

d

There are a number of widely-accepted myths about old age. Which of the following is not a myth? a. Older adults are more difficult and rigid than younger adults. b. Older adults are more similar than younger adults in terms of personality, cognitive skills, interests, etc. c. Older adults have higher rates of depression and anxiety than younger adults. d. Older adults tend to exhibit more stability in personality traits than younger adults.

d

Threats to internal validity include: I. maturation II. statistical regression III. demand characteristics a) I only b) I and III c) II and III d) I and II

d

Tolman's demonstration of latent learning suggests that. a. a student studying algebra in September may not learn algebra until he begins to study geometry. b. a student studying algebra will come to a sudden and whole understanding of it. c. a student will not learn algebra unless the proper incentive for learning is provided. d. a student might appear to know little about algebra until he takes an algebra test.

d

What happens to the shape of the distribution when raw scores are transformed into z-scores? a) It is positively skewed b) It is negatively skewed c) It is a nonlinear transformation d) It is a linear transformation

d

When a study has two or more dependent variables, what test is used? a) one-way ANOVA b) t-test for correlated samples c) factorial ANOVA d) MANOVA

d

When asked to recall an accident you witnessed two weeks ago, which of the following will be involved. a. procedural memory. b. semantic memory. c. representational memory. d. episodic memory.

d

When attempting to expand your private practice, it is important to keep in mind that. a. in-person solicitations are always unethical. b. solicitations of individuals receiving similar services from another professional are unethical. c. solicitations are unethical only when they include deceptive or misleading information. d. solicitations are unethical when they include deceptive information or when the person solicited is susceptible to undue influence.

d

Which of the following is true about the retrograde amnesia that occurs as the result of moderate to severe head injury? a. it is limited to recent long-term memories. b. it is limited to remote long-term memories. c. when it begins to resolve, the most recent memories return first. d. when it begins to resolve, the most remote memories return first.

d

Which one of the following is true regarding treatment for premature ejaculation. a. premature ejaculation is one of the more difficult sexual dysfunctions to treat. b. treatment usually involves temporary abstinence from sexual contact. c. treatment will involve helping the client maintain an erection for at least four minutes. d. treatment will probably include the use of the "stop-start" or "squeeze" technique.

d

With regard to bartering, APA's Ethics Code. a. clearly prohibits bartering of services only. b. clearly prohibits bartering of goods only. c. discourages bartering unless alternative methods of payment for services are not feasible. d. discourages bartering unless it is not clinically contraindicated and is not exploitative.

d

With regard to the effects of age on global job satisfaction, the best conclusion is that these variables have. a. a U-shaped relationship. b. an inverted U-shaped relationship. c. an inverse relationship. d. a positive relationship.

d

Work by Perry and Busey (1977) suggests that highly aggressive children. a. were exposed prenatally to high levels of androgens. b. are less sensitive to physical discomfort than less aggressive children. c. exhibited a disoriented/disengaged attachment pattern as young children. d. show little remorse after hurting another child.

d

You administer the MMPI-2 to several clients. One client, a 27-year old male, obtains a T score of 100 on the F scales. This means that. a. the client has attempted to "fake good". b. the client is insecure, intolerant of unconventional behavior, and has a lack of insight. c. you should use the F scale score as a correction factor for other scales. d. assuming no scoring errors, you should consider the test results invalid.

d

You are working as an industrial psychologist and are hired as a consultant by the owner of a medium-sized company who says she is having problems with employee productivity. As an advocate of "goal-setting theory," you are most likely to tell the owner that, to maximize the productivity of her workers, she should. a. let the workers set their own goals. b. let the workers set their own goals when the task is simple and structured, but assign goals when the task is complex and unstructured. c. assign the workers moderate goals that are within the capabilities of the least proficient workers. d. assign the workers relatively difficult goals and give them feedback about their attainment of those goals.

d

In a scatterplot, the regression lines ofr a test for 2 diff groups of examinees differ substantially in terms of slope. This suggests the test has: a) a lack of factorial validity b) a lack of convergent validity c) divergent validity d) differential validity

d) differential validity

The _________ law states that the frequency of responding matches the frequency of reinforcement.

matching

You receive a subpoena duces tecum from the court requesting that you provide it with information about one of your clients. Your best course of action would be to. a. immediately provide the information requested to the court. b. obtain the client's consent and then provide the information requested to the court. c. assert the privilege not to reveal confidential information and then provide the information requested to the court. d. assert the privilege not to reveal confidential information and provide the information only after obtaining the client's consent or being ordered to do so by the court.

d

__________ a ___________ null is the goal of almost all research. a) retaining, true b) retaining, false c) rejecting, true d) rejecting, false

d

Individual psych assumes that all behavior is goal directed and purposeful: a)systematic training for effective parenting b)systematic training for effective teaching c) collective unconscious d) deterministic

d)

What percentage of new mothers exhibit postpartum blues, which is less severe than postpartum depression and involves mood swings and sleep disturbances? a) 15 to 30% b) 25 to 40% c) 35 to 60% d) 50 to 80%

d) 50 to 80%

On the MMPI, a T-score of 70 means that an examinee's score is at the: a) 68th %ile b) 70th %ile c) 84th %ile d) 97th %ile

d) 97th %ile

Who is incorrectly matched to his view of dvpt? a) Carl Rogers: As long as a person doesn't experience distortion of denial of his/her self-structure, the person will naturally grow toward self-actualization b) Erik Erikson: People are propelled by psychological and social forces and will develop in healthy ways as long as they are provided with appropriate social experiences c) B. F. Skinner: Maladjustment is specific to the culture, the time, the social class, and the situation d) Aaron Beck: Personality reflects cognitions that develop through a process of "reciprocal determinism"

d) Aaron Beck: Personality reflects cognitions that develop through a process of "reciprocal determinism" `

Socially acceptable reason to explain an unacceptable behavior or thought a)displacement b)Projection c)Reaction Formation d) Rationalization Rationalization

d) Rationalization

Savin-Williams and Diamon investigated the sexual identity trajectories amoung sexual-minority youth and found that: a) adolescent females had an earlier onset of all milestones. b) adolescent males had an earlier onset of all milestones. c) adolescent females had an earlier onset of all milestones except first same-sex attraction. d) adolescent males had an earlier onset of all milestone except first disclosure of sexual orientation to another person

d) adolescent males had an earlier onset of all milestone except first disclosure of sexual orientation to another person

An assessment center is most likely to include which: a) a measure of psychomotor ability b) a lie detector c) a "vestibule" d) an "in-basket test"

d) an "in-basket test"

In Cross's Nigrescence/Ethnic Identity Dvpt Model, change in the direction of greater racial or ethnic awareness is: a) a natural consequence of one's developmental level b) the result of peer influence c) one of the possible resolutions of an identity crisis d) due to one or more events that increase the salience of race

d) due to one or more events that increase the salience of race

Survey feedback is a method of organisational dvpt that is based on the premise that: a) people are not usually open and honest with each other b) work groups are the basic foundation of an organization c) organizational output is a function of both production and people d) employees need to understand an organization's strengths and weaknesses

d) employees need to understand an organization's strengths and weaknesses

Super's life-career rainbow depicts the relationship btwn the individual's life stages and his: a) level of career differentiation. b) stage of ego identity development. c) career interests. d) life roles.

d) life roles

PIaget-a child exhibiting horiz decalage when he: a) reasons from one particular event to another particular event b) can perform a task with assistance but not alone c) gradually makes finer and finer distinctions between stimuli over time d) manifests inconsistency in exhibiting a particular type of ability

d) manifests inconsistency in exhibiting a particular type of ability

Which technique should be used to eval the cause and effect or predictive relationships btwn measured variables and latent factors? a) cluster analysis b) multitrait-multimethod matrix c) discriminant function analysis d) structural equation modeling

d) structural equation modeling -used to explore or confirm hypothesised relationships btwn both measured and latent variables

specific ways person compensates for inferiority & to achieve superiority a)mistaken life style b)healthy life style c)open life style d) style of life

d) style of life (Adler)

In expectancy theory, valence is: a) degree of effort. b) level of performance. c) need strength. d) subjective worth of outcomes

d) subjective worth of outcomes

Metcalfe and Mischel use a hot/cool systemto explain which of the following? a)delay of gratification b) risk and resilience c) aggression d) attitude change

delay of gratification

Which of these is characterised by a disturbance in consciousness and cognitive changes? a) dementia b) pseudo dementia c) delirium d) alzheimers

delirium

An interviewer unintentionally nods when research participants answer his question in a way that confirm the study's research hypothesis. The interviewer's behavior is best described as an example of which of the following:

demand characteristic

Which PD is characterised by clinginess and fear of separtion? a) dependent b) antisocial c) histrionic d) schizotypal

dependent

What psychiatric symptom is most commonly associated with stroke?

depressed mood

Minuchin distinguished between three types of rigid triangles that help alleviate tension between two family members - these are:

detouring, stable coalition, and triangulation.

gathering information; defining the problem & setting goals; & generating possible interventions

diagnosis

used to describe the tendency of health professionals to attribute all behavioral, social, & emotional problems to client's diagnosis or situation.

diagnostic overshadowing

Unfairness is when: a) the predictor has significantly different validity coefficients for different groups. b) the predictor is equally invalid for all groups. c) different groups obtain consistently different scores on the predictor but similar scores on the criterion. d) different groups obtain consistently different scores on the criterion but similar scores on the predictor.

different groups obtain consistently different scores on the predictor but similar scores on the criterion.

Any predictor that is more valid for one class or subset of examinees than it is for another is said to have _______

differential validity

The __________ effect refers to the weakening of an inference about someone due to the presence of irrelevant information about him/her.

dilution

yes, but often slower than younger adults. (APA 2002)

do older adults respond to therapy?

The General Guidelines for Providers of Psychological Services requires Professional Psychologists to have a ____ in psychology from a regionally accredited university or professional school. This is a correct description of the requirements of the General Guidelines.

doctoral degree

Huntington's, Marfan's syndrome, and Von Willebrand's disease are all due to an autosomal ____________ gene.

dominant

Morphine increases _________ activity at opioid receptors in the ventral tegmental area and nuclus accumbens, which are part of the brain's reward system.

dopamine

The ______region is involved in working memory and the executive functions. Damage to this area is associated with deficits in attention, planning, problem-solving, and other higher-order cognitive abilities.

dorsolateral prefrontal

A person with damage to which area of the frontal lobes is most likely to have impaired executive functioning, slowed info processing and mood/personality changes? a) medial frontal area b) anterior cingulate area c) dorsolateral prefronal area d) orbitofrontal area

dorsolateral prefrontal area

After a TBI, a man has mild impairments in attention, working memory and exec cognitive functions. Which area has been damaged a) dorsolateral prefrontal region b) orbitofrontal region c) medial temporal region d) posterior parietal region

dorsolateral prefrontal region

Research by Jerome Kagan has shown that the diff btwn high and low reactive infants is: a) attributable to nutritional differences during prenatal development b) due, in part, to genetic factors c) due largely to early infant-parent interactions d) not predictive of future behavior

due, in part, to genetic factors

Multiple regression is ordinarily used only when the predictors and criterion are continuous, but __________ _________ is a technique that allows categorical predictors to be used.

dummy coding

Frequency of aggressive acts and number of hours studied are: a)interval b)ordinal c)ratio d) nominal e)ratio

e

culture specific theories, concepts, & research strategies. used to understand a culture, to see things through the eyes of members of that culture.

emic orientation

An I/O psychologist will apply the concept of elements to which: a) job redesign b) employee selection c) criterion-dvpt d) employee training

employee training

Which technique is used to facilitate diagnosis and restructuring of the family in family therapy? a) reframing and relabeling b) tracking and mimesis c) constructing a family map d) enactment

enactment

What is the name for the system of channels in the cell body neuron which plays a role in the synthesis and transportation of proteins and fats.

endoplasmic reticulum

A drive for order that involves the testing of patterns and structures agains the real world is: a) elaboration b) deduction c) equilibration d) accommodation

equilibration- Piaget- involves a comb of assimilation and accommodation and is motivated by a drive for balance/order

The _________ law states that intact areas of the brain can assume the functions of other areas that have been destroyed.

equipotentiality

one cause can lead to different results. e.g. incest--> promiscuity or sexual inhibition

equipotentiality

serotonin has been implicated in both schizophrenia and mania, its impact differs for the two disorders. A(an) ____ of serotonin has been linked to schizophrenia, while - perhaps contrary to expectation - a (an) ____of serotonin has been linked to mania

excess, low level

The goal of sensate focus is to increase awareness and enjoyment of pleasurable sensations and to reduce performance anxiety. Therefore, it addresses responding during the _______ stage of the human sexual response cycle.

excitement

Cross's revised Nigrescence Identity Development Model predicts that an African American in the pre-encounter stage of identity dvpt will: a) identify strongly with the mainstream (white) culture b) exhibit either internalised racism or have low salience for race c) be hostile to members of all racial/ethnic minority groups d) exhibit healthy cultural paranoia

exhibit either internalised racism or have low salience for race

people believe that others think and act more like them than they actually do.

false consensus bias

The tendency to overestimate the degree to which others agree with you is: a) Barnum effect b) false consensus bias c) Zeigarnik effect d) illusory correlation

false consensus effect

When the MMSE is used as a screening tool for Alzheimer's with African-Am adults, it tends to result in a dispropor number of: a) true negatives. b) true positives. c) false negatives. d) false positives.

false positives

Recent research suggests that people who watch a lot of TV: a) fear their environment more than people who report being less frequent TV viewers b) fear their environment somewhat less than people who report being less frequent TV viewers c) fear their environment much less than people who report being less frequent TV viewers d) are about equal in terms of fear of their environment to people who report being less frequent TV viewers

fear their environment more than people who report being less frequent TV viewers

According the ________, the perception of objects involves two stages - preattentive in which the basic aspects of an object are perceived in parallel (e.g., edges, size, color) and attentive in which these features are "glued" together into a coherent whole through a serial process that depends on focal attention.

feature integration theory

identifying & altering oppressive forces in society that have affected their clients' lives. Primary goal: empowerment or helping women become more self-defining & self-determining

feminist therapy goals

What are the 4 stages of Sophie's gay/lesbian identity development model?

first awareness testing and exploration identity acceptance identity integration

there is a _____ when a test does not discriminate well among people with lower levels of achievement, ability, or aptitude. Conversely, when a test does discriminate among low achiev

floor effect

The object of a phobia is considered to be related in some way to (and symbolic of) a _____. By expressing the impulse in an external symbolic form, the individual is better able to avoid it.

forbidden instinctual impulse

What is the methodology of choice for uncovering malingering? a) multiple choice b) forced choice c) forced distribution d) open ended

forced choice

Which methodology in testing uncovers malingering? a) multiple choice b) true/false c) forced choice d) likert scale

forced choice

____refers to the form (shape) of the inkblot. It refers specifically to the degree to which an examinee's response matches the actual form of the inkblot.

form quality

A _____is conducted while a training program is being developed to determine if any modifications to the program are required.

formative evaluation

An ongoing debate in the literature centeres on a therapist's responsib when working with a client who is HIV+ and is engaging in risky sex. A central issue of the debate is: a) social responsib b) client abandonment c) foreseeability of harm d) vicarious liability

forseeability of harm (fiduciary relatonship and an identifiable victim)

During the ____ stage, infants reach for a partially hidden object but stop if the object disappears from view.

four to eight month

How many more men commit suicide than women? a) twice as many b) three-four times as many c) four-five times as many

four to five times

The research suggests that one distinguishing feature between the two disorders is that depression is more likely to involve impairments in ______, while dementia entails impairments in ____.

free recall, both free recall and recognition

Kohlber's cog-dvptl theory distinguishes btwn 3 stages in gender identity dvpt. The first stage is ___________ and involves recog one's own gender and is apparent at 2-3yrs gender constancy gender conservation gender identity gender stability

gender identity

A strategic family therapist is working with a fmaily in which husband and wife have avoided their own conflicts by focusing attenion on the problems of 9 yr old son. the therapist is most likely to: a) educate the parents about "triangulation" b) obtain a detailed family history from the husband and wife to identify the pattern of interactions in their families of origins c) give the husband and wife a homework assignment designed to foster recognition of the conflicts that exist between them d) have the family members adopt the roles of each other and role-play a typical family interaction

give the husband and wife a homework assignment designed to foster recognition of the conflicts that exist between them

____, proposed that, under conditions of task interdependence, "the optimal strategy, of course, is to set goals for the group as well as for each individual within the group."

goal-setting theory

Creation & Maintanace of the Group; Culture Building; Activation & Illumination of the Here & Now

group therapist's primary tasks

ideological system that denies, denigrates, & stigmatizes among nonheterosexual forms of behavior, identity, relationships, or community.

heterosexism (Herck, 2000)

Whichy off thel following isa not jan empirically validateds treatment for primary insomnia? a) stimulus control therapy b) sleep restriction c) cognitive-behavior therapy d) habit reversal training

habit reversal training

A DSM diagnosis of Opiod withdrawal requires the dvpt of 3 characteristic symptoms following cessation -symptoms include all EXCEPT: a) dysphoric mood b) yawning c) muslce aches d) hallucinations

hallucinations

Which of the following is most useful for explaining the results of Resenhan's study? a) central tendency error b) halo effect c) Hawthorne effect d) contrast effect

halo effect

Stage 3 of dementia is characterised by all of these except: a) severly deteriorated intellectual functioning b) apathy c) hand clumsiness d) incontinence

hand clumsiness-this is only seen in dementia due to HIV

As described by Tiedman & O'Hara, career counseling involves: a) assessing the individual's self-concept and then identifying careers that match his/her self-concept b) helping the individual transition from the anticipation/preoccupation phase to the implementation phase of career development c) instilling an attitude of self-development in the individual so that he/she can adapt to changing work requirements in the future d) helping the individual successfully accomplish the tasks of his/her current stage of career development

helping the individual transition from the anticipation/preoccupation phase to the implementation phase of career dvpt

Herzberg's two-factor theory identifies pay as a ________ factor, which leads to dissatisfaction when it is perceived to be inadequate but produces a state of neutrality when it is adequate (it has no impact on motivation).

hygiene

Herzberg's two-factor theory is actually a theory of both satisfaction and motivation. As its name implies, it proposes that satisfaction and motivation are affected by two different types of job-related factors. These factors are?

hygiene (job context) factors and motivator (job content) factors.

D. Herzberg's two-factor theory divides job components into two types: ______

hygiene factors and motivator factors.

adjunct to major therapeutic schools--psychoanalysis, behavioral, cognitive, etc.

hypnosis

Experiencing alterations of memory, perception, & mood in response to suggestions

hypnosis involves (according to Orne & Dinges, 1989)

When using the critical incident technique, a supervisor: a) identifies an employee's typical (average) job behaviors. b) identifies an employee's effective and ineffective job behaviors. c) determines if an employee has achieved his/her performance goals. d) determines if an employee's performance matches a predefined standard of performance.

identifies an employee's effective and ineffective job behaviors.

What are the 6 stages of Cass's gay/ lesbian identity development model?

identity confusion identity comparison identity tolerance identity acceptance identity pride identity synthesis

Which of these occurs when a person mistakenly sees a cat where there is actually a shoe: a) delusion b) hallucination c) illusion

illusion

The belief that there is a relationship btwn events when there actually isn't one is: a) Barnum effect b) false consensus bias c) Zeigarnik effect d) illusory correlation

illusory correlation

Studies suggest that African-American therapy clients suggest they are most likely to prefer an African-American therapist when they are in which stage of identity dvpt a) pre-encounter b) immersion c) internalisation d) autonomy

immersion

Damage to the prefrontal cortex is most likely to cause: a) impaired memory and attention b) psychic blindness c) alterations in the sleep/wake cycle d) impaired motor coordination

impaired memory and attention

In _________ therapy, a client repeatedly imagines a feared stimulus (CS) in the absence of the stimulus (US) that initially evoked the fear in order to eliminate the conditioned fear response. In this therapy approach, a much higher anxiety is evoked as the imagined scenes are exaggerated by the therapist who also introduces commentaries on the worst of the persons fears.

implosive

Which treatment entails a combination of exposure to a feared stimulus coupled with psychody interp of the fear? a) overcorrection b) implosive therapy c) covert sensitization d) PST

implosive therapy

A listener who is processing a persuasive message peripherally (i.e., is using the "peripheral route") is most likely to be persuaded by the messagel if she is: a) bored b) in a good mood c) in a bad mood d) emotionally unstable

in a good mood

In organisations, frame-of-reference training is used to: a) educe accidents b) improve team productivity c) help leaders adopt a more participative style d) increase rater accuracy

increase rater reliability = familiarising raters with the multidimensional nature of job performance and with the actual content of each performance dimension

Recent research on neurotransmitter abnormalities in Schizophrenia suggests that which neurotransmitter abnormalities are present?

increased norepinephrine, increased serotonin, increased dopamine

A researcher would be interested in the seletion ratio, base rate, and validity coefficient of a test when she is evaluating test's : a) differential validity. b) external validity. c) incremental validity. d) concurrent validity

incremental validity

The primary goal of self-instructional training (SIT) for children exhibiting hyperactivity and excessive impulsivity is to: a) eliminate dysfunc/unrealistic cognitions b) re-align inention and action c) clarify response contingencies d) insert thought btwn stimulus and response

insert thought btwn stimulus and response

When the results of two or more independent variables considered together:

interaction effect

Cross's model of balck identity dvpt includes 4 stages, the last of which is: a) autonomy b) internalisation c) incorporation d) integrative awareness

internalisation-the dvpt of inner security as identity conflicts are resovled (4stages are: pre-encounter, encounter, immersion-emmersion and internalisation)

Racial/Culturall Raciall Identity Development model: ___, an individual recognizes that rejection of the dominant culture and complete immersion in his/her minority culture is too inflexible and limiting.

introspection stage

General cognitive ability is predictive of job performance for: a) is useful only for managerial and other professional jobs b) is useful only for predicting performance in traditionally "male" jobs c) is not as accurate as specific ability tests for many different types of jobs d) is predictive of performance across a wide variety of jobs

is predictive of performance across a wide variety of jobs

Which is incorrect about computer-assisted instruction in the classroom? a) it increases social interactions among children b) it leads to improvements on standardized achievement tests c) it decreases students' phobias toward computers d) it is more beneficial for high- (versus low-) ability students

it is more beneficial for high- (versus low-) ability students ....it may benefit low-ability students more b/c it provides them with an opportunity for individualised instruction

The _______ method is mnemonic device that involves forming an image that links two words.

keyword Note: For example, to remember that ranidae is the scientific term for common frogs, you might create an image of a frog sitting in the rain under an umbrella.

A large heterotrait-monomethod coefficient indicates: a) a lack of differential validity b) a lack of discriminant validity c) adequate convergent validity d) adequate concurrent validity

lack of discriminant validity

A ___ distribution is more "peaked" than a normal distribution.

leptokurtic

The type of question used by family therapists to identify family problems are: a) circular b) lineal c) strategic d) reflexive

lineal

Raw scores are often transformed in order to simplify their interpretation, and these transformations can be either ______

linear or nonlinear.

Nausea and vomiting, loss of coordination, and seizures, are symptoms that suggest _____. a) tricyclic overdose b)Neuroleptic malignant syndrome (NMS) c)5-HT toxicity syndrome d)lithium toxicity

lithium toxicity

____ is characterized by nausea and vomiting, loss of coordination, and seizures

lithium toxicity

Someone with dissociative amnesia is unable to recall events that happened during the first few hrs after she was assaulted. This is referred to as: a) localised amnesia b) selective amnesia c) anterograde amnesia d) retrograde amnesia

localised amnesia

After brief electrical stimulation of the hippocampus enhances the nerve cell electrical activity in that area of the brain for mins-hrs. This is referred to as: a) graded potentiation b) hyperpolarization c) absolute refractory period d) long-term potentiation

long-term potentiation

___________ predicts a tendency for a loss of a partic magnitude to seem more aversive than a gain of the same magnitude seems attractive a) status quo bias b) psychological reactance c) loss aversion d) gain/loss theory

loss aversion

Paralanguage, a characteristic of communication influenced by culture, refers the use of: a) personal and interpersonal space b) facial expresssions, gestures, eye contact and other movements c) loudness of voice, silence, rate of speech & other vocal cues d) culturally shared meanings of certain words/phrases

loudness of voice, silence, rate of speech & other vocal cues

On the MMPI-2, an attempt to "fake good" is sugested by which of the following? a) elevated F scale score with low L and K sale scores b) elevated F and L scale scores with a low K scale score c) low F scale score and elevated L and K scale scores d) low K scale score with moderately high L scale score and very high F scale score

low F scale score and elevated L and K scale scores

23. Members of a cohesive group, as compared to members of a non-cohesive group, are more likely to have: A lower absenteeism B lower productivity C better decision-making D higher turnover

lower absenteeism Group cohesiveness has been associated with less absenteeism and turnover and, in some situations, greater productivity.

problems during separation-individuation or inadequate resolution of natural tendency to split mental representations of the self & others

maladaptive behavior per Object relations

Alcohol Dependence, is more common in ; ratio may be as high as 5:1.

males, 5:1

Faking syptoms for secondary gain is known as: a) factitious disorder b) malingering c) conversion disorder d) somatisation disorder

malingering

Which disorder is characterised by intentionally producing or faking symptoms in order to obtain an obvious external reward? a) somatization disorder b) factitious disorder c) malingering d) conversion disorder

malingering

The midbrain contains the following EXCEPT: a) inferior colliculus b) superior colliculus c) sunstantia nigra d) mammillary bodies

mammillary bodies (part of the hypothalamus)

A psych evaluates the effects of a 15 month training program on the conservation skills of preoperational children by administering a measure of conservation to the same group of children before and at the end of training. The psychologist finds that a significantly greater number of children conserve after the program than before. The big threat to internal validity is:

maturation

A personnel director conducts a study to eval the criterion-related validity of a new selection test. She finds that the slope of the regression line differs for males and females. This suggests that the test: a) may be "unfair" b) may have differential validity c) may be lacking in incremental validity d) may be lacking in convergent validity

may have differential validity-if the slopes of the regression lines for 2 diff groups differ this suggests that there may be differential validity

Benefits of a CBT interven for EDs seems be due to the ipact on the person's self-efficacy beliefs. So, self-efficacy beliefs are: a) moderator variable b) mediator variable c) confounding variable d) suppressor variable

mediator variable

Research has demonstrated that recall of a particular piece of information is greater after five hours of sleep than after five hours of wakeful activity. This finding tends to discredit which of the following theories of forgetting. a. interference. b. memory trace decay. c. retrieval cue. d. motivated forgetting.

memory trace decay

_____ theory which attributes memory loss to a decay in memory traces as the result of the passage of time. a. interference. b. memory trace decay. c. retrieval cue. d. motivated forgetting.

memory trace decay

derived from the medical/psychiatric model & is attributable largely to the work of Gerald Caplan. 4 types

mental health consultation

Piaget- the ability of a child to see someone else perform a behav and subsequently perform that behav himself depends on the dvpt of which ability? a) mental representation b) assimilation c) transductive reasoning d) centration

mental representation

Validity generalisation studies makes use of which technique? a) multiple-regression b) meta-analysis c) item response theory d) multitrait-multimethod matrix

meta-analysis

impaired concentration, attention, and motor activity these symptoms are characteristic of the ____ of AIDS

middle stage

A researcher wants to compare the efects of 3 different PTSD treatments. She randomly assigns individuals who were recently exposed to the same trauma to either EMDR, stress inocu, or in vivo exposure group and then assesses their symps one week, one month, and 3 months after the end of treatment. The research is using which design? a) btwn groups b) w/in subjs c) mixed d) counterbalanced

mixed

A ______________ scale uses three critical incidences, representing good, average and poor performance, and asks a rater to indicate when ratee's performance is equal to, worse than or better than the examples.

mixed standard scale

negatively skewed distribution

mode highest, median middle, mean lowest

positively skewed distribution

mode lowest, median middle, mean highest

People are most likely to be convinced by a position different to their own if the discrepancy btwn their initial position and the position of the other party is: a) low b) moderate c) high d) varies from member to member

moderate

According to two-factor theory, satisfaction and motivation are increased by providing an employee with_____

motivator factors

Does not require withdrawal of a treatment during the course of the study but, instead, involves sequentially applying the treatment either to different behaviors of the same subject, to the same subject in different settings, to the same subject in different subjects.

multiple baseline

To eval the effects of a behav interven for reducing the self-injurious behavs of a 5 yr old boy with autism in diff settings, you would use which of the following single-subj designs: a) ABA b) withdrawal c) multiple baseline d) changing criterion

multiple baseline

scores on two or more predictors are combined to predict membership in three or more criterion groups. This procedure is known as __________. a) multiple cutoff technique b)multiple discriminant analysis

multiple discriminant analysis

is a disorder of neuromuscular transmission. Although its underlying cause is unknown, it apparently involves inhibition of nerve action potentials as the result of a presynaptic abnormality or postsynaptic insensitiviry. a. myasthenia gravis. b. cerebral palsy. c. cystic fibrosis. d. Parkinson's disease.

myasthenia gravis

Although Kohlberg believed that there is a positive relationship between moral judgment and moral action, he proposed that this relationship is much stronger at higher levels of moral development because the range of actions is _______, and personal sense of responsibilty is _________.

narrower greater

maintains homeostasis by attempting to correct deviations in the status quo

negative feedback

A predictor's ____ indicates the probability that an individual identified as a negative is a true negative. It is calculated by dividing the number of true negatives by the number of true and false negatives. a) positive predictive value b) negative predictive value c) sensitivity d) specificity

negative predictive value

a growth disorder involving abandonment of the self for the self-image & a resulting lack of integration. stems from

neurotic (maladaptive)behavior (Gestalt)

For most questions about the relationship between therapist, client, or process variables and therapy outcome, the best answer is _____

no relationship

test's reliability coefficient = to 1

no standard error of measurement

focus more on individual factors & modifying personal behavior

non-sexist therapy

multiple cutoff technique is _____; that is, a high score on one measure does not compensate for a low score on another measure.

noncompensatory

when using coverts sensitisation, the US is a- a) noxious stimulus presented in vivo b) noxious stiumuls presented in imagination c) innocuous stimulus presented in vivo d) innocuous stimulus presented in imagination

noxious stimulus presented in imagination

The purpose of rotation in factor analysis is to: a) obtain a pattern of factor loadings that is easier to interpret. b) reduce the impact of measurement error on the factor loadings. c) reduce the magnitude of the communalities. d) obtain a clearer pattern of communalities.

obtain a pattern of factor loadings that is easier to interpret

Neuropsychology 53. The condition describing a side-effect of antipsychotic medication that is incorrectly matched with its symptoms is: A akathisia — inability to sit, constantly pacing, restlessness B dysarthia — imperfect articulation of speech C akinesia — appears to be slow moving, indifferent to stimuli, emotionally constricted D opisthotonus — spasms in the lower extremities

opisthotonus — spasms in the lower extremities

A person with damage to which area of the frontal lobe is most likely to engage in disinhibited, impulsive behavs an dexhibit emotional lability, distractibility and poor judgment and insight? a) medial frontal area b) anterior cingulate area c) dorsolateral prefronal area d) orbitofrontal area

orbitofrontal area

adopts a systems approach & defines the entire organization as the consultee

organizational consulting

When treating a client with paraphilia, whichtechnique would be most useful for replacing the inappro stimulus that currentlyelicits sexual arousal with a more a stimulus a) covert sensitization b) flooding (in vivo exposure) c) habit reversal training d) orgasmic reconditioning

orgasmic reconditioning

Recovery of cognitive ability following head trauma follows a predictable pattern in most patients. For example, with regard to disorientation, recovery usually occurs in the following sequence: a) orientation to personal information, then to place, and finally to time b) orientation to time and place followed by orientation to personal information c) orientation to place, then to personal information, and finally to time d) orientation to personal information, then to time, and finally to place

orientation to personal information, then to place, and finally to time

Teratogens account for ___% of MR a) 5% b) 15-25% c) 35-50% d) over 50%

over 50%

The need for physical guidance with possible resultant negative conseqs (eg aggressiveness/avoidance) is assoc with which of the following? a) aversive counterconditioning b) negative practice c) response cost d) overcorrection

overcorrection

The __________ hypothesis predicts that a person's internal motivation to perform a behavior will be weakened when he/she is provided with external reinforcement for doing so.

overjustification

A ______ requires the individual to respond with one member of a pair when presented with the other member.

paired associate task

supervisee replicates problems & symptoms with supervisor that are being manifested by the therapist's clients.

parallel process

In treating a client with a snake phobia, a therapist handles the snake and then guides the client through a series of steps until she is able to handle the snake herself. This technique is referred to as: a) symbolic modeling b) participant modeling c) in-vivo desensitization d) overcorrection

participant modeling

Which is NOT true about the effects of crowding a) men are more negatively affected than women by corwding b) increasing agge is assoc with an increasing negative impact of crowding c) people are more willing to discuss intimate details in a crowded situation d) crowded conditions are asoc with a higher risk for health probs

people are more willing to discuss intimate details in a crowded situation

norm-referenced score

percentile rank, T-score, and z-score

Schaie's longitudinal study found substantial age-related declines for only two abilities - __ and ____

perceptual speed and numerical ability.

Bandura proposed that reinforcement is more important for _________ than __________.

performance learning

Persuasion that occurs via the _____route is the result of simple cues (e.g., the attractiveness of the communicator) and/or occurs because the message was uninteresting or irrelevant, and any resulting attitude change is likely to be weak, temporary, and nonpredictive of future behavior.

peripheral

based on assumption that all people have an innate "self-actualizing tendency"

person centered therapy (Rogers)

goal is to help client achieve congruence b/w self & experiences so that he/she can become a more fully-functioning, self-actualizing person

person-centered therapy

According to McClelland, ____ is the ability to dominate others, is used to benefit oneself, and is less important than socialized power for management effectiveness.

personal power

consists of self & self-image

personality (Gestalt psychology)

borrowed from medical studies; means providing subjects with the non-specific (common) factors of psychotherapy such as attention & support. placebo conditions have a substantial impact with subjects in placebo control groups showing greater improvement in symptoms than no-treatment or wait-list control groups

placebo effect

A ____ distribution is flatter than a normal distribution. While the distribution of some transformed scores (e.g., percentile ranks) are flatter than the distribution of raw scores, not all nonlinear transformations produce a this type of distribution.

platykurtic

In family therapy, exaggerating the severity of a symptom is : a) reactance b) restraining c) positioning d) prescription

positioning

More ______ symptoms is associate with a better prognosis in Sx a) positive b) negative

positive

The left cerebral hemisphere appears to regulate _________ emotions.

positive

disrupts homeostasis by encouraging or creating deviations in the status quo

positive feedback

A predictor's ______ indicates the probability that an individual identified as a positive is a true positive. It is calculated by dividing the number of true positives by the number of true and false positives. a) positive predictive value b) negative predictive value c) sensitivity d) specificity

positive predictive value

Following a head injury, a middle-ages man exps loss of sensation in fingers of his left hand. The damage involved: a) postcentral gyrus b) precentral gyrus c) lateral fissure d) presentral sulcus

postcentral gyrus-the somatosensory cortex is located in the postcentral gyrus in the parietal lobe

Kohlberg's levels of moral development: legalistic orientation and universal Ethical Principles orientation

postconventional morality

The _____ is a major sensorimotor area of the brain, and damage can produce apraxia or contralateral neglect.

posterior parietal region

Kohlberg's levels of moral development: punishment-obedience orientation and instrumental-relativist orientation

preconventional morality

Kohlberg's levels of moral development:

preconventional morality, conventional morality and postconventional morality.

A 47 yr old politician teaching his aides the sentials of dirty tricks tells them "nothing is wrong unless you get caught". One of his aides, a delevopmentalpsychologist comments to ather aide that the politician is apparently ate ______________ level of moral dvpt a) preconventional b) conventional c) postconventional d) instrumental

preconventional-an action is moral to the degree that it doesn't result in punishment

The research suggests that sex therapy has been most successful for the treatment of: a) impotence b) premature ejaculation c) orgasmic anhedonia d) hypoactive sexual desire

premature ejaculation

10-35% of group members drop out of therapy during the first 12-20 sessions.

premature termination

Piaget believed that a major achievement of the _______ is _______, which is learning based on intuition.

preoperational stage insight learning

In the _____stage, children exhibit _______, or an inability to mentally hold two dimensions at the same time.

preoperational stage, centration

According to Strauss and Carpenter, long term effects of schizophrenia on employment is most affected by __________.

previous employment note: likewise past sx severity is the best predictor of future sx severity (various domains of functioning are independent)

Scores on WAIS _________ index are most negatively impacted by increasing age a) Processing Speed b) Perceptual Reasoning c) Working Memory d) Verbal Comprehension

processing speed

working with 1 or more administrators to resolve problems related to an existing problem

programmed centered administrative consultation

The Specialty Guidelines for Forensic Psychologists (1991) state that: a) records should be maintained and filed in a way that is consistent with state and federal regulations b) records should be maintained and filed in a way that is consistent with the APA's Ethics Code c) records should be maintained in a way that protects the best interests of the client d) records should be maintained using standards that are higher than the normative standards for records in general clinical practice

records should be maintained using standards that are higher than the normative standards for records in general clinical practice

An amputee experiences a sensation on his missing limb whenever he is touched on his lower lip. This is: a) phantom limb. b) super-added phantom. c) referred phantom sensation. d) adventitious synesthesia.

referred phantom sesation

The ____ is essentially a "running mean": It predicts the mean Y (dependent variable) score obtained by individuals in the validation or tryout sample for each value of X (independent variable).

regression line

1) hypnosis does not seem to enhance the accuracy of memories & may create more pseudo-memories than accurate memories. 2) hypnosis may exaggerate a person's confidence in the validity of uncertain memories, especially inaccurate memories. 3) even inaccurate memories often reflect issues & experiences that are relevant to treatment

repressed memories

Racial/Culturall Raciall Identity Development model:Rejection of the attitudes, values, and beliefs of the dominant culture is characteristic of the _____stage.

resistance and immersion

In family therapy, the adolescent daughter complains that she doesn't think she can change and she feels that doing what the therapist has requested is pointless. The therapist responds by saying "well in that case, i think you shouldn't even try". The therapists response is an example of: a) reactance b) restraining c) positioning d) prescription

restraining--telling a client not to change

Ebbinghaus's results on memorising lists of nonsense syllables indicated that: a)syllables in the middle of a list are better remembered than those at the beginning and end of the list b) when memorizing syllables, new learning tends to interfere with previous learning c) overlearning improves memory for syllables up to a point but thereafter has no effect d) rote learning of syllables tends to lead to rapid forgetting

rote learning of syllables tends to lead

As described by Piaget, a child in the autonomous stage of moral development beieves that: a) rule violations will be punished b) rules can be changed by consensus c) rules can be changed by authorities only d) rules are made to be broken

rules can be changed by consensus

At the first cousin level, schizotypals have an increased rate of _________ in the family a) schizoid PD b) paranoid PD c) antisocial PD d) schizophrenia

schizophrenia

Which theory would predict that job enrichment would have a detrimental effect on job performance? a) two-factor b) activation c) equity d) scientific management

scientific management-workers motivated only by external rewards (pay)

The _____stage of HIV lasts for about eight to ten years and involves few, if any, clinical symptoms. During this _____ stage, the immune system in fighting the virus.

second (middle), asymptomatic

creative aspect of the personality that promotes the individual's inherent tendency for self-actualization or the ability to live as a fully integrated person

self (Perls)

What phenomena is a researcher looking at when they investigate students' expectations regarding their ability to master a new skills, achieve a goal, or produce a partic outcome? a) locus of control b) need for achievement c) self-actualisation d) self-efficacy

self-efficacy

Bandura's _____________ theory proposes that people undertake activities they believe they are capable of performing.

self-efficacy Note: Self-efficacy beliefs are influenced by past accomplishments, observation of others, verbal persuasion and physiological state

the "darker side" of the personality. hinders growth & self-actualization by imposing external standards

self-image (Perls)

_____ is a method of "non-genital pleasuring" that is used in the early stages of the treatment of sexual dysfunctions.

sensate focus

Cocaine use by pregnant women increases the risk for spontaneous abortion and stillbirth; and, for babies who survive, is associated with ....

several physical and behavioral abnormalities, including low birthweight piercing cry exaggerated startle response irritability and hypersensitivity not responding in normal ways to caregivers

all negative attitudes based on sexual orientation whether target is homosexual, bisexual, or heterosexual

sexual prejudice (Hereck, 2000)

If an adult takes phenobarbital every night for weeks and then abruptly stops the result will be: a) she will not experience any change in REM sleep b) she will experience an increase in REM sleep c) she will experience a decrease in REM sleep d) she will experience an increased need for sleep

she will experience an increase in REM sleep -barbituate use causes decrease in REM and abrupt cessation can cause REM rebound, inability to sleep, poor quality sleep, and nightmares

Which symptom helps confirm a DSM diagnosis of Opioid Dependence a) signs of tolerance b) a persistent craving for the drug c) perceptual disturbances d) use of the drug when doing so is hazardous

signs of tolerance

When using this design, the effects of a treatment are evaluated by measuring the dependent variable several times at regular intervals both before and after the treatment is applied. This procedure allows subjects to act as their own no treatment controls:

single group time series design

In which type of research studies do participants act as their own controls:

single subject

This is often used to investigate the effects of an independent variable on the behavior of one subject or a small number of subjects, they can also be used with groups of subjects:

single subject design

the _____refers to the tendency to remember a message but, over time, to forget the source of the message. One outcome of the sleeper effect is that people may eventually adopt an attitude or belief that was originally communicated to them by an low-credible source because they have forgotten the source.

sleeper effect

When an elderly client displays paranoid reactions, __, ___, & ___ are the most effective forms of treatment.

small dose of major tranquilizer, such as phenothiazine, a supportive environment, correct any sensory deficits that may be contributing to the client's paranoia.

Social work is appropriate for people who score highest on the ____theme, which is the most dissimilar from the realistic theme.

social

The theory of ______ predicts that, under certain circumstances, the mere presence of others enhances performance.

social facilitation

the mere presence of others increases the likelihood that the dominant response will be activated.

social facilitation

people will work less hard when in a group than when alone.

social loafing

The primary component for structured learning therapy for depression is: a) social skills training b) covert sensitisation c) flooding d) overcorrection

social skills training

McClelland proposed that effective managers are characterised by a high need for ________ power a) legitimate b) expert c) personal d) socialised

socialised= power used to benefit others. Contributes to management effectiveness by helping managers achieve organisational goals throughthe action of subordinates

A predictor's ____ refers to the proportion of individuals in the validation sample without the characteristic measured by the predictor who were correctly identified by the predictor as not having that characteristic. It provides an index of the predictor's ability to identify true negatives. Specificity is calculated by dividing the number of true negatives by the number of true negatives plus false positives. a) positive predictive value b) negative predictive value c) sensitivity d) specificity

specificity

To find shared variability

square correlation coefficient e.g., .90 = .81 and .30 = .09

The correlation coefficient for television viewing and achievement scores is -.81. The proportion of variance is calculated by __ -.81 which is approximately____

squaring .66.

A _____ occurs when a parent and child consistently "gang up" against the other parent.

stable coalition

Minuchin- this occurs when a child and parent gang up against the other parent: a) triangulation b) detouring c) a stable coalition d) marital skew

stable coalition

entry; diagnosis; implementation; disengagement

stages of consultation

14 year olf Kevin uses swear words when he is with his friends b/c they respond positively. However, Kevin never uses these words when he is with his family b/c his parents become very upset. His differential use of these words in different setting illustrates the concept of: a) stiumuls control b) partial reinforcement c) shaping d) overcorrection

stimulus control-response is brough under stimulus control when person learns to respond in situations in which reinf is likely but not in situations in which no reinf is likely

The type of question used by family therapists to foster change are: a) circular b) lineal c) strategic d) reflexive

strategic

involuntary movements of the facial muscles, tongue, and limbs; a possible neurotoxic side effect of long-term use of antipsychotic drugs that target D2 dopamine receptors

tardive dyskinesia

Severe deficits in semantic memory are most likely due to damage in the a) frontal lobes b) parietal lobes c) temporal lobes d) occipital lobes

temporal lobes

The Woodcock Johnson is based on: a) Spearman's notion of general intelligence b) Luria's distinction between simultaneous and sequential processing c) the Cattell-Horn-Carroll approach to cognitive abilities d) Sternberg's model of "successful intelligence

the Cattell-Horn-Carroll approach to cognitive abilities

The purpose of PL 94-142 (the Education of all Handicapped Children Act) was to improve the educational opportunities for children and adolescents with disabilities, including providing the "least restrictive" educational environment. This law is now known as ...?

the Individuals with Disabilities Education Act (IDEA)

In their propspective study of patients who underwent gender reassignement surgery, Smith et al. found that: a) the majority of patients no longer expressed gender dysphoria following surgery. b) the majority of patients continued to express gender dysphoria following surgery. c) the majority of patients expressed an increase in gender dysphoria following surgery. d) the majority of female-to-male (but not male-to-female) patients expressed an increase in gender dysphoria following surgery.

the majority of patients no longer expressed gender dysphoria following surgery.

The underlying premise of Ellis' REBT is that dysfunctional behaviours are: a) the result of irrational thoughts and beliefs. b) the result of incongruence between self and experience. c) the result of "automatic thoughts." d) the result of a lack of "awareness

the result of irrational thoughts and beliefs

The rotating shift is associated with a number of adverse consequences including fatigue, concentration and performance errors. The negative effects cabe reduced when: a) the rotation is clockwise and weekly b) the rotation is clockwise and either rapid (every 3 days) or slow (every 3 weeks) c) the rotation is counterclockwise and weekly d) the rotation is counterclockwise and either rapid (every 3 days) or slow (every 3 weeks)

the rotation is clockwise and either rapid (every 3 days) or slow (every 3 weeks) (weekly rotation is most detrimental)

What is the formula for the Stardard Error of the Estimate?

the standard deviation of the criterion scores times the square root of one minus the validity coefficient squared. (Thus, the standard error of estimate ranges from 0 [which occurs when the validity coefficient is 1.0] to the standard deviation of the criterion scores [which occurs when the validity coefficient is 0].)

Guy, Polestra, & Stark (1989) surveyed 749 psychologists 74.3% experienced personal distress in past 3 years & for 36.7% it decreased the quality of their work; 4.6% admitted the distress resulted in inadequate treatment

therapist distress

we randomly select the population

thereby affecting external validity

we randomly assign to groups within the experiment

thereby affecting internal validity

_______ biofeedback has been found to be useful for treating migraines, essential hypertension, Raynaud's and certain types of pain including intermittent claudication associated with diabetes: a) Thermal b) Respiratory c) Brain wave activity d) Muscle tension

thermal

To eval the effectiveness of a training program designed to promote safe behavs at work, Dr Juarez observes and records each employees behaviour for 30mins each day for one week before and one week after his participation in the training program. Dr Juarez is using which research design? a) multiple baseline b) changing criterion c) time sampling d) time series

time series-variable assessed several times at regular intervals before and after administration of the IV

Fiedler's LPC theory of leadership proposes a) leaders can alter their leadership style to fit the demands of the situation b) the most effective leaders are person- (versus task-) oriented c) to be most effective, a leader should match his/her leadership style to the characteristics of the employee d) to be most effective, a leader's personality should match the demands of the situation

to be most effective, a leader's personality should match the demands of the situation

When treating a couple in which the husbanas physically abused his wife, and ithas been clearly determined that the abuscan be classified as instrumental (vs expressive) the best initial approach is: a) to ensure that the husband and wife are physically separated and to provide them with separate therapy. b) to have the husband and wife sign a "no-violence contract" and to provide them with separate therapy. c) to have the husband and wife sign a "no-violence contract" and to provide them with conjoint therapy. d) to provide combined individual and group therapy for both the husband and wife.

to ensure that the husband and wife are physically separated and to provide them with separate therapy.

What is the best way to control the halo effect? a) make the results of assessments available to employees b) involve raters in the development of the rating scale c) train the raters d) use peer ratings

train the raters

Theories of Parsons (1939) and Holland (1988) are examples of _____________ approach to career choice and dvpt a) developmental b) trait-oriented c) reinforcement-based d) personality

trait-oriented

Kobasa and her colleagues found that hardiness reduces the negative effects of stressful life events on health by facilitating _____

transformational coping

This involves directly altering those events or thinking about them more optimistically) and promoting the use of social resources

transformational coping

PIaget-the ability to compare objs and find diffs and similarities btwn them is: a) transductive reasoning b) reversibility c) transitive inference d) decentration

transitive inference

multiple baseline design

treatment is applied consecutively. e.g., treatment to reduce headbanging in autistic child might first be applied in lab, then at home, then in school.

Minuchin- this occurs when a child's loyalty to one parent means rejection of the other: a) triangulation b) detouring c) a stable coalition d) marital skew

triangulation

____ is characterized by ataxia, cardiac arrhythmia, and delirium

tricyclic overdose

Ataxia, cardiac arrhythmia, and delirium, are symptoms of _____ a) tricyclic overdose b)Neuroleptic malignant syndrome (NMS) c)5-HT toxicity syndrome d)lithium toxicity

tricyclic overdose.

Major Depressive Disorder is about ___ as common in females as males.

twice

As defined by Campbell & Stanley, selectio is likely to be a threat to a study's internal validity when: a) two different intact groups act as the experimental group and control group. b) there is a differential loss of participants in the experimental and control groups. c) participants are not randomly selected from the population. d) participants in one group communicate important information to participants in another group

two different intact groups act as the experimental group and control group.

From adolescence onwards, females are ___more likely to suffer with depression than males a) equally b) two times c) three times d) four times

two times

The Zeigarnik effect occurs: a) under stressful conditions b) under nonstressful conditions c) in familiary circumstances d) in unfamiliar circumstances

under nonstressful conditions

One of the most consistent findings of the research on group therapy is that people who terminate prematurely from group treatment are those who have _____ and ______

unrealistic expectations and unfavorable attitudes.

latin square

used when more than one treatment or condition is consecutively applied to a group of subjects

ANCOVA

used when partial out a moderator variable

_____ relies on accounting procedures to evaluate the costs and benefits of training programs, selection procedures, and other personnel interventions. With regard to training, this might involve considering the costs of developing and implementing the training program, estimates of the productivity of trained versus untrained employees, and the length of time that training is expected to affect employee productivity.

utility analysis

assess the return-on-investment of the training program.

utility analysis

An adult with TBI is likley to obtain the highest score on which WAIS index? a) working memory b) verbal comprehension c) processing speed d) perceptual reasoning

verbal comprehension

1) suicidal statements by clients; 2) lack of therapeutic success--single most stressful aspect of their work; 3) issues of confidentiality most frequently encountered ethical/legal dilema

work related stress

According to path-goal theory, the optimal leadership style depends on certain characteristics of the _______ and the _________.

worker (self-confidence, locus of control, etc.) situation (degree of task challenge, ambiguity, etc.)

Delayed onset PTSD has a ________ prognosis a) similar b) better c) worse

worse

A delayed onset of PTSD is associated with what type of prognosis? a) better b) worse c) similar to early onset

worse (6months or more is associated with worse prognosis)

The NIMH has identified 10 symptoms of depression of Alzheimer's disease. Most are similar to those identified in the DSM-IV for Major Depressive Disorder. The new symptom is_______ and, for the diagnosis, this symptom or "clinically significant depressed mood" must be present.

"decreased positive affect or pleasure in response to social contacts and usual activities"

The_____standard is used to determine if a person's actions constitute sexual harassment but, if the supervisor stops his behavior, it is unlikely that the employee would have grounds for a suit against him.

"reasonable woman"

The correction for guessing formula is: Corrected Score =

(R) - (W/n-1), where R = number of right answers, W = number of wrong answers, and n = the number of choices per answer.

________ occurs when a person does not comply with a request because he/she feels that the request violates his/her sense of personal freedom.

(psychological) reactance

What are good predictors of reoffense of juvenile offenders?

- age of first contact with law (-.341) - length of first incarceration (.187) - age at commencement of the first offense (-.346) note: number of prior arrests not very good predictor (.058)

What does the most recent research on bilingual vs. immersion programs say (Thomas & Collier)?

- immersion programs okay short-term, bilingual programs are better long-term - Language minority children in bilingual programs at least through sixth grade catch up or surpass native English speakers and language minority children in immersion programs - Two-way bilingual programs best long-term outcomes for language minority children

What 3 areas are involved in the duty to warn?

1. fiduciary relationship 2. identifiable victim 3. forseeability of harm note: forseeability of harm is the biggest problem in debate about duty to warn when HIV-infected pt who is engaging in risky sexual behavior

Piaget- emergence of the ability to create mental reps of reality occurs btwn the ages of: a) 4-8months b) 8-12months c) 12-18months d) 18-24months

18-24months

Conflict, Dominance, Rebellion. Advice giving is replaced by criticism, judgmental statements, & other negative comments, & some members may express hostility toward the therapist as a result of resistance and the realization that they are not the therapist's favorite child

2nd stage of group development (Yalom)

Sleep terrors begin during what sleep stages?

3 and 4 (NREM)

Stages ___ & ___ sleep show a gradual decline over the lifespan, with older adults spending most of their sleep time in Stages ___ & ___ sleep.

3 and 4 (especially 4) 1 and 2

For most kids, expressions of pride, shame, and guilt are first evident by _____ months of age a) 12 b) 20 c) 30 d) 42

30

A GAF score in the range of _____ indicates the need for inpatient a) 20-25 b) 25-30 c) 30-35 d) 40-50

30-35

A GAF score btwn________ would be assigned to an individual who is severly depressed, unable to work, avoids friends, etc a) 21-30 b) 31-40 c) 41-50 d) 51-60

31-40

Development of cohesiveness. unity, intimacy, & closeness become the chief concerns. he describes cohesiveness as the analogue of the therapist-client relationship in individual therapy.

3rd stage of group development (Yalom)

introjection, projection, retroflection, confluence

4 major boundary disturbances

According to DSM, for approx 30-40% of indviduals with MR seen in clinical settings there is no clear cause. For those for whom the etiology is known-heredity is responsible in ____________ of cases a) less than 1 b) 5 c) 20 d) more than 50

5

Enuresis cannot be diagnosed before what age? a) 3years b) 4 years c) 5 years d) 6 years

5 years

Headache, tremor, and confusion, are characteristic of _____. a) tricyclic overdose b)Neuroleptic malignant syndrome (NMS) c)5-HT toxicity syndrome d)lithium toxicity

5-HT toxicity syndrome

People who quit smoking by age 50 reduce thr risk of dying prematurely from a smoking-related illness by ________ a) 85% b) 50% c) 25% d) 10%

50%

arise spontaneously in response to specific stimuli or situations. Reflect APPRAISAL of situation NOT actual situation. When dysfunctional they lead to maladaptive emotional & behavioral responses.

Automatic thoughts

role of societal factors in personality development. Identified 5 character styles that a person may adopt in response to demands of society: the receptive, the exploitative, the hoarding, the marketing, & the productive.

Erich Fromm (neo-Freudian)

addresses issues related to perception. 1) People tend to seek closure; 2) a person's 'gestalts' reflects his/her current needs; 3) a person's behavior represents a whole that is greater than the sum of its parts; 4) behavior can be fully understood only in its context; & 5) a person experiences the world in accord with the principle of figure/ground

Gestalt Psychology

based on premise that each person is capable of assuming personal responsibility for his/her own thoughts, feelings, & actions and living as an integrated "whole."

Gestalt Therapy

________ involves digression to irrelevant topics (in the extreme form it becomes loose associations/ derailment).

Tangentiality

An emphasis on contact, awareness and experimentation is central to which: a) Gestalt therapy b) solution-focused therapy c) Adlerian therapy d) rational-emotive therapy

Gestalt therapy

Freudian therapists emphasize intrapsychic determinants of personality. Neo-Freudian therapists emphasize social & cultural determinants of personality

A major difference b/w Freudian & neo-Freudian therapists

_______requires the individual to learn and recall a list of words in a particular order

A serial learning task

All of these are associated with highest risk of suicide completion except: a) male gender b) single c) older d) African Am

African Am...Whites have higher rates of completed

Who is most likely to have hypertension?

African American women over 60

The mandate that employers make reasonable accommodations when testing examinees with disabilities is part of what law?

Americans with Disabilities Act (ADA)

Which PD is least disrupted by affect and symptom quieting in mid-life? a) BPD b) Antisocial PD c) Narcissistic PD d) Schizoid PD

Antisocial PD

The ______is a 15-point scale that evaluates level of consciousness through eye, verbal, and motor responses.

Glasgow Coma Scale

_____ is used to eliminate habits and other undesirable repetitive behaviors including tics, trichotillomania, and stuttering.

Habit reversal training

recognized the role of cognitive experience in personality development. Identified 3 modes of cog. experience, which develops sequentially in the infant--Prototaxic, parataxic, & syntaxic mode

Harry Stack Sullivan (neo-Freudian)

____ occurs when a CS blocks the ability of a second neutral stimulus from becoming a CS.

Blocking

____ occurs when an association between the CS and the US has been established before the CS and a new neutral stimulus are presented together before the US. In this situation, the new neutral stimulus provides redundant information and, consequently, does not become associated with the US and does not produce a CR.

Blocking

health behaviors influenced by: 1) person's readiness to take a particular action, which is related to his/her perceived susceptibility to the illness & perceived severity of its consequences; 2) the person's evaluation of the benefits & costs of making a particular response; 3) internal & external cues to action that trigger the response

Health belief model (Becker, 1974)

grounded in the situation, depends on group understanding, relies heavily on non-verbal cues, helps unify a culture, & is slow to change.

High context Communication

Transference. Parataxic distortions occur when an individual deals with others as if they were significant persons from his/her early life.

Sullivan's concept of parataxic distortion is most similar to the Freudian notion of

At the 1st cousin level, schizotypals have increased rate of ______ in their family tree: a) schizoidPD b) Sx c) Antisocial PD d) histrionic PD

Sx

T/F A speed (or speeded) test contains a large number of questions that are so easy that most examinees would be able to answer them correctly if given sufficient time.

T note: the oppositive is a power test (with hard items that couldn't all be answered correctly, regardless of the time)

Hippocampus is to explicit memory as __________ is to implicit memory: a) septum b) thalamus c) medulla d) cerebellum

Cerebellum

Which is true regarding family influences in the dvpt of language? a) Children without siblings (only children) tend to display accelerated language development. b) Identical twins reared together tend to display accelerated language development. c) Socioeconomic status appears to have no effect on language development. d) The presence of an older sibling accelerates language development.

Children without siblings (only children) tend to display accelerated language development.

Researchy investigating thel benefitsa off psychologicall treatmentsa for children, jadolescents, jands jadultsa withy cystic fibrosisa isa very limited. However, thel existing studiesa suggest that whichy off thel following japproachesa isa likely to bel most beneficiall for treating depression, janxiety, pain, jands stressa jassociateds withy thisa illness? a)cognitive-behavioral therapy b)client-centered therapy c)solution-focused therapy d)reality therapy

Cognitive behavioral therapy

absence of boundary between the self and the environment. causes intolerance of any differences between oneself & others & often underlies feelings of guilt & resentment

Confluence

to enhance the consultee's performance in delivering services to a particular population or group of clients. focus on consultee's skills, knowledge, abilities, &/or objectivity

Consultee-centered case consultation

T/F Rett's disorder only occurs in females.

T note: this disorder involves receptive and expressive language deficits following a period of normal development.

designed to reduce the duration & consequence of mental & physical disorders. Rehab programs, partial programs; education programs

Tertiary Prevention

choosing an intervention; formulating a plan; & implementing the plan

Implementation

Movement toward others; movement against others; or movement away from others. A healthy individual incorporates all 3 behaviors. A neurotic person relies on one type of behavior.

In Horney's conceptualization how does a child defend against feeling of anxiety?

With the elderly, paranoid reactions are possible associated features of ___, ____ and _____.

Delirium and Dementia and are specifically associated with Alzheimer's Disease

_____ occurs when parents avoid the tension between them by either blaming or overprotecting a child.

Detouring

On the WISC, the working memory index composite score is derived from scores on which subtests? a) Similarities, Vocabulary, and Information b) Block Design, Picture Concepts, and Matrix Reasoning c) Coding, Symbol Search, and Cancellation d) Digit Span, Letter Number Sequencing, and Arithmetic

Digit Span, Letter Number Sequencing, and Arithmetic

_____ seems to be responsible for at least some forms of both mania and schizophrenia, with increased levels of or oversensitivity to this neurotransmitter being predictive of both disorders.

Dopamine

Who distinguishes btwn Steady State, Linear, Spiral & Transitory Career Concepts? a) Super b) Holland c) Driver d) Krumboltz

Driver-he emphasized these career concepts in career decision-making

correlational or quasi-experimental

Effectiveness studies

______ refers to the process of increasing the number of associations between items of information, which enhances the meaningfulness of the information and is associated with improvements in retention and retrieval.

Elaboration

What therapy: assumes that irrational ways of thinking underlie dysfunctional behaviors.

Ellis' REBT

What is the difference btwn escape and avoidance conditioning? a) Escape conditioning is the result of positive reinforcement,‭ ‬while avoidance conditioning is the result of negative reinforcement. b) Escape conditioning is the result of negative reinforcement,‭ ‬while avoidance conditioning is the result of negative punishment. c) Escape and avoidance conditioning are both the result of negative reinforcement,‭ ‬but only escape conditioning combines negative reinforcement with classical conditioning. d) Escape and avoidance conditioning are both the result of negative reinforcement,‭ ‬but only avoidance conditioning combines negative reinforcement with classical conditioning.

Escape and avoidance conditioning are both the result of negative reinforcement but only avoidance conditioning combines negative reinforcement with classical conditioning

personality is an outgrowth of the struggle b/w the individual & the "ultimate concerns" of existence--death, isolation, meaninglessness, & the fact that we are ultimately responsible for our own lives.

Existential therapy

T/F Partial seizure cannot involve loss of consciousness.

F Simple - no LOC, Complex - LOC

T/F The race of an examiner has no effect on testing performance for minority group examinees

F The research is inconsistent

T/F Matching a therapist and client on race/ethnicity is shown to reduce attrition but have NO impact on outcome for ANY groups.

F - there have been mixed results for different groups

The MMPI-2 includes several validity scales that provide information on an examinee's test-taking attitudes and style. An extremely high score on the ____indicates carelessness in responding or a deliberate attempt to "fake bad."

F scale

The right of parents to request a school to amend their minor child's school records if they believe the record is inaccurate or misleading is part of what law?

Family Educational Rights and Privacy Act (FERPA) Note: also called Buckley Ammendment

_____, is the belief that every event has a purpose and is characteristic of the ____ stage.

Finalism, preoperational

Orientation, Hesitant Participation, Search for Meaning. Determine the group's structure & meaning. members look to leaders for approval & acceptance as well as for answers to their questions.

First stage of group development (Yalom)

is a hereditary condition that causes mental retardation and other cognitive impairments as well as certain physical and behavioral abnormalities.

Fragile X syndrome

_____ perspective, feedback refers to the return or provision of information to the system about the consequences of an event or action.

From a family systems

Clinical Psychology 79. Traditional psychoanalytic psychotherapy and brief therapies derived from it share several characteristics but also differ in a number of ways. Which of the following is not true about brief forms of psychodynamic psychotherapy? A In brief psychotherapy, past experiences are important to the degree that they contribute to the client's current conflicts. B Brief therapy targets specific therapeutic goals, which are identified during the first few sessions. C In order to solve a client's problems rapidly, brief therapy makes use of the "transference cure" rather than fostering the client's insight into his or her problems. D Brief therapy can be described as a problem-solving process in which the therapist helps the client use his or her own resources to resolve current difficulties.

In order to solve a client's problems rapidly, brief therapy makes use of the "transference cure" rather than fostering the client's insight into his or her problems. This is the opposite of what is true. In brief forms of psychodynamic psychotherapy, client insight is desirable, although it is limited to those areas that are identified as the targets of therapy.

in order to be effective feminist therapists believe they must be social & political activists.

Involvement in Social Action

analytic psychology was developed by

Jung

What theory: the basic elements of the psyche are the conscious (which includes the ego) and the unconscious (which includes a personal and collective component).

Jung's analytic psychology

The MMPI-2 includes several validity scales that provide information on an examinee's test-taking attitudes and style: This is used to correct scores on several clinical scales.

K scale

Parenteral behaviors such as indifference, overprotection, & rejection cause child to feel basic anxiety--a feeling of helplessness & isolation in a hostile world.

Karen Horney (neo-Freudian)

The MMPI-2 includes several validity scales that provide information on an examinee's test-taking attitudes and style: This includes items that apply to or describe the majority of people.

L scale

effect size estimates for psychological treatments equal to or exceed those for medical & educational interventions& that reported effect sizes are sufficiently large to support the claim that psych. treatments are "generally efficacious in practical as well as statistical terms."

Lipsey & Wilson (1993) concluded what?

average effect size of .67 when treated subjects are compared to no treatment controls, but a smaller effect size of .48 when subjects in the treatment group are compared to those in a placebo control group.

Lipsey & Wilson (1993) on placebo control groups

Who? applied the term "critical period" to the period during which ducklings bond (or imprint) with their mothers. ___ found that the "critical period" for imprinting in ducklings is 2-3 days after birth. In other words, ducklings will stay close to stimuli which they are exposed to 2-3 days after birth. Research investigating the existence of a similar critical period in humans has been inconclusive.

Lorenz

relies primarily on the explicit, verbal part of a message. less unifying & can change rapidly & easily. characteristic of Euro-American cultures.

Low context communication

personality development: first month--normal autism--infant essentially oblivious to external environment; 2-3 months--normal symbiosis--fusion with mother; a) individual b)psychoanalyisis c)object re

Mahler object relations personality development

_____ distinguished between personal and socialized power

McClelland

What racial group has the highest lifetime rate of MDD in the United States?

Native Americans

____ is characterized by painful joints, mutism, tachycardia, urinary retention muscle rigidity, hyperthermia, and stupor

Neuroleptic malignant syndrome (NMS)

is a rare, idiosyncratic reaction to neuroleptic drugs

Neuroleptic malignant syndrome (NMS)

The ____ is a screening test for dementia.

Neuropsychiatric Inventory Questionnaire (NPI-Q)

Who is most likely to receive psychiatric hospitalization?

Never married White men between 25 and 44

_______ incorporates several strategies including cognitive restructuring, relaxation training, breathing retraining, and in vivo exposure.

Panic Control Treatment (PCT)

involves private or autistic symbols. person sees causal connections between events that are not actually related. the connections serve the developing self & reduce anxiety

Parataxic mode of cognitive development

What therapy: the result of a lack of "awareness."

Perls' Gestalt therapy

is a congenital disorder that is caused by a lack of anti-Mullerian hormone or receptors for this hormone.

Persistent Mullerian duct syndrome

_________ __________ theory (George Kelly) is a personality theory and therapy approach baed on the premise that people construe their own experiences.

Personal Construct (constructive alternativism) Note: influenced narrative constructivist therapy approaches

aimed at reducing the prevalence of mental & physical disorders by decreasing incidents of new cases. E.g. immunization programs, "Meals on Wheels," etc.

Primary prevention

____ are designed to keep a problem from developing.

Primary preventions

__________ interference occurs when the first thing learned interferes with the second thing learned.

Proactive

If a clinician moves from one clinic to another, should she take clients from her previous clinic with her?

Probably not Instead, take steps to minimize confusion and conflict when a client is receiving services elsewhere.

Disavowing aspects of the self by assigning them to other people. extreme projection can result in paranoia

Projection (Gestalt)

discrete, unconnected momentary states. occurs in the 1st months of life.

Prototaxic mode of cognitive development

Although sedative drugs increase the total sleep time, they decrease the amount or proportion of ________.

REM sleep Note: "REM rebound" (more than normal REM sleep) occurs after abrupt cessation of sedative drugs.

_____ refers to the fact that each individual responds to the environment in a unique way because of his/her genetic endowment.

Range of reaction

treatment where illogical self defeating thoughts are identified and the client is helped to replace them with more realistic and beneficial ones

Rational emotive therapy

For _____, temperature biofeedback appears to be superior to relaxation training.

Raynaud's disease

The underlying problem of all clients is the same: they are either involved in a present unsatisfying relationship or lack what could even be called a relationship. Primary need is to be love, belong, and feel worthwhile "WDEP" W= wants and needs; D= direction and doing; E= evaluation; and P = planning and commitment. Chief figures is William Glasser and

Reality therapy

_____ is a paradoxical technique that involves providing an alternative meaning for a behavior so that it can be seen from a new perspective.

Reframing

3rd phase of therapy. focus on "unlearning troublesome, maladaptive habitual behaviors & establishing new ways of dealing with various aspects of life. # of sessions depends on type & severity of problem

Rehabilitation

_______ are about equally effective for tension headaches.

Relaxation and biofeedback

____ has been found helpful in some cases of chemotherapy-induced nausea and vomiting; ____ is not a recommended treatment for this problem.

Relaxation, biofeedback

2nd phase of therapy. focus on symptoms that brought person to therapy. symptomatic relief usually requires about 16 session

Remediation

feelings of hopelessness & desperation respond quickly to therapy, usually accomplished during 1st few session

Remoraliziation

doing to oneself what one wants to do to another

Retroflection

Any single subject design that includes the removal of treatment to determine if the subject reverts to baseline:

Reversal design

Taking aspirin during or shortly after a viral, fever-causing illness has been linked to ________ syndrome, a rare disorder affecting the brain and liver.

Reyes

The MMPI-2 includes several validity scales that provide information on an examinee's test-taking attitudes and style: This is a measure of response consistency.

TRIN and VRIN scales

_______ involves an inability to recognize familiar objects by touch alone despite adequate sensation in the fingers.

Tactile agnosia

1) Intercultural Nonparanoiac Disorder (low function, low cultural)--willing to disclose to any therapist; 2) functional paranoiac (high functional, low cultural paranoia)--nondisclosive to AA or white therapists primarily due to pathology. 3) healthy cultural paranoiac (low functional, high cultural) client will disclose to an AA therapist, but reluctant to disclose to a white therapist. 4) Confluent Paranoiac (high functional, high cultural) nondisclosing to either AA or white therapist combination of pathology & effects of racism

Ridley (1984) model of paranoia has 2 levels (low v. high) & the combination of these levels produces 4 disclosure modes:

The most effective treatment for OCD involves exposure and ________ a) tricyclics b) SSRIs d) MAOIs d) benzos

SSRIs

____ have been found effective for treating bulimia.

SSRIs

attempts to decrease prevalence of mental & physical disorders by reducing their duration via early detections & intervention. identifying specific individuals & providing those individuals with appropriate treatments

Secondary Prevention

____ are aimed at early identification and intervention

Secondary preventions

serves as person's major source of motivation & guides them toward positive, healthy growth

Self-actualizing tendency

begins around 4 months. 1st sensory exploration of the the environment. then physical exploration of the world. Followed by period of conflict between independence & dependence--separation anxiety. by 3 yrs. child has developed a permanent sense of self & object & is able to perceive others as both separate & related

Separation-Individuation phase of Mahler's development

To determine the degree of assoc btwn 2 variables that are reported in terms of ranks. you would use which of the following correlation coeffs? a) contingency b) biserial c) phi d) Spearman

Spearman

young, white females. the most frequent callers (Miller, et al., 1984)

Suicide hotlines reduce rates in what group?

"Just noticeable differences" are: A psychologically equal intervals. B physically equal intervals. C psychologically and physically equal intervals. D psychologically and physically unequal intervals

The Correct Answer is "A" A just noticeable difference (or difference threshold) is the smallest physical difference between two stimuli that can be recognized as a difference. Although just noticeable differences are considered to represent psychologically equal units, their corresponding physical differences are unequal. For example, a just noticeable difference for 10 pounds might be the addition of 1 pound, while the just noticeable difference for 100 pounds might be the addition of 10 pounds.

Chronic pain treatment with tricyclic antidepressants is most effective for: A neuropathic pain and headaches B neuropathic and musculoskeletal pain C headaches and musculoskeletal pain D neuropathic pain, headaches and musculoskeletal pain

The Correct Answer is "A" A. Chronic pain research estimates 50-90% of patients can be expected to achieve at least 50% pain relief with antidepressants, with others achieving less pain relief. Tricyclic antidepressants, such as amitriptyline (Elavil), have been found to be most effective particularly if the pain is neuropathic (pain in a numb area, burning or shooting pain) or one of the headache syndromes. SSRI antidepressants have also been found to be effective at preventing headaches, including migraines, but are considered less effective. It is theorized these medications may reduce migraines by affecting the level of serotonin and other brain chemicals. Individuals do not have to have depression to take and benefit from these drugs. A therapeutic trial of antidepressants may follow the failure of conventional analgesics or be used in addition to conventional analgesics. This can be particularly effective in cancer patients who have pain at multiple pain sites, some nociceptive and some neuropathic. (See: McQuay, H. J., Tramer, M., Nye, B.A., Carroll, D., Wiffen, P. J., Moore, R. A. (2000). A systematic review of antidepressants in neuropathic pain. Pain. 1996, 68(2-3), 217-227. and Barkin, R.L., Fawcett, J. (2000). The management challenges of chronic pain: the role of antidepressants. American Journal of Therapy, 7(1),31-47.)

Starvation in Anorexia is believed to be precipitated by: A high levels of serotonin B low levels of serotonin C low levels of tryptophan D low levels of serotonin and tryptophan

The Correct Answer is "A" A. High levels of serotonin have been linked to both appetite suppression and anxiety while low levels have been linked to depression and appear to contribute to binge eating. Research by Kaye et al (1997) proposes that anxiety is caused by high levels of serotonin and individuals with Anorexia temporarily relieve anxiety through starvation which reduces tryptophan, which then reduces the level of serotonin. The lowered serotonin fosters a sense of calm and personal control. Inversely, low levels of serotonin are believed to precipitate binge eating in Bulimia as sweets and carbohydrates increase serotonin and elevate mood. (See: Kaye W.H., Weltzin T.E. Serotonin activity in anorexia and bulimia nervosa: relationship to the modulation of feeding and mood. Journal of Clinical Psychiatry, 1991; 52:41-58. and Kaye W.H. Serotonin function in eating disorders. Biological Psychiatry, 1997: 42:56.)

A patient displays apraxia, tactile agnosia, difficulties with movement, and confusion between the left and the right side of the body. Which lobe of the person's cerebral cortex is most likely damaged? A parietal B frontal C occipital D temporal

The Correct Answer is "A" Questions like this can be difficult to answer, since damage to any of the lobes can cause a wide variety of impairment, depending on the location and extent of the damage. Moreover, some types of impairment (including the type described by this question) can be caused by damage in the association areas of the cortex, or the areas where the different cortical lobes merge. Nonetheless, the parietal lobe is the best answer to this question. More than any other lobe, damage to the parietal lobe has been associated with left-right confusion. In addition, tactile agnosia (the inability to identify objects by touch using the contralateral hand) is associated primarily with the parietal lobe, which contains the primary somatosensory cortex. Difficulties in body movement and apraxia (inability to carry out motor activities despite intact comprehension and motor function) can be caused by damage to the frontal lobe as well as damage to the parietal lobe.

The most common type of work-family conflict is: A time-based B strain-based C behaviorally-based D systemically-based

The Correct Answer is "A" A. Research suggests that high levels of work-family conflict are related to negative outcomes for the individual (life dissatisfaction, anxiety, poor health), for relationships (increased interpersonal conflict, divorce), and for the organization (absenteeism, tardiness, loss of talented employees). Work-family conflict can be time-based, strain-based, or behavior-based. Time-based conflict, which occurs when role pressures stemming from the two different domains compete for the individual's time, is the most common type of work-family conflict. It is based on the scarcity hypothesis, that the sum of a person's energy is fixed; therefore multiple roles inevitably reduces the time and energy available to meet all role demands, resulting in strain and work-family conflict. Much of the research on work-family conflict has been based on this premise proposed by role theory and role scarcity. Work-family conflict is bi-directional; work can interfere with family and family can interfere with work. These are referred to as work-to-family conflict and family-to-work conflict. Recent research, based on the enhancement theory, suggests quality multiple roles provide additional sources of social support, increased skills, and heightened self-esteem and well being. Evidence of positive spillover, both from work to home and from home to work, continues to be found. Strain-based (b.) conflict occurs when the strain experienced in one role domain interferes with effective performance of role behaviors in the other domain. Behavior-based (c.) conflict is described as conflict stemming from incompatible behaviors demanded by competing roles. Recent research, using a systems perspective, demonstrates that one member of a dyad can impact the other's experience of work-family conflict. The effects of this work and family stress and strain are referred to as "crossover effects." (See: Barnett, R. C. and Hyde, J. S. (2001) Women, men, work, and family: An expansionist theory. American Psychologist, 56(10), 781-796.)

A person's negative attitudes toward an individual because of his or her sexual orientation is referred to by Gregory Herek (2000) as: A sexual prejudice B homophobia C homosexism D gay inequality

The Correct Answer is "A" Gregory Herek, an authority on prejudice against lesbians and gay men, suggests the use of the term "sexual prejudice" to refer to all negative attitudes based on sexual orientation, whether the target is homosexual, bisexual, or heterosexual. Herek suggests that "sexual prejudice" is a more appropriate term than "homophobia" (B) - which refers to heterosexuals' dread of being in close quarters with homosexuals and homosexuals' self-loathing.

In assessing perceptual abilities in a 3 or 4-month old infant you could use all of the following indicators except: A head turning B sucking C reaching D heart rate

The Correct Answer is "A" Head turning does not become an appropriate measure of perception in infants until 5 ½ months of age. Sucking (B) is often used as a measure of perceptual abilities for infants aged 1 to 4 months. Reaching (C) is used at 12 weeks or older, and heart rate (D) can be a useful measure at any age.

A difference between tangentiality and circumstantiality is that: A in circumstantiality, the person is delayed in reaching the point, whereas in tangentiality, the person avoids reaching the point at all. B in tangentiality, the person is delayed in reaching the point, whereas in circumstantiality, the person avoids reaching the point at all. C tangentiality is speech that is not understandable, due to the lack of a meaningful connection between words or sentences; by contrast, circumstantial speech is understandable. D circumstantiality is speech that is not understandable, due to the lack of a meaningful connection between words or sentences; by contrast, tangential speech is understandable

The Correct Answer is "A" If you were not familiar with the term "tangentiality," thinking about the phrase "going off on a tangent" might have helped you. Tangentiality is defined as responding to a question in an oblique or irrelevant way; in other words, the person avoids the point entirely. Circumstantiality, by contrast, is a pattern of speech that is delayed in reaching the point due to much irrelevant detail or many parenthetical remarks. Eventually, however, the point is reached.

The clinical scales of the MMPI-2 were developed using empirical criterion keying. This means that A items which distinguish between specific subgroups of people are retained for the final version of the test. B items that have high correlations with the previous version of the test are retained for the final version of the test. C items that have high correlations with other tests designed to measure similar constructs are retained for the final version of the test. D items which appear to measure what they are designed to measure are retained for the final version of the test.

The Correct Answer is "A" In empirical criterion keying, items are assessed to determine how well they distinguish between prespecified criterion groups (e.g., between depressed and non-depressed individuals). The items that best distinguish between the groups are maintained for the final version of the scale or the test

An 18-year-old high school student is referred to you for personality assessment due to behavioral problems. Which of the following assessment instruments would be most appropriate to use? A MMPI-A B MMPI-2 C WAIS-III D Halstead-Reitan

The Correct Answer is "A" Knowing that the MMPI-A is recommended for use with ages 14 to 18, while the MMPI-2 is recommended for individuals 18 years and older, may have left you a bit stumped about which to use for an 18-year-old. The MMPI-A manual acknowledges this period of overlap and recommends choosing the instrument on a "case by case basis," but it also suggests using the MMPI-A for high school students. Furthermore, in a recent study comparing the MMPI-A and MMPI-2 administered to 18-year-olds, the researchers concluded that the MMPI-2 tended to overpathologize and the MMPI-A tended to underpathologize. Due, in part, to the concern with overpathologizing individuals, the researchers recommended using the MMPI-A with 18-year-olds. But to optimally gain sensitivity to psychopathology, they also recommended using T-score cutoffs of 60 instead of 65 on the MMPI-A [T. M. Osberg and D. L. Poland, Comparative accuracy of the MMPI-2 and the MMPI-A in the diagnosis of psychopathology in 18-year-olds, Psychological Assessment, 2002, 14(2), 164-169].

Following a stroke, a patient exhibits right hemiplegia. Other symptoms are likely to include: A speech-language deficits and slow-cautious behavior style. B spatial-perceptual deficits and slow-cautious behavior style. C speech-language deficits and quick-impulsive behavior style. D spatial-perceptual deficits and quick-impulsive behavior style

The Correct Answer is "A" Note that the individual has right-sided hemiplegia, which makes the left side of the brain the area that has been affected. This should have helped you narrow the choices down to responses a and c since the left side of the brain is responsible for language. At that point you may have had to guess. Now you know that left hemisphere damage is associated with a slow-cautious behavioral style (and that right hemisphere damage is associated with a quick-impulsive style).

A seizure that begins with uncontrollable twitching of a small part of the body and gradually spreads throughout one side or the entire body is referred to as: A partial B petit mal C tonic-clonic D generalized

The Correct Answer is "A" Seizures are generally divided into two main types: "partial" or "focal," and generalized. Partial (focal) seizures typically begin with uncontrollable twitching of a small part of the body, for example, one finger, which may start to jerk, followed by jerking of the entire arm and then the rest of that side of the body, and may lead to jerking of the entire body. Although it can eventually affect the entire body, it is referred to as a "partial" seizure because of how it initially developed. This is sometimes called "partial seizure with secondary generalization." Partial seizures are further subdivided into "simple partial seizures," which have no alteration of consciousness, and "complex partial seizures," which do alter consciousness. The generalized type of seizures include the petit mal and tonic-clonic (grand mal) types. Petit mal (absence) seizures are characterized by minimal motor activity and a lack of awareness. Generalized tonic-clonic (grand mal) seizures begin as bilaterally symmetrical at onset and involve episodes of violent shaking, during which the person becomes blue and stiff.

The coefficient of determination indicates A the proportion of variability in one variable that is accounted for by variability in another variable. B the correlation between two variables with the effects of a third variable removed. C the correlation between two variables without the effects of a third variable removed. D the proportion of variability accounted for in a variable by all the factors in a factor analysis

The Correct Answer is "A" The coefficient of determination is calculated by squaring a correlation coefficient. As compared to the correlation coefficient, it provides a more direct way of interpreting the calculated relationship between two variables. Specifically, it indicates the proportion of variability shared by the two variables, or the proportion of variability in one variable that can be accounted for by variability in the other.

According to Theodore Millon, people with Narcissistic Personality Disorder rely primarily on which of the following defense mechanisms? A repression, rationalization, and projection B repression, displacement, and sublimation C rationalization, reaction formation, and sublimation D identification, intellectualization, and displacement

The Correct Answer is "A" This is a difficult question unless you are a Theodore Millon fan. According to Millon, when narcissistic people experience personal failure and public humiliation, they resort to defense mechanisms -- first repression, and if that fails, then rationalization and projection.

In order to increase the chances that Mary Ann is altruistic it would help if she A has younger siblings. B comes from a large extended family. C is inherently wealthy. D has at least one older sibling

The Correct Answer is "A" This is one of those questions that would even cause social psychologists to despair. Nevertheless, a very similar question has been on the examination. You would have chosen answer A if you were familiar with B. Whiting and J. Whiting's study of six cultures. They found that responsibility for the welfare of others was the most important factor in the development of altruism (Children of Six Cultures, Cambridge, MA, Harvard University Press, 1975). Answer B is incorrect; Cohen's study found altruism linked more to a nuclear than an extended family structure (R. Cohen, Altruism: Human, cultural, or what? Journal of Social Issues, 28(3), 39-57). Answer C and D have not been found to correlate with any of the research.

Your client has moved out of state. She has called and left you a voice mail message requesting that you send her records to a new therapist. She leaves her new therapist's number and address but not her own. You have a current written release from her on file. You note that she has one outstanding balance of $200. You should A contact the therapist and try to get the client's number. B send all the records. C send a summary along with an invoice. D send the summary along with an invoice indicating the amount still owed.

The Correct Answer is "A" This would be your best choice. It never hurts to take the most conservative approach, which in this case would be to try and contact your client and discuss the situation with her. It will be helpful to know the purpose of the request. Your client has a right to a copy of her records and a right to have them sent to where she chooses, but your best course of action would be to discuss the situation with her. You would not send her bill to another therapist

. A percentile rank is A a norm-referenced score, but not a standard score. B a standard score, but not a norm-referenced score. C a standard score and a norm-referenced score. D neither a standard score nor a norm-referenced score.

The Correct Answer is "A" To answer this question, you have to be able to define and understand three terms: norm-referenced, standard score, and percentile rank. A norm-referenced score is one that is interpreted in terms of a comparison to others who have taken the same test. A standard score is a type of norm-referenced score that is interpreted in terms of how many standard deviation units a score falls above or below the mean. Examples include z-scores and T-scores. A percentile rank indicates the percentage of scores that fall below a given score. For example, a person who achieves a percentile rank of 90 on the SAT scored better than 90% of others who took the test. Since interpretation of percentile ranks involves a comparison between scorers, a percentile rank is a norm-referenced score. However, since it is not interpreted in terms of standard deviation units, it is not a standard score.

When several individuals are to be compared by a supervisor in a work setting, paired comparisons, as compared to order of merit comparisons, are A more precise but more difficult. B less precise but easier. C more precise and easier. D less precise and more difficult

The Correct Answer is "A" When using the paired comparison technique, a rater compares each person with every other person. It's specific but time-consuming, as you can see. The order of merit comparisons has the rater rank order the people in terms of some criterion, such as overall job performance. You can see that it wouldn't take as long as paired comparisons, but wouldn't be as precise either.

Berry, who views acculturation as a multidimensional construct, would describe an integrated client as one who: A Has a low retention of the minority culture B Has high maintenance of the mainstream culture C Rejects the mainstream culture but has a high retention of the minority culture D Gets along with others in the workplace

The Correct Answer is "B" J. Barry conceptualizes acculturation as many models existing on a continuum, with the minority culture and the majority or mainstream culture at opposite poles. Integration would be displayed by an individual who has high retention of the minority culture and high maintenance of the mainstream culture

Kohler's classic studies of the role of cognition in learning examined the behaviors of: A apes B dolphins C ducklings D rats

The Correct Answer is "A" Wolfgang Kohler is best known for his research with chimpanzees, which is a type of ape. (Apes are considered more like humans than monkeys or lower primates.) In his classic experiments Kohler placed food outside the reach of the chimps to assess their problem solving abilities. In one study, food was placed outside the chimp's cage, and the chimp was given two sticks, neither of which was long enough by itself to reach the food. As the chimp was sitting with the two sticks in his hand, he suddenly seemed to have an "a-ha!" experience: He quickly fit the two sticks together and used the new, elongated stick to reach the food. Kohler called this sudden novel solution "insight learning," which he attributed to a sudden cognitive restructuring of the environment. If you chose ducklings (A) you were probably thinking of Conrad Lorenz' research on instinctual behaviors.

. To determine the relationship between a dichotomous variable and a continuous variable, you would use which of the following correlation coefficients? A point biserial B biserial C Spearman's Rho D eta

The Correct Answer is "A" You should memorize the different correlation coefficients and when they are used. The point-biserial coefficient is used when a dichotomous variable (e.g., gender) is correlated with continuous variable (e.g., IQ score). You might have thought the biserial coefficient is also correct, since it is used to correlate an artificial dichotomy with a continuous variable. An artificial dichotomy is one that is created arbitrarily by setting a cutoff score on a test; for instance, if you give the WAIS-III and classify everybody who scores over 110 as having "high intelligence" and everybody who scores below 110 as having "low intelligence," you have created an artificial dichotomy. Because an artificial dichotomy is not, in a pure sense, a dichotomous variable, choice B is not as good an answer as choice A.

A new professor is teaching her first class on feminist theory. What is she required to do? A Present a balanced view of feminist theory vs. other theories B Concentrate solely on feminist theory; it is a given that there are other views of psychology C Use it as a forum for presenting her dissertation findings D Present other theories, but explain why feminist theory is the most appropriate paradigm for women

The Correct Answer is "A" is the best choice. While B and D could both possibly be an option, (A) is more in line with Standard 6.03 (a) which deals with objectivity: "When engaged in teaching or training, psychologists present psychological information accurately and with a reasonable degree of objectivity

The technique which allows a researcher to identify the underlying (latent) factors that relate to a set of measured variables and the nature of the causal relationships between those factors is: A structural equation modeling (SEM) B cluster analysis C Q-technique factor analysis D survival analysis

The Correct Answer is "A" A. Structural equation modeling is a multivariate technique used to evaluate the causal (predictive) influences or test causal hypotheses about the relationships among a set of factors. Cluster analysis (b.) is used to identify homogeneous subgroups in a heterogeneous collection of observations. Q-technique factor analysis (c.) determines how many types of people a sample of people represents. Survival analysis (d.) is used to assess the length of time to the occurrence of a critical event.

Most well-controlled studies of the effect of cultural moderators on the validities of job selection tests indicate that there are A no significant differences in validities between majority and minority groups. B significant differences in validities between majority and minority groups. C significant differences in validities between majority and minority groups on general aptitude tests but not on assessments of motor skills. D significant differences in validities between majority and minority groups on assessments of motor skills but not on tests of general aptitude.

The Correct Answer is "A" While group differences exist between African-Americans and Caucasians on test performance (that is, while mean scores differ on these various standardized tests), it has not been shown conclusively that the predictive validity coefficients of the tests are differentially moderated by ethnic group membership. In other words, a test is likely to have the same validity coefficient for all subgroups of the same population -- even though those subgroups may score differently, on the average, from each other.

The concept of "identical elements" is most applicable to: A personnel selection B employee training C job analysis D Equity Theory

The Correct Answer is "B" "Identical elements" is a concept based on the work of learning theorists Thorndike and Woodworth in 1901. It refers to the notion that training is best transferred over to situations which are similar to, or which have "identical elements" with, the training environment. Research has confirmed that identical elements improves transfer of training for both verbal and motor tasks.

A parent brings his 7-year old child in for evaluation, with the complaint that the child can't sit still, is constantly on the go, and never pays attention. You suspect that the diagnosis might be Attention Deficit/Hyperactivity Disorder. Of the following, which would be the best way to confirm this diagnosis? A determine if the child displays the same exact behaviors in other settings B investigate the child's behavior in different settings with the awareness that the disorder may manifest itself differently in different situations C investigate the parent's interaction with the child D administer Ritalin to the child for a limited period of time to determine if the symptoms respond to the medication

The Correct Answer is "B" A diagnosis of ADHD requires the presence of characteristic symptoms in at least two settings. Therefore, it is necessary to investigate the child's behavior in other settings, such as school. However, the clinician needs to be aware that the disorder's nature and severity can differ across different situations. For instance, according to the DSM-IV, symptoms typically worsen in situations that require sustained attention or mental effort, and they may be minimal or absent when the child is frequently rewarded, is under very strict control, or is in a novel setting.

Cognex and Aricept are used in the treatment of Alzheimer's Dementia and are believed to A increase serotonin levels. B increase ACh levels. C decrease glutamate levels. D decrease serotonin levels

The Correct Answer is "B" Don't expect to get this right the first time. What is important is that you now know that tacrine hydrochloride (Cognex) and donepezil hydrochloride (Aricept) have beneficial effects on memory and reasoning for some patients with mild to moderate Alzheimer's Dementia. It is hypothesized that these drugs prevent the breakdown of ACh and thus increase the availability in the brain. While these drugs seem to help temporarily, they are not considered a cure. Serotonin has not been shown to be involved in Alzheimer's. While glutamate may be involved, the drugs mentioned do not target this neurotransmitter

A psychologist believes that physical exercise can reduce a person's anxiety level, which reduces the strength of substance cravings in people recovering from substance dependence. According to this hypothesis anxiety is a: A suppressor variable B mediator variable C moderator variable D criterion contaminator

The Correct Answer is "B" A mediator variable is a variable that accounts for or explains the effects of an IV on a DV. That is, the IV affects the mediator variable, which affects the DV. In this example, the IV is exercise, the mediator variable is anxiety, which explains how the DV, substance craving, is reduced. A moderator variable (C) is similar to a mediator variable, but a moderator variable only influences the strength of the relationship between two other variables, it doesn't fully account for it. For example, if a job selection test has different validity coefficients for different ethnic groups, ethnicity would be a moderator variable because it influences the relationship between the test (predictor) and actual job performance (the criterion) but it does not fully account for the relationship. A suppressor variable (A) reduces or conceals the relationship between variables. For example, the K scale in the MMPI-2 is a suppressor variable because it measures defensiveness, which can suppress the scores on the clinical scales. The K scale is, therefore, used as a correction factor for some of the clinical scales. Criterion contamination (D) is the artificial inflation of validity which can occur when raters subjectively score ratees on a criterion measure after they have been informed how the ratees scored on the predictor.

According to Edgar Schein, the concept of career anchor refers to: A the tendency to stay in a financially secure position regardless of personal interest B the motivation of priorities that define how people see themselves and their work C the motivation of advancement opportunities to stay with an organization D the tendency to stay in a position regardless of advancement opportunities

The Correct Answer is "B" A person's career anchor is his or her self-concept consisting of self-perceived talents and abilities, basic values, motives, and needs as they pertain to the career. Schein says that people are primarily motivated by one of eight anchors — priorities that define how they see themselves and how they see their work. The eight anchors include: technical/functional competence - a desire to excel in a chosen line of work; general managerial competence - closely allied with the traditional career path of the corporation; autonomy/independence - individuals just want to be alone; security/stability - value predictable environment in which tasks and policies are clearly codified and defined; entrepreneurial creativity - desire to create something of own and run it; service/dedication to a cause - the need to focus work around a specific set of values; pure challenge - seek to solve or master challenges; and lifestyle - organized around an individual's private life. (See: Schein, E. H. (1978). Career Dynamics: Matching Individual and Organizational Needs. Reading, MA.: Addison-Wesley; Schein, E. H. (1990). Career Anchors (Rev. Ed.). San Diego: Pfeiffer, Inc.)

If a psychologist begins treatment with a client knowing that there is a high probability that the records will be subpoenaed, the psychologist should: A not maintain any records in order to protect the client's best interests B maintain adequate records C maintain only information about the client which will not be damaging if disclosed D maintain records, but only include the minimal requirements in adherence with the law

The Correct Answer is "B" According to APA's Record Keeping Guidelines (1993, 48(9), 984-986) records should minimally include: identifying data, dates and types of service, fees, any release of information obtained, any assessment, plan for intervention, consultation, summary reports, and/or testing reports. It would, therefore, be unethical (and possibly illegal), not to maintain any records (A). Although records should be written with an awareness of the risk of the client or others gaining access to them, it would likely be incomplete and, therefore, unethical to include only information that will not be damaging (C). Finally, patient records should meet the minimal requirements of federal, state, and local laws; however, Choice D indicates that the psychologist should only meet those minimum requirements. There may be jurisdictions in which the Ethics Code or guidelines have a higher standard than the law; in which case, the psychologist should adhere to the higher standard. Thus, of the options offered, Choice B is the best choice.

Which of the following would most indicate malingering (i.e., an attempt to "fake bad") on the MMPI-2? A high K score B high F score C low F score D high L score

The Correct Answer is "B" An elevated F score, particularly within the range of 80 to 99 suggests malingering ("faking bad"), exaggeration of difficulties, resistance to testing, or significant psychopathology. All of the other choices, i.e., high K, high L, and low F scores indicate the reverse, that is, an attempt to be viewed in an overly favorable light ("faking good"). (G. Groth-Marnat, Handbook of psychological assessment, 2nd ed. New York, John Wiley, 1990).

Anticholinergic effects include all of the following EXCEPT: A constipation B diarrhea C blurred vision D dizziness

The Correct Answer is "B" Anticholinergic (side) effects occur as a result of interference with acetylcholine in the brain and peripheral nervous system. Tricyclic antidepressants and antipsychotics often have anticholinergic effects which include dry mouth, blurred vision, constipation, dizziness, urinary retention, tachycardia, and confusion

. While Sleep Terror Disorder and Sleepwalking Disorder are similar in terms of sleep and EEG patterns, there are behavioral differences that distinguish them. Sleepwalking Disorder, in contrast to Sleep Terror Disorder,: A is accompanied by high levels of autonomic arousal during the episode B is associated with prominent, organized motor activity during the episode C is not associated with amnesia for the episode when the individual awakens in the morning D is not associated with a family history of Sleepwalking and/or Sleep Terror Disorder

The Correct Answer is "B" B. Both Sleep Terror Disorder and Sleepwalking Disorder have been linked to a family history (d.) and are associated with amnesia for the episode upon awaking in the morning (c). Sleepwalking Disorder is usually associated with low levels of autonomic arousal (a.) and is characterized by prominent, organized motor activity such as walking around, talking, and eating. Both disorders do involve motor activity, although the activity associated with Sleep Terror Disorder is less organized and usually involves resisting being touched or held and sitting up.

Which one of the following is least likely to attenuate a measure of correlation? A restricted range B homoscedasticity C curvilinear relationship D the use of unreliable measures

The Correct Answer is "B" Homoscedasticity refers to even scatter around the regression line. Homoscedasticity is actually a good thing. It wouldn't attenuate the correlation at all. The other three choices list factors that would attenuate the correlation coefficient.

A researcher inquires about the subjects' performance expectations and beliefs about the purpose of the study at the conclusion of the experiment. The researcher finds the subjects' actual performance is consistent with their beliefs and expectations when analyzing the data. The results of the study may be confounded by: A the Hawthorne effect B demand characteristics C carryover effects D changing criteria

The Correct Answer is "B" B. Demand characteristics are unintentional cues in the experimental environment or manipulation that affect or account for the results of the study. In this situation, the subjects' may have acted in ways consistent with their expectations rather than simply in response to the experimental manipulation. The Hawthorne effect (a.) occurs when research subjects act differently because of the novelty of the situation and the special attention they receive as research participants. Carryover effects (c.) occur in repeated measures designs when the effects of one treatment have an impact on the effects of subsequent treatments.

The probability of a parent with Huntington's Chorea passing the disorder on to an offspring is: A 25% in female or male offspring B 50% in female or male offspring C increased if the offspring is female D increased if the offspring is male

The Correct Answer is "B" B. Huntington's Chorea is a dominant, autosomal disorder. Individuals with autosomal dominant diseases have a 50% chance of passing the mutant gene, and therefore the disorder, onto each of their children. Most dominant traits are due to genes located on the autosomes (the non-sex chromosomes). An autosomal dominant trait typically affects males (d.) and females (c.) with equal likelihood and similar severity. Although Huntington's usually does not manifest until mid-life, research indicates some variability, with the most severe cases showing up as early as two years of age, and on the other hand, some individuals remain symptomless until age 60. Research shows that the symptoms generally emerge earlier and become worse with each passing generation. Diagnosed by genetic testing, there is presently no cure although medication is used to control affective and motor symptoms

A score of 85 on the Lie (L) Scale of the MMPI-2 may indicate: A most Lie (L) Scale items were answered as "true" B a lack of insight or denial C similar or identical items were answered in an inconsistent way D acute psychological distress was experienced while taking the test

The Correct Answer is "B" B. The Lie Scale is one of the original MMPI validity scales, which were designed to evaluate test-taking attitudes. A high Lie Scale score may indicate a lack of insight into one's own behavior, denial, an attempt to create a favorable impression, or the tendency to answer items "false." A low Lie Scale score may be the result of answering items as "true" (a.). Inconsistent responding (c.) is suggested by a high score on the VRIN scale. Acute psychological distress (d.) is associated with an extremely low score on the K (Correction) Scale.

The mesencephalon does not contain which of the following? A substantia nigra B hypothalamus C inferior colliculus D superior colliculus

The Correct Answer is "B" B. The area of the brain referred to as the "midbrain" is also known as the mesencephalon and is divided into two regions: the tectum and the tegmentum. The substantia nigra (a.) is located in the tegmentum and is part of the brain's sensorimotor system. The inferior colliculus (c.) is part of the tectum and is involved in audition. The superior colliculus (d.) is also part of the tectum and mediates vision. The hypothalamus is located in the diencephalon. The diencephalon and the telencephalon make up the forebrain

A person stops and asks a parking attendant for directions after getting lost while driving to an appointment. The attendant states, "Make a left at the first red light. Go four blocks until you reach a stop sign and turn right onto Main Street. About three blocks down Main, look for a large red sign at the entrance to the parking lot." What type of memory is needed to hold such information? A procedural memory B working memory C short term memory D long term memory

The Correct Answer is "B" B. The type of memory needed to hold the directions in one's mind while working on it is called working memory. In the 1980s, Baddeley and Hitch coined the term "working memory" for the ability to hold several facts or thoughts in memory temporarily while solving a problem or performing a task. Baddeley defined the process of rapid verbal repetition of the to-be-remembered information to facilitate maintaining it in working memory, as an "articulatory loop." He found there is a neural system or central executive in the frontal portion of the brain responsible for processing information in the "working memory." Short-term memory (c.) holds information in mind for only a few seconds as it is processed. Long-term memory (d.) is where such processed information is permanently stored. Working memory is an intermediary and active memory system in the information processing area of the brain. (See: Baddeley, A.D., & Hitch, G.J. (1974). Working memory. In Bower, G.A. (Ed.) Recent advances in learning and morivation (Vol. 8). pp. 47-90. New York: Academic Press. See: Baddeley, A.D. (1986). Working Memory. Oxford: Oxford University Press.)

The behavior therapy technique for reducing arousal to inappropriate stimuli by first masturbating to orgasm while imagining appropriate stimuli and then continuing to masturbate while fantasizing about paraphilic images after orgasm is called: A Orgasmic Reconditioning B Satiation Therapy C Systematic Desensitization D Aversive Conditioning

The Correct Answer is "B" B. When treating individuals with paraphilias, behavior therapies are used based on the reasoning that maladaptive behavior has been learned and can therefore be unlearned. Orgasmic reconditioning (a.) instructs a client to begin masturbating while fantasizing about the inappropriate stimulus then switch from the paraphilic to more appropriate fantasies at the moment of masturbatory orgasm. Systematic desensitization (c.) pairs slow, systematic exposure to anxiety-inducing situations with relaxation training. Aversive conditioning (d.) substitutes a negative response for a positive response to inappropriate stimuli such as through pairing paraphilic urges with negative experiences, for example electric shocks or unpleasant odors.

Head trauma causing damage to the temporal lobes is most likely to produce deficits in A procedural memory. B episodic memory. C semantic memory. D implicit memory

The Correct Answer is "B" Damage to the temporal lobe produces deficits in declarative memory with episodic memory being affected more than semantic memory. Procedural memory is left relatively undisturbed as is implicit memory.

Alloy, Abramson, and Metalsky have revised the learned helplessness model of depression and de-emphasized the role of A punishment. B reinforcement. C helplessness. D attributions

The Correct Answer is "D" Alloy, Abramson and Metalsky (1989) theorize that attributions are only important when they contribute to feelings of hopelessness. They expand upon this idea in their article, Hopelessness depression: A theory-based subtype of depression, Psychological Review, 1989, 96(2), 358-372

A young woman is very anxious because, lately, she finds that she frequently misperceives things in the environment, which has caused some embarrassing moments at work and elsewhere. For example, at work, she mistook the coat rack for the company president. As a result, she is not enjoying her job or usual social activities as much as she used to. The woman's symptoms are most suggestive of A hallucinations. B illusions. C agoraphobia. D social phobia

The Correct Answer is "B" Note that the woman is misperceiving actual stimuli and that her other symptoms are directly related to this misperception. A misperception of a real stimulus is referred to as an illusion.

Of the following relationships, the one which, according to empirical research, is stronger is: A job enlargement and job satisfaction. B job turnover and job dissatisfaction. C gender and job dissatisfaction. D job satisfaction and productivity

The Correct Answer is "B" Of the four choices, only the relationship between job dissatisfaction and job turnover has been found to be significant

An adult woman comes to an outpatient clinic. She is foreign-born and has almost no skills in English. She appears depressed. If one were to attempt a standardized assessment of her level of cognitive functioning, the best test would be A Stanford-Binet. B Ravens Progressive Matrices. C Perdue Peg Board Test. D Otis-Lennon Test.

The Correct Answer is "B" Of these, only the Ravens is an intelligence test completely non-dependent on language. It's a non-verbal assessment of abstract reasoning and a fairly good predictor of general cognitive functioning

Which of the following is most associated with unusual perceptual experiences? A Generalized Anxiety Disorder B Panic Disorder C Adjustment Disorder D Dysthymic Disorder

The Correct Answer is "B" Panic attacks are characterized by at least four of ten symptoms. These may include derealization or depersonalization, which are types of perceptual disturbances

Positive and negative life events are likely to have which of the following types of effect on a person's sense of satisfaction and well-being? A neither short-term nor long-term effects B short-term but not long-term effects C long-term but not short-term effects D both short-term and long-term effects

The Correct Answer is "B" Research suggests that positive and negative life events can create great joy or distress but don't really affect a person's long-term sense of satisfaction and well-being. Only recent life-events seem to influence a person's well-being, and this effect drops off in about three months. For example, lottery winners have not been found to be significantly happier than non-winners

The levels-of-processing model of memory includes three levels--acoustic, phonetic, and semantic. All three of these levels have been compared to self-referencing, and the findings indicate A semantic is more easily recalled than the others. B self-referencing is more easily recalled than the others. C phonetic is most easily recalled. D semantic is the most difficult to recall

The Correct Answer is "B" Rogers and associates (Self-reference and the encoding of personal information, Journal of Personality and Social Psychology, 1977, 35, 677-688) found that self-referencing (Have I done this before? Where was I?) results in significantly more recall than semantic referencing. Previous to this finding, semantic processing (answer A) had been considered to be the most effective type of information processing.

A therapist who assigns a couple the task of looking for things in their relationship that they would like to continue is most likely using which of the following therapeutic approaches? A behavioral B solution-focused C structural D strategic

The Correct Answer is "B" Solution-focused therapy uses a variety of techniques including formula tasks that help clients become aware of solutions they already have in place. For example, using the formula first session task the therapist asks the clients to observe, and describe in the next session, what happens in their lives that they would like to continue to have happen.

A psychologist is hired as a consultant by an agency which works with homosexual men whose behavior places them at very high risk for the HIV virus. The psychologist is asked to work with groups of the agency's clients, with the goal of decreasing high-risk activity in this population. Assuming the psychologist is familiar with the research in this area, she is most likely to take which of the following approaches? A attempting to engender a group norm of disapproval for high-risk activity B providing knowledge to the group about AIDS C threatening the group with punishment if they don't change their behavior D taking a laissez-faire approach and letting the group learn on its own how dangerous its behavior is

The Correct Answer is "B" Studies show that, among individuals who are at high-risk for the AIDS virus, knowledge about AIDS is a better predictor of less risk-taking behavior than perceived peer norms. Thus, choice B is the best answer. By contrast, among low-risk groups, perceived peer norms are a better predictor. So if this question was about the best strategy for low-risk groups, choice A would have been a better answer.

*The cingulate cortex is most associated with: A spatial memory B emotions and motivation C somatosensory processing D balance and posture

The Correct Answer is "B" The cingulate cortex is part of the limbic system and is believed to play an excitatory role in emotions and in motivating behaviors. It's also known as the satisfaction center - mediating feelings of satisfaction following eating and sex.

In computing test reliability, to control for practice effects one would use a(n): I. split-half reliability coefficient. II. alternative forms reliability coefficient. III. test-retest reliability coefficient. A I and III only B I and II only C II and III only D II only

The Correct Answer is "B" The clue here is the practice effect. That means that if you give a test, just taking it will give the person practice so that next time, he or she is not a naive person. To control for that, we want to eliminate the situation where the person is administered the same test again. So we do not use test-retest. We can use the two other methods listed. We can use split-half since, here, only one administration is used (the two parts are thought of as two different tests). And, in the alternative forms method, a different test is given the second time, controlling for the effects of taking the same test twice

Thorndike's notion of "identical elements" is most applicable to A attentional capacity. B transfer-of-training. C elaboration. D reinforcement effects

The Correct Answer is "B" The name Thorndike may have sent you in the wrong direction on this one since you're probably most familiar with his work on the "law of effect." However, he also studied other phenomena including factors that increase transfer of training. He found that similarities in the learning and performance environments ("identical elements") resulted in better transfer.

Job commitment has the highest correlation with A job satisfaction. B absenteeism and turnover. C productivity. D quality of work

The Correct Answer is "B" The research on job commitment, or the degree to which a person identifies with an organization and is willing to work to help the organization reach its goals, suggests that it has a moderate to strong negative correlation with absenteeism and turnover. In this respect, it is similar to job satisfaction, which also is more highly correlated with absenteeism and turnover than with other variables (such as productivity) typically studied in the organizational psychology literature

There is some evidence that children of divorced parents have more academic problems than children from intact families and that A girls of divorced families exhibit poorer academic performance than boys of divorced families. B boys of divorced families exhibit poorer academic performance than girls of divorced families. C the negative effects of divorce on academic performance are not moderated by gender. D the negative effects of divorce on academic performance are observed in younger (but not older) children.

The Correct Answer is "B" The studies have consistently shown that children of divorce do more poorly in school than children of intact families. In addition, these negative effects tend to be more pronounced for boys than for girls and for children who are older at the time of the divorce

The phenomenon of "theme interference" in an industrial/organizational setting is most analogous to which of the following phenomena in psychotherapy? A cognitive restructuring B transference C resistance D catharsis

The Correct Answer is "B" The term "theme interference" is associated with Gerald Caplan's model of mental health consultation. According to Caplan, theme interference occurs when a worker displaces past or present personal problems onto a task situation at work. More precisely, a theme is a continuing cognitive representation of an unresolved problem or a defeat, and theme interference occurs when the person views a particular work situation in terms of a theme. As you can see, the phenomenon is analogous to transference in psychotherapy, as both involve displacement of past feelings or unresolved conflicts onto a present person or situation. In fact, Caplan refers to theme interference as a minor transference reaction of a special type

In their often-cited research study, Sue and his colleagues (1991) found that ethnic matching of therapist and client: A improves therapy outcome and reduces premature termination for African-American clients but not for Asian-American or Mexican-American clients. B improves therapy outcome and reduces premature termination for Asian-American and Mexican-American clients but has less of an effect on these variables for African-American clients. C substantially improves therapy outcome and reduces premature termination for Asian-Americans, Mexican-Americans, and African-Americans. D is related to premature termination for African-Americans and Whites but has little or no impact on other measures of treatment outcome.

The Correct Answer is "B" This is a difficult question because the research is so inconsistent on this issue. However, this question is referring to a specific study, which makes it easier as long as you're familiar with that study's results. One of the things that Sue and his colleagues learned was that the effects of ethnic matching differ for different minority groups. Specifically, ethnic matching seems most beneficial for Asian- and Hispanic-Americans, so this response best summarizes their findings.

An older woman has had a gradual but consistent onset of poor self-care, apathy and slight deficiencies in the WAIS-III processing and attention scales. She has a normal MRI. One might suspect: A Vascular dementia B Pseudodementia C Normal aging D Malingering

The Correct Answer is "B" This is a question where one has to "rule out" the other choices. With vascular dementia, the onset is "patchy". Poor self-care and apathy "should not be considered part of normal aging. There is nothing in the question to indicate malingering." It is very possible that this person is depressed and that this is being overlooked due to her age, thus the suggestion of "pseudodementia".

You are hired as a consultant by a large corporation. You evaluate the company and come up with a list of interventions that you believe would increase the company's profitability. To encourage employees to accept these changes, you assemble them in a large meeting hall and give a speech explaining how your proposed changes will benefit the company and, by extension, the employees. Which of the following strategies for increasing acceptance to change are you using? A normative-reeducative B empirical-rational C power-coercive D advocacy consultation

The Correct Answer is "B" This question refers to a framework that identifies three general strategies to bring about change in an organization: empirical-rational, power-coercive, and normative-reeducative. Empirical-rational strategies are based on the assumption that people are basically rational and, if they have all relevant information about a situation, will act in accord with their self-interest. Thus, a proposed change will be adopted if the proposer can rationally justify the change and show that it will benefit the members of the organization. The consultant in this question is explaining how the proposed changes will advance the self-interest of the employees; thus, he is adopting an empirical-rational approach

After reading a best-selling book on parenting, a mother decides to stop using all forms of punishment on her child. By following her self-imposed rule, when her child misbehaves the mother could only continue to: A send the child to time-out, which previously resulted in an improvement in the child's behavior B spank the child, which previously led to additional misbehavior C yell "STOP!," which caused the child to discontinue misbehaving for a short period of time D gently explain to the child why his behaviors are inappropriate, which previously resulted in an improvement in the child's behavior

The Correct Answer is "B" This question requires you to have a clear understanding of the definitions of reinforcement and punishment. "Reinforcement" always increases the frequency or likelihood of the targeted response. "Punishment" always decreases the frequency or likelihood of the targeted response. "Positive" means that a stimulus is applied, and "negative" means that a stimulus is removed. Choice A is an example of negative punishment because time-out involves the removal of normal activities and because it previously decreased the misbehavior. Choice C involved the application of a stimulus (yelling "Stop!"); thus it is positive, and it resulted in a decreased frequency of the misbehavior, and therefore, it is also a form of punishment. Choice D involves the application of attention; thus it is positive and because it reduces the frequency of the misbehavior, it would also be a form of punishment. Ironically, only Choice B would be permitted by the mother's self-imposed rule because the spanking has resulted in further misbehavior; thus, it would be considered a reinforcement procedure. It would also be positive, because it involves the application of a stimulus.

Which of the following statements is true about the results of meta-analyses investigating the effectiveness of cognitive therapy in the treatment of depression? A Cognitive therapy is more effective than insight-oriented and interpersonal therapy, but less effective than anti-depressant medication. B Cognitive therapy is more effective than insight-oriented therapy but equal in effectiveness to interpersonal therapy and anti-depressant medication. C Cognitive therapy is more effective than anti-depressant medication and insight-oriented therapy, but equal in effectiveness to interpersonal therapy. D Cognitive therapy is more effective than insight-oriented therapy, interpersonal therapy, and anti-depressant medication

The Correct Answer is "B" Though some individual studies have found cognitive therapy to be the most effective treatment for depression, the results of meta-analyses (such as the one conducted by the National Institute of Mental Health) indicate that cognitive therapy, interpersonal therapy, and anti-depressant medications are about equal in efficacy in the treatment of depression. All three treatments are superior to psychodynamic, or insight-oriented, therapies.

In the last session, your gorgeous new client complimented you on how nice you look in your red outfit. You noticed you were very happy before this session-you even caught yourself singing, "Oh What a Beautiful Morning" while looking in the mirror to do a final 'visual check" before seeing your client in. Ethically, A You need to cancel this session B You need to consult with a colleague C You need to behave more appropriately D You need to get out more

The Correct Answer is "B" While no Ethical Standards have been violated at this point; there is a strong indication that you are experiencing some type of "extra attraction" toward this client. You need to seek consultation and determine how to process and handle your countertransference. Cancelling the session may not be in the client's best interest

You are a small town psychologist in private practice and a client has written a letter to the Psychology Board complaining that you have a very unprofessional staff. The Board has now written you to inform you of this matter. You should: A Discipline your staff B Respond to the Board C Call the client and let them know that you will make the appropriate changes with your staff D Refer the client to a psychologist in another town

The Correct Answer is "B" You must respond to the Board. To not do so is itself an ethical violation. While it is possible that this is a clinical issue between yourself and your client it is no longer in the therapeutic realm. Your client has gone to an outside source and that source is concerned enough to begin a preliminary investigation

R2 is the: A coefficient of stability B coefficient of multiple determination C coefficient of internal consistency D silver character on "Star Wars"

The Correct Answer is "B" You probably had to take a guess on this one. Nevertheless, you now know that R2 is known as the "coefficient of multiple determination." It is a correlation coefficient like the Pearson r. However, uppercase "R" is a multiple correlation coefficient, which is used when there are multiple predictors. Like the Pearson r, the multiple correlation coefficient can be squared (R2), which indicates the percent of variance in the criterion explained collectively by all of the predictors.

Studies examining the predictions of goal-setting theory for work groups suggests that A having employees work as a team with team goals results in lower productivity than having employees work as individuals with individual goals. B having employees work as a team with team goals results in higher productivity than having employees work as individuals with individual goals. C having employees work as a team with team goals results in similar levels of productivity as having employees work as individuals with individual goals, as long as the goals have been consciously accepted by the employees. D having employees work as a team with team goals results in similar levels of productivity as having employees work as individuals with individual goals, as long as the goals have been determined by the employees.

The Correct Answer is "B" A current emphasis in the I/O literature is on team work, and the increased reliance on teams in organizations is supported by research showing that teams are more productive than individuals. Note that, in some situations, group performance is not as good as individual performance (e.g., in certain types of decision-making). However, this question is asking specifically about research related to goal-setting theory, so this is the best response. See A. J. DuBrin, Fundamentals of Organizational Behavior: An Applied Approach, Cincinnati, OH, South-Western College Publishing, 1997

In more severe cases of depression, Beck's cognitive therapy initially focuses on: A the individual's cognitive distortions. B appropriate behavioral tasks. C using free associations to identify automatic thoughts. D Socratic questioning.

The Correct Answer is "B" All of the techniques listed (except free association) are characteristic of Beck's Cognitive Therapy. However, when a client has severe depression, the first goal in therapy is to increase his or her activity level. This is accomplished through the use of behavioral tasks. Once the client is active, cognitive procedures are used to identify and modify cognitive distortions.

The kappa statistic is used to evaluate reliability when data are: A interval or ratio (continuous) B nominal or ordinal (discontinuous) C metric D nonlinear

The Correct Answer is "B" B. The kappa statistic is used to evaluate inter-rater reliability, or the consistency of ratings assigned by two raters, when data are nominal or ordinal. Interval and ratio data is sometimes referred to by the term metric

You are concerned that the validity coefficient of a new selection test might be significantly lower for females than for males. Specifically, your concern is that use of the test might lead to: A unfairness. B adverse impact. C statistical regression. D criterion contamination.

The Correct Answer is "B" The situation described in this question is referred to as differential validity, which can be a cause of adverse impact. (Unfairness is another cause of adverse impact but is different from differential validity.)

Which of the following is most true of Type I Schizophrenia as described by Timothy Crow? A There will be a poor prognosis. B The symptoms are due to a neurotransmitter abnormality. C A majority of symptoms will begin in adolescence. D It will not respond to most antipsychotic medications

The Correct Answer is "B" Timothy Crow distinguishes between two types (I and II) of Schizophrenia. Type I includes symptoms of delusions or hallucinations, inappropriate affect, and disorganized thinking. It is also thought to more likely be due to neurotransmitter irregularities. Type II symptoms are described by the other three choices, and is also considered to be more likely due to brain structure abnormalities (T.J. Crow, The two syndrome concept: Origins and current status, Schizophrenia Bulletin, 1992, 11, 471-486).

Therapists working with elderly patients should be aware that A elderly patients have very poor memory. B elderly patients cannot benefit much from psychotherapy. C individual differences among elderly patients are greater than individual differences among younger patients. D elderly people tend to drop out of therapy prematurely

The Correct Answer is "C" A number of authors have noted that elderly individuals tend to vary greatly -- even more so than younger people -- in terms of physical abilities, intellectual abilities, interests, and personality characteristics. The other choices represent myths or stereotypes about the elderly that should be avoided

When conducting a custody evaluation, a psychologist should: A advocate for the party that retained him B advocate for the party that retained him, unless retained by the court, in which case, he should remain impartial C remain impartial regardless of whether he is retained by the court or either party in the proceedings D acknowledge the fallacy of impartiality in an adversarial legal system

The Correct Answer is "C" According to "Guidelines for Child Custody Evaluations in Divorce Proceedings," "The psychologist should be impartial regardless of whether he or she is retained by the court or by a party to the proceedings. If either the psychologist or the client cannot accept this neutral role, the psychologist should consider withdrawing from the case. If not permitted to withdraw, in such circumstances, the psychologist acknowledges past roles and other factors that could affect impartiality (American Psychologist, 49(7), 1994, 677-680).

. You are the only provider in a rural community and your spouse's coworker asks if he could schedule an appointment with you. Which of the following is the best course of action? A You must refer him. B You must schedule an appointment with him. C You should refer him if you believe that treating him would impair your objectivity. D You should discuss this with your spouse and this coworker to determine if this constitutes a dual relationship.

The Correct Answer is "C" According to APA's Ethics Code regarding multiple relationships (3.05[a]), "A psychologist refrains from entering into a multiple relationship if the multiple relationship could reasonably be expected to impair the psychologist's objectivity, competence, or effectiveness in performing his or her functions as a psychologist, or otherwise risks exploitation or harm to the person with whom the professional relationship exists." Choice A is too prohibitive. Although it may be necessary to refer this person, if it appears unlikely that entering into a professional relationship would impair your objectivity or would otherwise be harmful to the person, you could treat him. Choice B can be eliminated because psychologists and other health professionals have no legal or ethical obligation to enter into a professional relationship. And Choice D can be eliminated because discussing the matter with your spouse would be a breach of confidentiality.

For children with Enuresis (Not Due to a General Medical Condition), the most likely co-diagnosis is: A Major Depression B Separation Anxiety Disorder C Sleepwalking or Sleep Terror Disorder D Specific or Social Phobia

The Correct Answer is "C" According to DSM-IV-TR, although the prevalence rates of some disorders are higher among children with Enuresis, most do not have a co-existing disorder. DSM lists Encopresis, Sleepwalking Disorder, and Sleep Terror Disorder as other disorders that are most likely to be present. (Note that Sleepwalking and Sleep Terror Disorders are the only disorders given that occur during sleep, which is also usually the case with Enuresis. Noticing this would have helped you pick the correct answer.)

. All of the following statements are consistent with Minuchin's concept of enmeshed or disengaged families, except: A in an enmeshed family, parents may devote an excessive amount of attention to one symptomatic family member who serves as the "symptom bearer" for the entire family. B enmeshed parents tend to be loving and considerate and do a lot for their children. C in a disengaged family, excessive conflict prevents effective communication. D in a disengaged family, parents may not notice when their children need support or guidance.

The Correct Answer is "C" According to Minuchin, boundaries in a family system can be described as enmeshed or disengaged; the two terms represent opposite ends of a continuum. Enmeshed boundaries are overly diffuse; the result is a family in which members are overly close and dependent. Disengagement describes boundaries that are overly rigid; members of a disengaged family tend to be isolated from each other. Contrary to choice C, members of disengaged families tend to avoid conflict. The other choices are true of enmeshed or disengaged families.

A college graduate is hired by a small firm and is promised bonuses and promotions based on his work performance. After working at the firm for two years, he has not received a bonus, raise, or promotion, despite the fact that he has regularly worked overtime and is generally recognized to be one of the firm's best workers by the other employees. From the perspective of expectancy theory, which of the following would be lowered in the man as a result of this experience? A valence B expectancy C instrumentality D idealism

The Correct Answer is "C" According to expectancy theory, motivation is a function of expectancy, valence, and instrumentality. Instrumentality refers to the degree to which performance leads to certain desired outcomes. In this situation, the man's performance is not leading to promotions and bonuses; as a result, according to the theory, instrumentality and, as a result, motivation, would be lowered. Valence refers to the importance of outcomes that follow from performance. In this situation, valence would be related to how much the man values promotions and bonuses. And expectancy refers to belief that effort will pay off in performance. In this situation, expectancy refers to the man's belief that working hard would lead to good performance on the job.

Alzheimer's is related to an under production of A serotonin B GABA C Acetylcholine D Glutamate

The Correct Answer is "C" Acetylcholine plays a role in learning and memory. Loss of acetylcholine receptors in the cortex and hippocampus occur in Alzheimer's disease

Your client arrives at the fourth session with a small handgun in his coat. He says that when he leaves the session he is going to go by his ex-wife's house and "shake up her new husband a bit." He assures you that the gun is not loaded. You should A insist he remain in your office and try to calm him down. B ask him to turn over the gun. C alert the new husband if possible. D do nothing; this particular client has an odd sense of humor.

The Correct Answer is "C" Answer C is referring to a situation in which the Tarasoff law applies. You have a client who is threatening harm to an intended victim. "The discharge of such duty...may call for the therapist to warn the intended victim of the danger..." (Tarasoff vs. Regents of University of California 1976, p. 426). Choice A and B are heroic, but may place you at considerable risk; they also require clinical judgments you cannot make with the limited inform ation available in this question

The onset of sexual maturation in boys under 9 years old or girls under 7 years old is related to activation of the: A adrenal cortex B thyroid gland C hypothalamic-pituitary axis D mesocorticolimbic-dopaminergic axis

The Correct Answer is "C" Both normal and premature sexual maturation is due to activation of the hypothalamic-pituitary-gonadal axis (also known simply as the hypothalamic-pituitary axis). However, this system is activated prematurely in precocious puberty. Precocious puberty is defined as any sign of secondary sexual characteristics in boys younger than 9 years old, or in girls younger than 7 years old. Precocious puberty is estimated to occur in 1 in 5,000 to 1 in 10,000 children. The adrenal cortex (A) is mostly associated with the stress hormone cortisol, which converts energy stores into glucose for fuel. The thyroid gland (B) is related to regulation of overall metabolic rate. And the mesocorticolimbic-dopaminergic axis is considered the brain's reward pathway, which is critical in the reinforcing effects of drug abuse

With respect to productivity, cohesive groups are associated with: A low levels of productivity for the group B high levels of productivity for the group C higher levels of productivity for the group only when management is supportive D higher levels of productivity for the group only when management is hostile or indifferent

The Correct Answer is "C" C. High group cohesiveness has positive and negative consequences. In terms of productivity, the relationship between group cohesiveness and group productivity is affected by several moderator variables. One of which is management supportiveness. High management support is associated with high levels of productivity for cohesive groups, while management hostility is associated with low productivity for cohesive groups. (See: A. Tziner and Y. Vardi, Effects of command style and group cohesiveness on the performance effectiveness of self-selected tank crews, Journal of Applied Psychology, 1982, 67, 769-775.)

Factitious Disorder treatment typically involves symptom management rather than curing the disorder. Which of the following is currently considered to be the most effective treatment? A confrontational therapy in an inpatient setting B individual and group therapy in an inpatient treatment C supportive psychotherapy in an outpatient setting D group or family therapy in an outpatient setting

The Correct Answer is "C" C. Studies indicate no specific treatment has been identified as consistently effective for Factitious Disorder; however, many agree that establishing a good therapeutic relationship and providing supportive therapy and consistency of care is the best way to manage its symptoms. (See: J. C. Huffman and T. A. Stern, The diagnosis and treatment of Munchausen's syndrome, General Hospital Psychiatry, 2003, 25(5), 358-363.) Family and group therapy (d.) have not been identified as the most effective treatments, although family therapy may be useful for helping family members cope with the patient's symptoms. Inpatient treatment (a.) is often contraindicated because of the underlying need of individuals with Factitious Disorder "to adopt the sick role." Confrontational techniques (b.) may elicits denial and result in the individual terminating the therapeutic relationship so they must be used with caution

Which of the following is a measure of "amount of variability accounted for" A alpha B Cohen's d C eta squared D F-ratio

The Correct Answer is "C" C. The "amount of variability accounted for" is assessed by a squared correlation coefficient. Eta squared is the square of the correlation coefficient (i.e., the correlation between the treatment and the outcome) and is used as an index of effect size. Alpha (a.) is the level of significance set by a researcher prior to analyzing the data. Cohen's d (b.) is used as an index of effect size, but it is a measure of the mean difference between two groups. The F-ratio (d.) is the statistic calculated when using the analysis of variance.

According to the DSM-IV-TR, Bipolar I Disorder is differentiated from Bipolar II Disorder by: A at least one Major Depressive Episode and one Mixed Episode without a Manic or Hypomanic Episode B at least one Major Depressive Episode and one Hypomanic Episode without a Manic or Mixed Episode C one or more Manic or Mixed Episodes D one Mixed Episode or Hypomanic Episode

The Correct Answer is "C" C. The DSM diagnosis for Bipolar II Disorder requires the history or presence of Major Depressive and Hypomanic Episodes (b.) in the absence of Manic or Mixed Episodes (a., d.). If an individual diagnosed with Bipolar II Disorder develops a Manic or Mixed Episode then the diagnosis is changed to Bipolar I Disorder

The predictor cut-off score of a selection test is lowered. How will this impact successful hiring? A increase the probability of false negatives B decrease the probability of false positives C decrease the probability of true positives D increase the probability of true positives

The Correct Answer is "C" C. The term "positive" refers to individuals selected or hired and "negative" to those who are not. "True" refers to a correct decision and "false" refers to an incorrect decision. When a predictor cutoff is lowered, more people are selected overall, or there are more positives. Since the predictor criterion is lower there are more true and false positives. Therefore, the probability that those hired will be successful (true positives) decreases and the probability of unsuccessful hiring (false positives) increases.

The consultant's role in client-centered case consultation is to: A educate the consultee in methods for evaluating treatment outcomes for a particular type of diagnosis or client B advise the consultee on the assessment and treatment of a particular type of diagnosis or client C assess one of the consultee's clients and advise the consultee of the best treatment approach for the specific client D assess and treat a specific client of the consultee

The Correct Answer is "C" Client-centered case consultation focuses on a specific client and the consultant advises the consultee in regard to the best course of action in working with the client. The consultant gathers information about the client and then makes recommendations to the consultee regarding a treatment plan or course of action. The consultant may directly assess the client but does not administer treatment (response "D"). Responses "A" and "B"describe consultee-centered consultation, which focuses on the skills, knowledge, and ability of the consultee

Patterson's research on delinquency has shown that the parents of antisocial adolescents are most often A hostile and rejecting. B overindulgent. C lax and uninvolved. D overcontrolling

The Correct Answer is "C" Delinquency has been linked to a number of parental characteristics, especially lax supervision, nonenforcement of rules, and noninvolvement in the child's life

the rejection from consciousness of painful or shameful experiences: a)defense mech b)repression c)denial d) conflict

b) repression

When applied to substance dependence, the specifier "early full remission" means that the client has been without any symptoms of substance abuse or dependence: A for at least six months but less than one year B for at least three months but less than one year C for at least one month but less than one year D for at least one year

The Correct Answer is "C" Early full remission, early partial remission, sustained full remission, or sustained partial remission are specifiers used to describe a substance dependence diagnosis. "Early" means that there is more than one month but less than 12 months of remission. "Sustained" means that there is 12 months or longer of remission (response "D"). "Full" means that the person no longer meets any of the criteria for substance dependence or abuse, and "partial" means that one or more of the criteria for substance dependence are still met but the full criteria are no longer met.

*Consuming foods containing tyramine while taking a monoamine oxidase inhibitor would most likely result in: A depression B hypotension C hypertension D no adverse reactions MAOI (change item seen earlier)

The Correct Answer is "C" Eating foods containing tyramine while taking MAOIs is likely to cause a hypertensive crisis, that is, a severe rise in blood pressure. The person may also experience headache, diaphoresis (perspiration), and palpitations. On rare occasions the combination has caused cardiac failure and cerebral hemorrhage. Some of the foods that must be avoided while taking MAOIs include: beer and wine, aged cheese, beef or chicken liver, orange pulp, pickled or smoked meats, packaged soups, and yeast vitamin supplements.

In order to get an infant's attention, a photographer shakes a rattle, which results in the infant turning toward the rattle and smiling. After numerous uses of this technique, the infant stops responding. This is most likely due to: A adaptation B extinction C habituation D satiation

The Correct Answer is "C" Habituation is defined as a decrease in response strength due to repeated stimulation. Satiation (D) occurs when a physiological need has been met, resulting in a decreased desire for that reinforcement. Extinction (B), in operant learning, involves eliminating reinforcement that previously followed a response. In classical conditioning, extinction refers to the presentation of a CS without the US. Neither of these two models adequately apply to this question

In order to get an infant's attention, a photographer shakes a rattle, which results in the infant turning toward the rattle and smiling. After numerous uses of this technique, the infant stops responding. This is most likely due to: A adaptation B extinction C habituation D satiation

The Correct Answer is "C" Habituation is defined as a decrease in response strength due to repeated stimulation. Satiation (D) occurs when a physiological need has been met, resulting in a decreased desire for that reinforcement. Extinction (B), in operant learning, involves eliminating reinforcement that previously followed a response. In classical conditioning, extinction refers to the presentation of a CS without the US. Neither of these two models adequately apply to this question.

Which of the following item difficulty levels maximizes discrimination among test-takers? A .10 B .25 C .50 D .90

The Correct Answer is "C" If a test item has an item difficulty level of .50, this means that 50% of examinees answered the item correctly. Therefore, items with this difficulty level are most useful for discriminating between "high scoring" and "low scoring" groups.

A psychologist gets a call on a Friday night from a current patient who says she's feeling so depressed she is really thinking of suicide. The psychologist believes her and advises her to go to the hospital immediately. The psychologist is concerned and calls the next day and finds out that the patient did commit suicide. The psychologist's intervention was A unethical, because he should have consulted a psychiatrist before he suggested hospitalization. B ethical, since he made a referral for appropriate treatment. C unethical, because he should have been more active in assuring that the patient's needs were met. D ethical, since he did all that he could on a weekend

The Correct Answer is "C" If the psychologist believed that the patient was actively suicidal, it was unrealistic to assume that she'd get herself to the hospital. The psychologist should have been more active; for example, he could have called the police, called someone close to the patient, or gone to the patient's location.

Overlearning would be most useful when you are trying to: A learn how to play the violin. B understand fractal geometry. C memorize the state capitals. D learn to solve complicated calculus problems

The Correct Answer is "C" In general, overlearning (practicing or rehearsing beyond the level of mastery) is good. However, it seems most effective for simple tasks that must be remembered for a long period of time

A quality assurance review would likely focus on all of the following except A treatment effectiveness. B access to treatment. C cost of treatment. D patient satisfaction

The Correct Answer is "C" In the health care business, quality assurance reviews are conducted by organizations such as HMOs or independent overseers to review the quality of services provided. Because the cost of services is not directly related to their quality, it would probably not be a focus of a quality assurance review.

All of the following statements are true of the technique of stress inoculation training, except: A its mechanism of action is analogous to that of medical inoculation. B it consists of three distinct phases. C it is based on the notion that immediate imaginal exposure to a fearful stimulus at its maximal intensity results in rapid extinction of the fear. D it relies heavily on the technique of self-instruction.

The Correct Answer is "C" Meichenbaum's stress inoculation training is based on the medical inoculation model: the idea is to give the person the opportunity to deal with a low "dose" of anxiety-provoking stimuli, in order to help the person build coping skills (which are analogous to immunity). Contrary to choice C, the technique consists of three steps (education, rehearsal, and real-life application) in which the person's exposure to stressful stimuli proceeds from least-anxiety to most anxiety-provoking. As part of the training, clients are taught to use appropriate self-statements (i.e., self-instruction) to guide them through the coping process

Recent research indicates the most effective treatment for antisocial behavior in juvenile offenders is: A behavioral-reinforcement B structured family intervention with parental training C multi-systemic therapy D victims family therapy

The Correct Answer is "C" Multisystemic Therapy (MST), targeting chronic, violent, or substance abusing juvenile offenders at high risk of out-of-home placement, is consistent with social-ecological models of behavior and findings from causal modeling studies of delinquency and drug use. The approach views individuals as being nested within a complex network of interconnected systems that encompass individual, family, and extrafamilial (peer, school, neighborhood) factors. MST is a goal-oriented, intensive family- and community-based treatment that addresses the multiple determinants and factors in each youth's social network that are contributing to his or her antisocial behavior. It is provided using a home-based model of services delivery with a typical treatment duration of approximately 4 months. Intervention strategies include strategic family therapy, structural family therapy, behavioral parent training, and cognitive behavior therapies. MST interventions typically aim to improve caregiver discipline practices, enhance family affective relations, decrease youth association with deviant peers, increase youth association with prosocial peers, improve youth school or vocational performance, engage youth in prosocial recreational outlets, and develop an indigenous support network of extended family, neighbors, and friends to help caregivers achieve and maintain such changes. MST has demonstrated long-term reductions in criminal activity, drug-related arrests, violent offenses, and incarceration. Controlled studies also showed that MST outcomes were similar for youths across the adolescent age range (i.e., 12-17 years), for males and females, and for African-American vs. white youths and families. (See: Curtis, N. M., Ronan, K. R., & Borduin, C. M. (2004). Multisystemic treatment: A meta-analysis of outcome studies. Journal of Family Psychology, 18, 411-419.)

If there are 25 people in the treated group (ie., N=25) and the standard deviation of their scores on the Beck Depression Inventory (BDI) is 10. What is the standard error of the mean? a) 2 b) 2.5 c) 250 d) 2500

a

All of the following are true regarding Feminist Object Relations Theory except: A it emphasizes the importance of the maternal relationship with the child. B it proposes that gender differences are the result of girls being taught to remain attached to their mothers while boys are taught to separate from their mothers. C it proposes that girls develop a healthier self-object when raised primarily by their father due to earlier separation from their mother. D it proposes that mothering "reproduces itself" since mothers perpetuate the oppressiveness of the division of labor

The Correct Answer is "C" Nancy Chodorow was one of the first to apply a feminist perspective to an Object Relations model. In The Reproduction of Mothering (Berkeley, University of California Press, 1978) she focused on the relationship between mother and child and how gender differences result from the mother being the primary caretaker. The division of labor which occurs in families (although less so in recent years), is characterized by women being more involved in affective, interpersonal relationships than men. This is inevitably passed on to both boys and girls who 'reproduce' this sexual and familial division of labor.

Your client comes to session and tells you that he is having trouble recognizing your face and those of others familiar to him. You realize that he may be suffering from prosopagnosia which is believed to be due to damage to the A central sulcus. B parieto-occipital sulcus. C bilateral occipitotemporal area. D optic chiasm

The Correct Answer is "C" Prosopagnosia involves deficits in both visual processing and memory. Since the temporal lobe mediates long term memory and the occipital lobe is involved in visual processing, you may have been able to figure out the correct answer to this question by putting this information together. The central sulcus (answer A) divides the parietal and frontal lobes. The parieto-occipital sulcus (answer B) separates the occipital lobes. The optic chasm (answer D) is the point at which the optic nerve from one eye partially crosses to join the other.

According to the current research, the effects on children of observing aggressive models, such as through television violence, are A short-term but not long-term. B long-term but not short-term. C both short-term and long-term. D neither short-term nor long-term

The Correct Answer is "C" Research on the effects of television violence on children's behavior demonstrates that viewing aggressive models is associated with increased aggressive behavior. Moreover, these effects can be long-lasting; they have been observed in longitudinal studies lasting up to 22 years.

Studies on the effectiveness of training for improving the cognitive skills of older people who have already experienced normal age-related declines have found that training A is useful for verbal but not for nonverbal skills. B is useful for nonverbal but not for verbal skills. C can improve verbal and nonverbal skills to levels comparable to pre-decline levels. D can improve verbal and nonverbal skills but to a level significantly below pre-decline levels

The Correct Answer is "C" Results of the Seattle Training Study not only demonstrated that behavioral interventions can remediate verbal and nonverbal skills (e.g., inductive reasoning, spatial orientation) in elderly adults but, for about 40% of participants, raise these skills to pre-decline levels.

According to the peripheralist theory of emotions, "we feel afraid because we tremble." This approach is reflected in which of the following theories? A Cannon-Bard B Lazarus-Schachter C James-Lange D Kuder-Richardson

The Correct Answer is "C" The James-Lange theory, one of the earliest theories of emotion, proposes that we feel after our body reacts (i.e., that we are sad because we cry).

The Racial Identity Attitude Scale, developed by Helms and Parham (1996), is used to assess the stages of African-American racial identity development which are: A conformity, dissonance, resistance and immersion, introspection, and articulation and awareness B separation, marginalization, assimilation, and integration C pre-encounter, encounter, immersion-emersion, and internalization D encounter, resistance, integration, and awareness

The Correct Answer is "C" The Racial Identity Attitude Scale, developed by Helms and Parham (1996), is based on Cross' (1971, 1978) stages of African-American identity development. The stages are pre-encounter, encounter, immersion-emersion, and internalization. Choice A includes the stages of Atkinson, Morten, and Sue's Minority Identity Development Model. Choice B refers to Berry's Acculturation model, which consists of independent dimensions, rather than stages. And choice D contains terms from each of the other mod

Which of the following brain structures could be described as a "satisfaction center"? A hippocampus B basal gangllia C cingulate gyrus D reticular activating system

The Correct Answer is "C" The cingulate gyrus acts as a "satisfaction center" and mediates the feelings of satisfaction that follow eating and sex. The hippocampus (choice A) is more associated with learning and memory consolidation. The basal ganglia (choice B) are important in the regulation of muscle tone and fine motor control, and the reticular activating system (answer D) is vital to consciousness and arousal

A colleague of yours is interested in studying the effects of aging on IQ scores. He consults with you for some ideas regarding how to proceed with this research. Which of the following types of research designs would you recommend? A longitudinal B cross-sectional C cross-sequential D multiple baseline

The Correct Answer is "C" The colleague is interested in conducting developmental research, in which the effects of development (e.g., aging) on a dependent variable (in this case, IQ scores) are investigated. Longitudinal, cross-sectional, and cross-sequential are all types of developmental research designs. Of these, cross-sequential research is the strongest from a scientific point of view. Cross-sequential research is a combination of cross-sectional and longitudinal research. In cross-sequential research, as in cross-sectional research, subjects are divided into age groups (e.g., young, middle-aged, and old). And, as in longitudinal research, subjects are assessed repeatedly on the dependent variable over time. Because cross-sequential research combines the methodology of the two strategies, it is not associated with the limitations of one or the other.

Mental imaging or visualization, in the absence of external visual stimulation, is believed to be most associated with the: A basal ganglia B frontal lobe C hippocampus D parietal lobe

The Correct Answer is "C" The medial temporal lobe (limbic system) contains the hippocampus, amygdala, and other structures which have recently been associated with the ability to generate mental images (G. Kreiman, C. Koch, & I. Fried, Category-specific visual responses of single neurons in the human medial temporal lobe. Nature Neuroscience, 2000, 3, 946-953).

In any organized setting of more than a few people, paired-comparisons is the preferred rating system when the goal is to A establish a standard performance criterion for the average productivity rate in that organization. B rank-order the people in the organization in the most cost-effective manner. C maximally differentiate among the people in the organization. D set the most attainable and acceptable production goals for each person in that organization

The Correct Answer is "C" The paired comparison technique involves comparing each person to every other person in the organization in reference to one or more variables such as productivity. It is a time-consuming and expensive process but results in highly precise and differentiated evaluations of each person in the group evaluated

According to meta-analytic studies of psychotherapy, A so few well-controlled studies have been done that no conclusions can be made concerning treatment outcome. B behavior therapy is better than psychoanalysis. C behavior therapy is the treatment of choice for phobias. D a person participating in therapy would have just as much chance of improving as a control subject

The Correct Answer is "C" The research on psychotherapy outcome is pretty consistent in its conclusions. Common findings include that therapy is generally effective for children and adults of all backgrounds and that, with the exception of certain specific problem areas, all therapies are about equal. This question is about one of those exceptions. According to outcome research, phobias respond best to behavioral techniques such as exposure treatments.

You have been retained by the court to make a recommendation concerning custody of a 4 year old girl. You have met with the mother on two occasions and also administered some psychological tests. You have completed a phone interview with the father who lives out of state. In reporting to the court, you should A explain that you cannot present any data. B recommend custody go to the parent you have determined is most competent to parent. C present the results of your evaluation but refrain from making a recommendation. D make a recommendation, but note that your evaluation of the father was incomplete.

The Correct Answer is "C" This is another question where it is tempting to do more than you are qualified to do. Answer C is correct; Standard 9.01(b) states that "psychologists provide opinions of the psychological characteristics of individuals only after they have conducted an examination of the individuals adequate to support their statements or conclusions. When, despite reasonable efforts, such an examination is not practical, psychologists document the efforts they made and the result of those efforts, clarify the probable impact of their limited information on the reliability and validity of their opinions, and appropriately limit the nature and extent of their conclusions or recommendations." A phone conversation is not a psychological evaluation. You may have chosen response D, but it is pertinent to ask yourself the question that if you could not make a complete evaluation why would you make a recommendation.

Who has the highest suicide rate? A people aged 24-44 B people aged 55-64 C people over 65 D teenagers

The Correct Answer is "C" While the highest rates of suicide attempts occur between the ages of 24-44, the highest rates of completed suicides are among the elderly (65 and over). The greatest increase in suicide rates in recent years has been among teenager and young adults

Peripheral vision is processed in the: A frontal lobe B temporal lobe C anterior occipital lobe D posterior occipital lobe

The Correct Answer is "C" You probably were able to narrow the choices down to C and D, knowing that vision is processed in the occipital lobe. However, you should also know that peripheral vision is processed in the anterior occipital lobe. Central vision is processed in the posterior occipital lobe.

A psychologist is working for a company that changes health care providers due to cost-effectiveness. The new provider has a policy that compromises the employee's future limits to confidentiality. The psychologist should A refuse to follow the new procedures and leave immediately. B outline her points of dissatisfaction and send it to the new provider. C make clear with each client the company's new limits to confidentiality. D organize a strike and call CNN

The Correct Answer is "C" According to Standard 3.11 (Psychological Services Delivered To or Through Organizations), "Psychologists delivering services to or through organizations provide information beforehand to clients and when appropriate those directly affected by the services about (1) the nature and objectives of the services, (2) the intended recipients, (3) which of the individuals are clients, (4) the relationship the psychologist will have with each person and the organization, (5) the probable uses of services provided and information obtained, (6) who will have access to the information, and (7) limits of confidentiality. As soon as feasible, they provide information about the results and conclusions of such services to appropriate persons." If you choose answer A and leave abruptly, you are abandoning your client. You may also want to do answer B—outline your points of dissatisfaction—but this wouldn't be your first or most important consideration

The findings from longitudinal research on divorce risk factors by John M. Gottman and his colleagues indicate: A suppression of negative affect is associated with a high risk of divorce B suppression of negative and positive affect is associated with a high risk of early divorce C criticism, defensiveness, contempt and stonewalling predicts a high risk of early divorce D criticism, defensiveness, contempt and stonewalling predicts a high risk of later divorce

The Correct Answer is "C" C. Longitudinal research by John M. Gottman and his colleagues investigated the interactions between married couples that predict divorce. The results indicated a combination of criticism, defensiveness, contempt and stonewalling, referred to as the "four horseman of the Apocalypse," is associated with a high risk for early divorce. This is considered to be the first seven years of marriage, which is when half of all divorces are known to occur. The suppression of affect, both positive and negative, is predictive of later divorce or the period during which the first child reaches 14 years of age. (See: Gottman, J.M. & Levenson, R.W. (2000). The Timing of Divorce: Predicting When a Couple Will Divorce Over a 14-Year Period. Journal of Marriage and the Family, 62, 737-745.)

If two or more subjects are treated as one group, what design is this considered to be? a) single subject design b) double subject design c) group subject design d) multiple subject design

a

Driver, Broussseau, and Hunsaker identified which of the following decision-making styles? A transformational and transactional B rational-economic and administrative C decisive, flexible, integrative, systemic, and hierarchic D autocratic, democratic, and laissez-faire

The Correct Answer is "C" Driver, Brousseau, and Hunsaker (1993) identified five types of decision-makers based on how much information is considered and how many alternative solutions are sought. The five types are: Decisive, Flexible, Hierarchic, Integrative, and Systemic. The Decisive is fast, efficient, and relies on a minimal amount of information (i.e., a satisficer) and a single solution. The Flexible decision-maker also moves fast and is a satisficer, but is willing to adapt and change solutions if indicated. Hierarchics rely on a lot of information (i.e., are maximizers) but stay rigidly focused on a single solution. The Integrative decision-maker relies on a lot of information (i.e., a maximizer) but pursues multiple solutions. Systemics keep their eyes on the "big picture," rely on maximum information (i.e., a maximizer), and develop a prioritized set of strategies for dealing with a situation, rather than a single solution or a collection of alternative plans. The other choices refer to other leadership styles and decision-making models which should be familiar to you. Transformational and transactional (A) are types of leaders. Transformational leaders raise the consciousness of their employees and transactional leaders focus on rules and maintaining the status quo. The administrative model (B) is a decision-making model proposed by Simon, which he contrasted to the rational-economic model. According to the administrative model, decision-makers can't afford to be rational and must be satisficers. Autocratic, democratic, and laissez-faire (D) refer to three styles of leadership proposed by Lewin, Lipitt, and White. Autocratic leaders make decisions alone, democratic leaders involve subordinates in decision-making, and laissez-faire leaders allow subordinates to make decisions on their own with little guidance.

One advantage of standard scores as compared to percentile ranks is that standard scores A allow you to determine the relative standing of examinees who take the same test. B set cutoff scores above which a given percentage of examinees will score. C provide more meaningful information about differences between examinees' test scores. D when used properly, can decrease the cultural bias of test scores in many cases.

The Correct Answer is "C" One disadvantage of percentile ranks is that a given distance between two percentile ranks does not necessarily reflect the same distance between the examinees' raw scores. Specifically, percentile ranks tend to overestimate raw score differences in the middle of the score distribution and underestimate raw score differences at the end of the distribution. Let's take an example: Say that Examinee A has a percentile rank score of 93, Examinee B has a percentile rank score of 96, Examinee C has a percentile rank score of 50, and Examinee D has a percentile rank score of 53. If you're just looking at percentile ranks, you might assume that the score difference between Examinee A and B is equivalent to the score difference between Examinee C and D. However, because examinees A and B scored at the extreme high end of the distribution, their raw score difference will be greater than that between examinees C and D, who scored in the middle of the distribution. Choices A and B are true of both standard scores and percentile ranks, while choice D is true of neither

Delinquency in adolescence is most associated with: A low income families and low IQ score. B a mother who works outside the home and an unemployed father. C weak parental supervision and parental rejection. D harsh physical punishment and unreasonable rules

The Correct Answer is "C" The research is far from consistent on this issue. However, lack of parental supervision is frequently found as a correlate of delinquency along with a hostile relationship between parent and adolescent and parental rejection. Low SES, low IQ score, single-parent homes, and harsh discipline have weaker relationships with delinquency.

A study is conducted to determine the effectiveness of 3 different reading programs on reading comprehension. The participants are 5th grade students who are divided into 3 levels based on their past reading comprehension (below average, average, and above average). Results from a factorial ANOVA indicate that there are significant main effects of each variable and a significant interaction effect. Based on these results, one could conclude that: A each of the reading programs is equally effective for students at every reading level B only one of the reading programs is effective for students at every reading level C the reading programs are only effective for students at a particular reading level D the most effective reading program is dependent on the student's reading level

The Correct Answer is "D" A factorial ANOVA is used when a study has more than one independent variable. Factorial designs also allow for the assessment of both main effects (the effects of each independent variable considered individually) and interaction effects (the effects of each variable at the different levels of the other variable). The study described in this question has two "significant main effects" for the independent variables: type of reading program and past level of reading comprehension. And a "significant interaction effect" means that the effects of the different reading programs varied significantly for students at different reading levels. For example, "Reading Program A" may have been highly effective for above average students, moderately effective for average students, yet ineffective for below average students. On the other hand, "Reading Program B" may have been only effective for below average students, while "Reading Program C" may not have been effective for any students.

The Drugs-R-Us company wants to compare the effectiveness of 3 new antidepressant medications. Patients with depression are randomly assigned to one of the three medications and depressive symptoms are measured at weeks 1, 6, and 12. Which type of research design would be most appropriate for this study? A ABAB B between subjects C within subjects D mixed

The Correct Answer is "D" A mixed research design has at least one between-subjects independent variable and at least one repeated measures variable (or within-subjects variable). Since this study is comparing the effects on three different groups of subjects (i.e., a between-subjects variable) combined with the use of a repeated measures (within-subjects) variable, it would be considered a mixed design. An ABAB design is a type of reversal design, in which a baseline measure of a behavior is obtained (the "A" phase), the behavior is again measured after a treatment is administered (the "B" phase), the treatment is removed or reversed and the behavior is measured again (the second "A"), and the treatment is then re-applied)

A primary reinforcer is the same as: A A conditioned reinforcer B The first reinforcer used C Pseudoconditioning D An unconditioned reinforcer

The Correct Answer is "D" A primary reinforcer is the same as an unconditioned reinforcer. These are items that acquire their reinforcing value without special training. Food and water are examples of primary reinforcers.

A test with limited ceiling would have a ____________ distribution shape. A normal B flat C positively skewed D negatively skewed

The Correct Answer is "D" A test with limited ceiling has an inadequate number of difficult items resulting in few low scores. Therefore the distribution would be negatively skewed.

According to the Specialty Guidelines for Forensic Psychologists (APA, 1991), if a psychological evaluation is ordered by a court and the defendant to be evaluated informs the evaluating psychologist that he does not wish to cooperate, the psychologist should: A conduct the evaluation B refuse to conduct the evaluation C refuse to conduct the evaluation unless the defendant consents D postpone the evaluation and notify the defendant's attorney

The Correct Answer is "D" Although it is not necessary to obtain informed consent for court-ordered evaluations, the subject of the evaluation should be informed of its purpose. Furthermore, if the subject does not want to be evaluated, the subject's attorney should be contacted before taking further action. If the attorney also objects to the evaluation, the psychologist should then notify the court and respond as directed

You have a client who has been abusing cocaine. He has recently read an article about neurotransmitters and does not want to take any type of medication because he does not want to "mess with his brain." You provide him with the information to reduce his anxiety. You explain that cocaine affects the brain in following manner. A It increases epinephrine. B It increases glutamate. C It decreases serotonin. D It increases dopamine

The Correct Answer is "D" Cocaine is believed to block the reuptake of dopamine. As dopamine collects in the neurons of the limbic system, it continues to stimulate receiving cells.

Which of the following seizures is also known as temporal lobe epilepsy? A Absence B Myoclonic C Grand mal D Complex partial

The Correct Answer is "D" D. Complex partial seizures begin in the temporal lobe, involve impaired consciousness and involuntary chewing, lip smacking, fidgeting, and walking in circles. "Complex" refers to the seizures which impact consciousness. While partial seizures begin in a limited area of the brain, generalized seizures are caused by electrical impulses throughout the entire brain. The other responses are all types of generalized seizures. Absence seizures (a.) involve a brief loss of consciousness with few or no other symptoms. The primary symptom of a myoclonic seizure (b.) is brief jerky contractions of muscles in different parts of the body, most often in the legs and arms. A grand mal (c.) seizure involves convulsions, unconsciousness, and muscle rigidity.

*A selection test that has a validity coefficient of .50 will have the greatest incremental validity when: A the selection ratio is .95 and the base rate is .20 B the selection ratio is .95 and the base rate is .50 C the selection ratio is .05 and the base rate is .20 D the selection ratio is .05 and the base rate is .50

The Correct Answer is "D" D. Information on a test's incremental validity for differing combinations of base rates, selection ratios, and validity coefficients is provided by the Taylor-Russell tables. Decision-making accuracy can improve on a test with a low or moderate validity coefficient when the selection ratio is low (e.g., .05) and the base rate is moderate (near .50).

Research on constructive memory has linked false recollection to lesions in the: A occipital lobe B parietal lobe C temporal lobe D frontal lobe

The Correct Answer is "D" D. Instead of a literal recollection of past events, the current view of memory is that it is a constructive process - vulnerable to distortions, errors, and factors limiting the accuracy of memories. Neuropsychological research has most consistently identified lesions in the frontal lobes, especially the prefrontal cortex, with false recollections. (See, e.g., D. L. Schachter and S. D. Slotnik, The cognitive neuroscience of memory distortion, Neuron, 2004, 44, 149-160.)

Research on the relationship between self-esteem, biological sex and gender-role suggests: A biological sex has a greater impact than gender role on self-esteem in children B biological sex has a greater impact than gender role on self-esteem in males C a combination of masculine and feminine characteristics and preferences is associated with the highest levels of self-esteem in males D a combination of masculine and feminine characteristics and preferences is associated with the highest levels of self-esteem in children

The Correct Answer is "D" D. J. A. Hall and A. G. Halberstadt's original research found gender-role identity to have a greater impact than biological sex on self-esteem in children and that androgyny (both masculine and feminine preferences and characteristics) is associated with the highest levels of self-esteem in both boys and girls (See: J. A. Hall & A. G. Halberstadt. Masculinity and femininity in children: Development of the Children's Personal Attributes Questionnaire, Developmental Psychology, 1980, 16, 270-280]. Subsequent studies supported these findings and some have also found that masculinity, to a somewhat lesser degree, is associated with higher levels of self-esteem than femininity in both boys and girls.

By the age of 6, which of the following are established? A gender identity and stability B gender identity and constancy C gender constancy and stability D gender constancy, identity and stability

The Correct Answer is "D" D. Kohlberg's cognitive-developmental theory of gender concept development, grounded in Piagetian theory, asserts that children gradually progress through three stages. Gender identity, usually gained by age three, is the ability to correctly label own gender and identify others as boys/men and girls/women. Gender stability - knowledge, usually gained by age four years, that reflects an understanding that one's gender remains the same throughout life. Gender constancy: realization, around age 5, 6 or 7 years, that one's gender stays the same even with alterations in appearance, behaviors, or desires.

Which of the following techniques is most similar to latent trait analysis (LTA)? A cluster sampling B analysis of covariance C multitrait-multimethod matrix D latent class analysis

The Correct Answer is "D" D. Latent class analysis, like latent trait analysis, is used to identify the underlying latent structure of a set of observed data. The techniques differ in that in LTA, the latent variable that determines the structure is continuous whereas in LCA, the latent variable is nominal. Cluster sampling (a.) is a sampling technique in which groups of participants are selected instead of individuals. Used to statistically remove the effects of the covariate, or an extraneous variable, on the dependent variable, the analysis of covariance (ANCOVA) (b.) makes it easier to determine the effects of the independent variable on the dependent variable. The multitrait-multimethod matrix (c.) is used to assess convergent and divergent validity

Which of the following is NOT generally considered a direct threat to external validity? A order effects B hawthorne Effect C interaction between selection and treatment D history

The Correct Answer is "D" Distinguishing between internal and external threats to validity can be difficult. Indeed, some experts disagree on how to categorize some of them. However, all of the choices except D are generally considered to be threats to external validity. Order effects (A) (also known as carryover effects) occurs in repeated measures designs, or in studies in which the same subjects are exposed to more than one treatment. For example, in a study on the effects of marital therapy interventions, couples are given relaxation training followed by communication training. If significant improvement occurs, it may be due to relaxation training preceding communication training; therefore, the results could not be generalized to situations in which subjects only receive communication training. The Hawthorne effect (B) occurs when subjects behave differently due to the fact that they are participating in research. Obviously this threatens external validity since the results cannot be generalized to real-life situations in which people are not participating in research. Interaction between selection and treatment (C) refers to when a treatment has different effects depending on the selection of subjects. For example, studies that only use undergraduate students (as many studies do) might not generalize to non-undergraduate students. Finally, history (D) refers to an external event, other than the experimental treatment, that affects scores on the DV. This is primarily considered a threat to internal validity. For example, if a study on the effects of a new treatment for depression began several weeks before the events on "9-11" and concluded several weeks after "9-11," the results might indicate that the new treatment is not effective. However, this might not be a valid conclusion due to the effects of history.

In the context of expectancy theory, valence refers to: A the willingness of a worker to exert effort. B the strength of the worker's needs. C the desirability of the job itself. D the desirability of the consequences of performance

The Correct Answer is "D" Expectancy theory predicts that motivation is related to three phenomena: beliefs about the relationship between effort and performance; beliefs about the relationship between performance and outcomes; and the desirability of those outcomes. The latter is referred to as valence

Which of the following is a mental health service that HMOs must provide under federal requirements? A pre- and post-pregnancy counseling. B psychodynamically oriented psychotherapy. C inpatient psychiatric care. D short-term outpatient evaluative crisis intervention.

The Correct Answer is "D" Federal laws regarding qualification for an HMO requires that the company provide "short-term (not to exceed 20 visits), outpatient evaluative crisis intervention services", and "medical treatment and referral for alcohol and drug addiction." These are the only qualifications related to mental health services. Most HMOs provide certain additional services, because doing so reduces their costs in the long-run.

Clomipramine and fluoxetine alleviate the symptoms of Obsessive-Compulsive Disorder by affecting levels of A epinephrine. B norepinephrine. C dopamine. D serotonin

The Correct Answer is "D" If you're unsure about the correct answer to a question about neurotransmitters, serotonin is usually a good guess since it's involved in so many functions and activities. And, in fact, in OCD, the usefulness of the antidepressant drugs is believed to be due to their ability to increase serotonin activity.

Which of the following would be used to determine the probability that examinees of different ability levels are able to answer a particular test item correctly? A criterion-related validity coefficient B item discrimination index C item difficulty index D item characteristic curve

The Correct Answer is "D" Item characteristic curves (ICCs), which are associated with item response theory, are graphs that depict individual test items in terms of the percentage of individuals in different ability groups who answered the item correctly. For example, an ICC for an individual test item might show that 80% of people in the highest ability group, 40% of people in the middle ability group, and 5% of people in the lowest ability group answered the item correctly. Although costly to derive, ICCs provide much information about individual test items, including their difficulty, discriminability, and probability that the item will be guessed correctly

Tiedeman and O'Hara emphasize which of the following in the process of career development? A innate drives B interests C aptitude D identity

The Correct Answer is "D" Knowing that Tiedeman and O'Hara based their model on Erikson's psychosocial theory of identity would have helped you select the correct answer. They proposed that a person develops a vocational identity through a process of differentiation (realizing that a career does not fit with one's personality) and integration (identifying with a career).

Kohlberg's theory of moral development focuses on moral reasoning. With regard to the linkage between moral reasoning and moral action, Kohlberg would most likely agree with which of the following? A There is a direct, one-to-one link between moral reasoning and behavior. B The link between moral reasoning and behavior is mediated by the individual's previous experience with the situation. C The link between moral reasoning and behavior is mediated by the individual's IQ. D There isn't a one-to-one correspondence between moral reasoning and behavior, although, the higher the stage of moral reasoning, the stronger the link is likely to be.

The Correct Answer is "D" Kohlberg believed there is a link between moral reasoning and moral action but felt that the correspondence was greatest at the higher stages of moral reasoning.

The cortex is the least developed part of the brain at birth. Subsequent development is due primarily to: A the growth of new neurons. B the growth of new neurons and glial cells. C increases in the size of existing neurons. D myelination of existing neurons

The Correct Answer is "D" Nearly all of the neurons are present at birth, and continued development of the brain following birth is due primarily to an increased number of dendrites and myelination of the existing neurons.

The psychiatric diagnosis most associated with completed suicide is A Bipolar I Disorder, Most Recent Episode Depressed. B Bipolar Disorder, Most Recent Episode Mixed. C Major Depressive Disorder, Recurrent. D Major Depressive Disorder, Recurrent, With Psychotic Features

The Correct Answer is "D" Of all the diagnostic categories, Mood Disorders are associated with the highest risk of completed suicide -- approximately 15% to 20% of individuals with Mood Disorders eventually commit suicide. And individuals with depressed and psychotic features are at five times greater risk for suicide as compared to patients with other Mood Disorders

*In Piaget's concrete operational stage the child is able to conserve due to the development of reversibility and decentration. Other achievements of this stage are: A Deferred imitation B Object permanence C Propositional thought D Transitivity

The Correct Answer is "D" Other achievements of the concrete operational stage are transitivity, or the ability to mentally sort objects; and hierarchical classification--the ability to sort object into classes and subclasses based on similarities and differences among groups. Deferred imitation and object permanence are achievements of the sensorimotor stage. Deferred imitation is the ability to imitate an observed act at a later point in time. Object permanence is the understanding that objects continue to exist even when they are not there. Finally, propositional thought (C) is developed in the Formal Operational Stage. It is the ability to evaluate the logical validity of verbal assertions without having to use real-world circumstances.

The parents of two children, Jimmy, 4, and Janie, 11, divorce without exposing their children to overt conflict. Based on the results on research investigating the effects of divorce on children, you would expect that: A Janie will experience more negative effects than Jimmy initially, but Jimmy will experience a more negative delayed reaction. B Janie will experience more negative effects initially as well as a more negative delayed reaction. C Jimmy will experience more negative effects initially as well as a more negative delayed reaction. D Jimmy will experience more negative effects than Janie initially, but Janie will experience a more negative delayed reaction

The Correct Answer is "D" Research by experts in this field, such as Wallerstein, has revealed that developmental level is an important mediator of the effects of parental divorce on children. Apparently, older children are better able to understand what is happening during a divorce than younger children. As a result, they exhibit fewer negative effects at the time of the divorce itself. However, they often display delayed negative effects, such as painful memories and fears that they will have an unsuccessful marriage

The Solomon four-group design is: A a quasi-experimental design B used to analyze the difference scores among four different treatment groups C used to reduce practice effects D used to evaluate the effects of pretesting

The Correct Answer is "D" The Solomon four-group design is a true experimental design used to evaluate the effects of pretesting, since some groups are pretested and others are not.

A self-report inventory that assesses general psychiatric symptoms on a Likert-type scale is the: A MCMI-III B Rorschach C MMPI-II D SCL-90

The Correct Answer is "D" The Symptom Checklist 90 (SCL-90) is a self-report inventory assessing general psychiatric symptoms of anxiety, depression, somatization, obsessive-compulsiveness, and hostility. Clients respond to the stimulus term, such as "feel nervous inside" on a 5-point Likert type scale from "not at all" which gives a score of 0, to "extremely" which gives a score of 4

Your client is terminating your psychological services by mutual agreement after two years. She's going to be married the following May and would like you to be one of her wedding attendants. You should A explain to her that the two of you should have no further contact. B accept; but under condition that you participate in the wedding only and no other type of friendship can be pursued. C explain that you cannot have a friendship with her until two years have passed. D decline the invitation; explaining that this would be a multiple relationship.

The Correct Answer is "D" The best solution in regard to the ethical standards is to avoid multiple relationships. It is possible that your client could need therapeutic services again and if you become her friend, you will no longer be able to offer her that option

"According to current research, the Rorschach has very low validity." This statement is A true. B true, except for children between ages 5-10. C true, except for its use with a nonverbal population. D false

The Correct Answer is "D" The correct answer is false. The Rorschach, while having a reputation for low validity due to its clinical origins, has increased in validity since the development of John Exner's scoring system. Hiller, Bornstein, Rosenthal, Berry and Brunell-Neuleib report in Psychological Assessment (September, 1999) that since earlier critiques, two recent developments have given the Rorschach a more favorable outlook. First is the development of Exner's Comprehensive System for the Rorschach (Exner, 1974; 1978) which systematized scoring procedures for the Rorschach, and reported findings from a large archive of Rorschach protocols of normal adults, as well as from various psychiatric groups. Second, two meta-analyses were published, both comparing criterion-related validity evidence for the Rorschach to that of its chief rival, the MMPI (Atkinson, 1986; Parker, Hanson, and Hemsley, 1989). These studies conclude that validity is roughly equivalent for both instruments. Moreover, in Hiller et al.'s (1999) article, "A Comparative Meta-Analysis of Rorschach and MMPI Validity," their findings indicate that the MMPI had larger validity coefficients than the Rorschach for studies using psychiatric diagnoses and self-report measures as criterion variables, whereas the Rorschach had larger validity coefficients than the MMPI for studies using objective criterion variables.

Kohlberg would agree with all of the following except: A moral development is an outgrowth of cognitive development. B each stage of development represents an organized whole. C stage 5 and 6 are not reached by most people. D moral development stages have an inherent male bias

The Correct Answer is "D" The first three answers are descriptions of Kohlberg's theory. Answer D is Carol Gilligan's criticism of Kohlberg. Gilligan thought males are likely to refer to principles of justice and fairness when making decisions, while females are more likely to refer to interpersonal connectedness and care. Research has not generally supported Gilligan's hypothesis

A 14-year old is brought to therapy after he is caught stealing from a local store. His parents say they have been having trouble with their son for the past two months - they are constantly fighting, and he is irritable and sullen most of the time, occasionally has outbursts of anger, spends most of his time in his room playing with video games, has been caught smoking, and doesn't seem to sleep as much as he should. These symptoms are most suggestive of A ADHD. B Conduct Disorder. C Dysthymic Disorder. D Major Depressive Disorder

The Correct Answer is "D" The key to recognizing the correct answer to this question is knowing that, in children and adolescents, Major Depression often involves an irritable (rather than depressed) mood as well as acting out behaviors

If you hear arguments against your opinion, followed by arguments against the opposing opinion, what is likely to happen? A You will become very confused. B You will become more dogmatic in your original opinion. C You will change your opinion. D Your resistance to future opposing arguments will be increased

The Correct Answer is "D" This question is indirectly referencing McGuire's inoculation theory, which proposes that a particular attitude or belief can be strengthened by exposing someone to the opposing belief -- especially when the opposing argument is weak or the person is supplied with counter-arguments against the opposing belief. Note that this is analogous to medical inoculation, which involves injection of a weak form of a germ so the body can build up defenses against that germ.

*In a clinical trial of a new drug, the null hypothesis is the new drug is, on average, no better than the current drug. It is concluded that the two drugs produce the same effect when in fact the new drug is superior. This is: A corrected by reducing the power of the test B corrected by reducing the sample size C a Type I error D a Type II error

The Correct Answer is "D" Type II errors occur when the null hypothesis is not rejected when it is in fact false; Type I errors are often considered more serious as the null hypothesis is wrongly rejected. For example, in the clinical trial of a new drug, this would be concluding that the new drug was better when in fact it was not. Type I and II errors are inversely related: as the probability of a Type I error increases, the probability of a Type II error decreases, and vice versa.

In the multitrait-multimethod matrix, a low heterotrait-heteromethod coefficient would indicate: A low convergent validity B low divergent validity C high convergent validity D high divergent validity

The Correct Answer is "D" Use of a multitrait-multimethod matrix is one method of assessing a test's construct validity. The matrix contains correlations among different tests that measure both the same and different traits using similar and different methodologies. The heterotrait-heteromethod coefficient, one of the correlation coefficients that would appear on this matrix, reflects the correlation between two tests that measure different (hetero) traits using different (hetero) methods. An example might be the correlation between vocabulary subtest scores on the WAIS-III for intelligence and scores on the Beck Depression Inventory for depression. Since these measures presumably measure different constructs, the correlation coefficient should be low, indicating high divergent or discriminant validity.

To reduce a client's fear of cats, a behavioral psychologist has the client imagine approaching a cat and then, when anxiety occurs, pair that image with deep muscle relaxation. This technique is known as: A covert sensitization. B guided imagery. C implosive therapy. D reciprocal inhibition

The Correct Answer is "D" You may have been looking for counterconditioning or systematic desensitization as the correct response. Both involve reducing anxiety by pairing it with relaxation or other incompatible response. This technique was originally described by Wolpe, who referred to it as reciprocal inhibition.

Long-term potentiation was initially observed in the cells of the: A basal ganglia. B amygdala. C corpus callosum. D hippocampus.

The Correct Answer is "D" Long-term potentiation refers to certain changes in the neuron following high-frequency stimulation. It has been linked to the formation of long-term memories. Not surprisingly, long-term potentiation was first found in the hippocampus, which has been identified as a brain structure involved in the transfer of information from short-term to long-term memory

An accident victim has been partially stabilized in a hospital emergency room and is being transferred to an intensive care unit for continued assessment and treatment. At this point, which of the following is commonly used to evaluate for brain injuries? A Rancho Los Amigo Scale B AVPU Scale C Glascow Coma Scale D Disability Rating Scale

The Glasgow Coma Scale is a standardized test that rates three categories of patient responses: eye opening, best motor response, and best verbal response. The eye opening tests indicate the function of the brain's activating center, the best verbal response indicates the condition of the central nervous system within the cerebral cortex, and the best motor response examines the ability to move arms and legs. The levels of responses indicate the degree of nervous system or brain impairment with "1" being the lowest possible score in each category. Using these scores, which range from 3 to 15 to form the Glascow Coma Scale, brain injuries are classified as mild, moderate or severe. The Rancho Los Amigo Scale (response "A") is an evaluation of eight levels of cognitive functioning administered in acute rehabilitative settings following release from intensive care. The AVPU Scale (response "B") is primarily used after injury occurs or as pre-hospital information. It rates if someone is alert, responds to voice, responds to pain or is unresponsive. The Disability Rating Scale or DRS (response "D") is for gauging general level of disability (e.g., self care, dependence on others) from "none" to "extreme vegetative state."

Scores on WAIS _________ index remain consistenty throughout adulthood with scores continuing to increase into the early 50s before starting to decline a) Processing Speed b) Perceptual Reasoning c) Working Memory d) Verbal Comprehension

Verbal comprehension

According to ___________, symbolic play offers young children opportunities to engagen activities that are dvptl more advanced than they could successfully perform alone: a) JH Flavell b) E Gibson c) L Vygotsky d) J Piaget

Vygotsky-symbolic play provides a zone of proximal dvpt

A problem with conducting multiple comparisons is that: a) the more comparisons that are done, the higher is the probability that at least one Type I error will be made b) the more comparisons that are done, the higher is the probability that at least one Type II error will be made c) post-hoc tests cannot take possible error into account because the alpha level cannot be controlled for individual comparisons d) post-hoc tests can only take possible error into account when pairwise comparisons are made

a

A psychologist would most likely administer the Vineland Scale when. a. she is assessing an individual for Mental Retardation. b. she is assessing an individual for Pervasive Developmental Disorder. c. she is checking an individual's recovery following brain injury. d. she wants to assess the cognitive skills of an individual who has limited English proficiency.

a

DSM-IV diagnosis is a ___ variable a)discrete b)continuous c)distinct d)categorical

a

Delirium shares many symptoms with schizophrenia, schizophreniform disorder, and certain other psychotic disorders, such as disordered thinking, hallucinations, and delusions. Thus, the differential diagnosis of these disorders is often difficult. Generally speaking, however, delirium can be distinguished from the psychotic disorders because. a. the symptoms of delirium tend to be random and haphazard while the symptoms of the psychotic disorders are ordinarily systematized. b. the onset of symptoms in delirium is insidious while the onset of symptoms in psychotic disorders is usually rapid. c. clouding of consciousness is rare in delirium but common in the psychotic disorders. d. the psychotic disorders involve hallucinations and delusions but these symptoms are never present in delirium.

a

If a study found that the treated and non-treated groups differed in terms of their scores, when in the population, no such difference exists, this would be known as: a) a Type I error b) a Type II error c) an extraneous variable d) the standard error of the mean

a

In a study testing the effects of a new drug, 40 subjects are selected from a hospital ward, with subjects randomly assigned to two groups of 20 each. Subject in one group are administered the drug and subjects in the other take a placebo. The researcher will use the: a) t-test for independent samples b) factorial ANOVA c) sample t-test d) chi-square

a

In contrast to conventional antipsychotic drugs, the atypical drug, clozapine. a. is more likely to alleviate symptoms of depression and hostility. b. is more likely to produce irreversible tardive dyskinesia. c. increases both dopamine and serotonin activity. d. is less likely to cause agranulocytosis.

a

Research suggests that, in contrast to normal-weight people, overweight people eat in response to. a. external cues. b.hunger pains, which are more frequent in overweight people. c. sugary types of foods. d. stressful events.

a

patient factors that correlate positively with therapy outcome include all of the following except (more than one answer): a)age b) educational level, c)ego strength, d)gender e) anxiety tolerance, f) social class, g) attractiveness, h) client participation. i) intelligence

a, d, f

Z-scores and percentile ranks are examples of what kind of scale? a) transformed scores b) stanine scores c) ordinal scales d) t-scores

a

_____________ offer the greatest internal validity, since subjects are randomly assigned and the experimenter has the greatest control over experimental conditions. a) True experiments b) Quasi-experiments c) Time-series designs d) Correlational designs

a

Note that these TWO scales sometimes have a zero point but that it's an arbitrary, not an absolute zero: a)interval b)ordinal c)ratio d) nominal e)ratio

a b

The Wonderlic Personnel Test is : a) brief test of mental ability for adults. b) measure of global and facet job satisfaction. c) test of English-language proficiency. d) apparatus test of psychomotor skills.

a) brief test of mental ability for adults.--use for job selection criticised on the grounds that it discriminates against culturally-diverse groups

Due to improved treatments for acute lymphacytic leukemia, at least 50% of chidren with this condition now have a normal life expectancy. Research investigating the cognitive impacts of treatments -CNS chemo and irradiation, hav found that: a) CNS chemo and irradiation are both assoc with neurocog deficits b) CNS chemo is assoc with neur deficits but not irradiation c) CNS irradiation is assoc with neuro deficits but not chemo d) any apparent negative impact of either treatment in neuro funcitoning is due to the disease

a) CNS chemo and irradiation are both assoc with neurocog deficits

Which best addresses the issue of distributive justice in an organisation? a) Did employees consider the outcomes of an organizational decision reasonable and just? b) Was the organizational procedure used to make the decision fair? c) Did employees feel they were treated fairly when informed about the decision? d) Were the employer's reasons for making the decision reasonable and valid?

a) Did employees consider the outcomes of an organizational decision reasonable and just?

When using flooding to treat Specific Phobia, which instrument would be most usitoring a client's progress: a) SUDS b) SCID c) CAPS d) BDI

a) SUDS

Martial satifsaction across the lifespan tends to be: a) a U-shaped pattern with the lowest levels of reported satisfaction during the middle adult years b) an inverted U-shaped pattern with the lowest levels of reported satisfaction during the early adult and older adult years c) a positive linear pattern with reported satisfaction gradually increasing over the lifespan d) a negative linear pattern with reported satisfaction gradually decreasing over the lifespan

a) a U-shaped pattern with the lowest levels of reported satisfaction during the middle adult years

Setting fees on the basis of sliding scale is: a) acceptable but not explicitly mentioned in the Ethics Code. b) unacceptable but not explicitly mentioned in the Ethics Code. c) explicitly recommended in the Ethics Code. d) explicitly prohibited in the Ethics Code

a) acceptable but not explicitly mentioned in the Ethics Code

All of the following enhance the effectiveness of punishment for eliminating a behav except: a) administering the punishment on an intermittent schedule b) administering the punishment immediately after the target behavior occurs c) administering the punishment from the beginning at a moderate strength d) giving a "warning" cue just prior to administering the punishment

a) administering the punishment on an intermittent schedule

When a corporation uses multiple cutoff in its employee selection process, which is most likely to occur: a) an applicant who obtains very high scores on tests of mathematical and mechanical intelligence will not be hired because he fails a test of verbal intelligence by one point b) an applicant who scores low on a test of verbal intelligence obtains very high scores on tests of mechanical and mathematical intelligence, so he is hired as an airline mechanic c) an applicant's score on tests of verbal, mathematical, and mechanical intelligence place him in the lowest of fifteen criterion groups, so he is not hired d) an applicant's score on a test of mechanical intelligence compares favorably to the scores of successful employees, so he is hired as an airline mechanic

a) an applicant who obtains very high scores on tests of mathematical and mechanical intelligence will not be hired because he fails a test of verbal intelligence by one point--when multiple cutoff is used, all the predictor tests must be passed

Researchers have found that absence of bio father is assoc with: a) an earlier age of first sexual intercourse for male and female adolescents. b) an earlier age of first sexual intercourse for male (but not female) adolescents. c) an earlier age of first sexual intercourse for female (but not male) adolescents. d) a later age of first sexual intercourse for male and female adolescents.

a) an earlier age of first sexual intercourse for male and female adolescents

A researcher conducts a study to compare 3 strategies for incr students' underst of stats. After admin each strategy to a diff group of students, she realises that the students with highest level of math aptitude were accidentally assigned to Strategy 1. Conseq, to compare the stats test scores obtained by participants in the 3 groups, she should use which technique? a) analysis of covariance (ANCOVA) b) randomized block ANOVA c) multivariate analysis of variance (MANOVA) d) split-plot ANOVA

a) analysis of covariance (ANCOVA)-removes the effects of an extraneous variable

survival analysis is used to: a) assess the length of time to the occurrence of a critical event b) evaluate the causal (predictive) influences of multiple latent factors c) determine, for a sample of people, how many types the sample represents d) identify natural grouping among a collection of observations

a) assess the length of time to the occurrence of a critical event

An antagonist drug works by: a) blocking a neurotransmitter's access to the site b) exerting the opposite effect of a neurotransmitter. c) facilitating the action of a neurotransmitter at the site. d) mimicking the effect of a neurotransmitter.

a) blocking a neurotransmitter's access to the site

When using network therapy with Native Am clients, the therapist acts primarily as: a) catalyst b) educator c) "blank screen" d) "wise parent"

a) catalyst

Regarding depression, which is true: a) children and adolescents are more likely than adults to become aggressive during depressive episodes b) children and adolescents are less likely than adults to become aggressive during depressive episodes c) children, adolescents, and adults are about equally likely to become aggressive during depressive episodes d) children, adolescents, and adults tend to become less aggressive during depressive episodes

a) children and adolescents are more likely than adults to become aggressive during depressive episodes

A researcher would use which technique to identify homogenous groups from a collection of observations? a) cluster analysis b) multitrait-multimethod matrix c) discriminant function analysis d) structural equation modeling

a) cluster analysis (eg diagnostic groups)

In psychodynamic theory, used to attempt to avoid conflict a)defense mech b)repression c)denial d) conflict

a) defense Mech

When features of a study influence a participant's behav: a) demand characteristics b) the Rosenthal effect c) experimenter bias d) the Hawthorne effect

a) demand characteristics

CD is diagnosed in the presence of at least 3 of these except: a) disobedience of authority figures b) aggression to people/animals c) destruction of property d) serious violation of rules

a) disobedience of authority figures

transferring an emotion from an original idea/object to a more acceptable/safer one: a)displacement b)projection c)fixation d)sublimation

a) displacement

Brain lateralisation for lang is first evident a) during the first year of life. b) during the second year of life. c) during the third year of life. d) after the fourth year of life.

a) during the first year of life.

As a dithesis-stress model, Beck's cog theory predicts that which of the following serves as the diathesis in the dvpt of depressive symptoms? a) dysfunctional schemas b) a genetic predisposition c) exposure to a negative event d) faulty attributions

a) dysfunctional schemas

Ho recommended the use of a ______ approach with Af-Am: a) ecostructural b) solution-focused c) systemic family therapy d) ethnographic

a) ecostructural

A 12 month old baby who is classified as resistant is reunited with mother in Strange Situation. The baby will: a) hit or push his mother when she approaches and continue to cry after she picks him up b) run away from his mother when she approaches and fail to cling to her when she picks him up c) seem very agitated and confused and not calm down when his mother picks him up d) seem happy to see his mother but resist her attempts to pick him up

a) hit or push his mother when she approaches and continue to cry after she picks him up

Adding a constant to every score in a distribution of scores will: a) increase the distribution's mean b) increase the distribution's standard deviation c) increase the distribution's mean and standard deviation d) not increase the distribution's mean or standard deviation

a) increase the distribution's mean

Which is true about sensory memory? a) information is retained for only ¼ to 2 seconds b) without rehearsal, information is retained for only 3 to 5 seconds c) with attention, information is retained for up to 30 seconds d) with rehearsal, information may be retained indefinitely

a) information is retained for only ¼ to 2 seconds

A mediator variable: a) is responsible for (causes) the relationship between independent and dependent variables. b) affects the direction or strength of the relationship between independent and dependent variables. c) is applied sequentially to participants during the course of the research study. d) is the outcome variable that is predicted in a correlational research study.

a) is responsible for (causes) the relationship between independent and dependent variables.

Criterion contamination is most likely a problem when the measure of job perf is a) is subjectively scored b) is objectively scored c) has limited "floor" or "ceiling" d) is susceptible to "practice effects

a) is subjectively scored

Cross cultural research on OCD suggests that: a) it is similar across countries and cultures in terms of gender distribution, age of onset, comorbidity, and symptom structure. b) it is similar across countries and cultures in terms of symptom structure only. c) it is similar across countries and cultures in terms of gender distribution and age distribution only. d) it is dissimilar across countries and cultures in terms of gender distribution, age of onset, comorbidity, and symptom structure.

a) it is similar across countries and cultures in terms of gender distribution, age of onset, comorbidity, and symptom structure. -differs in topic of obsession

characterized by goals reflecting self-centeredness, competitiveness, & striving for personal power a)mistaken life style b)healthy life style c)open life style d) style of life

a) mistaken life style

Which of these is a direct threat to external (vs internal) validity: a) multiple treatment interference b) statistical regression c) maturation d) Zeigarnik effect

a) multiple treatment interference

Research using a dismantling strat to ident the key component of systematic dysen has generally found that its effectiveness for alleviating anx is due primarily to a) nonreinforced exposure to anxiety-evoking situations b) gradual exposure to anxiety-evoking situations c) acquisition of a response that is incompatible with anxiety d) satiation

a) nonreinforced exposure to anxiety-evoking situations

Which of these is Sx linked to in the presence of more positive symptoms? a) oversensitivity to dopamine b) undersensitivity to dopamine c) structural brain abnormalities d) enlarged ventricles

a) oversensitivity to dopamine

Marlatt and Gordon's relapse prevention model focuses on: a) situations antecedent to relapse b) the perceived consequences of relapse c) controlled drinking d) contingency management

a) situations antecedent to relapse -relapse is a failure to maintain a behav change after treatment and is best avoided by identifyin and dealing with its antecedents (ie negative affect)

A practitioner of ______ family therapy views the initial interview as influencing the course of family therapy and consisting of 4 stages: social, problem, interaction and task-setting: a) strategic b) structural c) object relations d) Milan systemic

a) strategic

Which technique would be most useful for analyzine the time until first relapse for mild, moderate, and heavy smokers who particpate in a self-administered smoking cessation programme? a) survival analysis b) trend analysis c) time-series analysis d) cluster analysis

a) survival analysis-to analyse the amt of time that passes before a partic event occurs

Fielder's contingency model of leadership- a low LPC leader focuses on: a) task motivation b) relationship motivation c) satisficing factors d) hygiene factors

a) task motivation

Using in vivo with response preven: a) the CS is presented repeatedly without the US b) the US is presented repeatedly without the CS c) the CS and US are alternately presented d) the CS and US are presented simultaneously

a) the CS is presented repeatedly without the US

A personnel director raises the cutoff score on a selection test. Which of the following accurately describes a conseq of changing the cutoff score in this way a) the number of true positives is decreased b) the number of false negatives is decreased c) the number of true negatives is decreased d) the number of false positives is increased

a) the number of true positives is decreased

When men view erotic films depicting aggressive acts toward women: a) they become more accepting of violence toward women b) they become more willing to support women's rights c) they become sexually aroused and less aggressive, even when provoked d) they are less likely to act aggressively than males who view non-aggressive erotic films

a) they become more accepting of violence toward women

Triangulation occurs when: a) to reduce tension between them, a two-person system draws in a third party b) to reduce tension between them, the members of a two-person system focus their attention on a third party c) to avoid dealing with the conflict between them, a two-person system forms an alliance against a third person d) to reduce conflict, two members of a three-person system assume a submissive role when interacting with the third dominant member

a) to reduce tension between them, a two-person system draws in a third party

patient projects feelings, thoughts, wishes, or attitudes about persons in past onto analyst. a) transference b)secondary process c)primary process d)free association

a) transference

yields information about client's family constellation, fictional (hidden) goals, & basic beliefs (distorted beliefs & attitudes): a) mistaken lifesyle a)lifestyle investigation c)healthy lifesyle d) style of life.

a)Lifestyle Investigation

analyst develops understanding what, why, how the patient is resisting. issues motivating the behavior explored. a) clarification b)interpretation c)working through d)resistances

a)clarification

Relationships with others as a basic drive: a)object-seeking b)introjects c)archetypes d) collective-seeking

a)object-seeking

love longings displaced from parents (original objects) onto the therapist a) positive transference b) negative transference c) repetition compulsion d)confrontation

a)positive transference

Which model explains why intrinsic motivation decreases after external rewards have been applied? a) the overjustification hypothesis b) cognitive dissonance theory c) the approach-avoidance conflict d) equity theory

a)the overjustification hypothesis

Which type of PD is characterised by symptoms quieting in mid life? a) schizoid b) schizotypal c) antisocial d) histrionic

antisocial

Adults with Alcohol Dependence are most likely to also receive which diagnosis? a) Dependent Personality Disorder b) Avoidant Personality Disorder c) Paranoid Personality Disorder d) Antisocial Personality Disorder

antisocial PD

Exceptions to the psychotherapist-patient privilege: a) apply only when a patient is believed to be a danger to him/herself or others. b) apply only when the exception has been agreed to by the patient or his/her legal representative. c) apply to legal settings and vary from jurisdiction to jurisdiction. d) apply to legal and non-legal settings and vary from jurisdiction to jurisdiction.

apply to legal settings and vary from jurisdiction to jurisdiction.

Manuel, a college student, moved to USA from mexico with his family when he was 4. He has many Anglo friends and only dates Anglo women. He does not participate in cultural traditions with his family. He considers himself American. He can be described as: a) fused b) separated c) assimilated d) bicultural

assimilated

A distinguishing feature of the Montessori method is: a) emphasis on cooperative learning and group achievement b) use of art and music to teach academic subjects c) assumption that all learning stems from sense perception d) placement of children in classes on the basis of developmental level rather than chronological age

assumption that all learning stems from sense perception (Montessori method= use of special materials designed to teach sense discrimination)

118. One of the most influential theories in the field of attitude change is Leon Festinger's "cognitive dissonance theory." Based on the principles of this theory, it can be predicted that. a. the less an individual suffers to obtain something, the more he/she will like it once it has been obtained. b. to relieve dissonance after choosing one of two equally attractive alternatives, an individual will attribute greater attractiveness to the chosen alternative. c. to relieve dissonance after choosing one of two equally attractive alternatives, an individual will attribute his or her decision to dispositional factors. d. voluntary participation in an undesirable activity will result in greater dislike for the activity.

b

A primary distinction between Freud and those psychologists who are collectively identified as "neo-Freudians" (e.g., Fromm, Homey, and Sullivan) is that the latter. a. emphasized the "innate wisdom" of the individual. b. placed greater emphasis on the ego functions and the impact of social influences. c. extended the number of and placed greater emphasis on instinctual (unconscious) forces. d. adopted a teleological approach that viewed behavior as being "pulled" rather than "pushed".

b

A securely attached infant. a. may or may not become upset when the parent leaves the room; is comforted by a stranger; and seeks contact with the parent on reunion. b. becomes upset when the parent leaves the room; is unlikely to be comforted by a stranger; and seeks comfort from the parent on reunion. c. becomes upset when the parent leaves the room; is comforted by a stranger; and may or may not seek contact with the parent on reunion. d. becomes upset when the parent leaves the room; seeks comfort from a stranger if upset; and resumes play when the parent returns.

b

A test developer designs an arithmetic achievement test to be used with junior high school students. When assessing the test's reliability, the test developer is likely to get an overestimate of its reliability if. a. the test contains items that are primarily of a moderate difficulty level. b. the examinees in the sample are more heterogeneous in terms of arithmetic achievement than the examinees the test was designed for. c. the test is a speeded test and the test developer uses the alternate forms method to assess its reliability. d. all of the above.

b

Based on your knowledge of the statistics on suicide, you would predict that a client would be at the highest risk for suicide if. a. he is between the ages of 17 and 27. b. he has expressed a desire to kill himself. c. he has a personality disorder. d. his depressive symptoms have recently worsened.

b

First-and second-grade children who have been identified as being at risk for academic under achievement are provided with a special after-school program. This is an example of. a. primary prevention. b. secondary prevention. c. tertiary prevention. d. crisis intervention.

b

All of these are required for a diagnosis of MR except: a) IQ at or below 70 b) chromosomal abnormalities c) impaired adaptive functioning d) onset prior to 18

b) chromosomal abnormalities

The owner of a company has decided that, to determine how well employees are performing, employees will be watched while performing their jobs for a predetermined period of time. Based on the relevant research, you predict that the employees' performance during the period of observation will. a. be inhibited if the task is easy. b. be inhibited if the task is complex. c. be enhanced whether the task is easy or complex. d. be inhibited whether the task is easy or complex.

b

The standard error of the mean, the standard error of measurement, and the standard error of estimate are all estimates of how far an BLANK value deviates from an BLANK value. a. sample; population. b. observed; true. c. predicted; criterion. d. given; estimated.

b

The tendency of individuals to behave differently when they are being observed in a research setting is due to the: a) carry over effect b) Hawthorne effect c) Pygmalion effect d) Rosenthal effect

b

This research design involves the use of intact groups, rather than groups that are constructed on the basis of random assignment. a) True experiments b) Quasi-experiments c) Time-series designs d) Correlational designs

b

Threats to external validity do not include: a) Hawthorne Effect b) instrumentation c) demand characteristics d) order effects

b

Time is a ____ variable. a)discrete b)continuous c)distinct d)categorical

b

To determine whether or not to reject the null hypothesis, you must compare the statistical value obtained from the test to a critical value. The critical value you use depends on which of the following two factors? a) the availability of the statistical instrument and the pre-set alpha level b) the degrees of freedom for the statistical test and the pre-set alpha level c) the power of the test and the t-score d) the nonparametric distribution and the degrees of freedom for the statistical test

b

To help a depressed client identify the cognitive errors and distortions that are affecting her interpretation of current life events, a practitioner of Beck's cognitive therapy would. a. rely primarily on didactic techniques. b. make use of Socratic questioning. c. rely on verbal persuasion. d. use paradox.

b

What test is used to adjust dependent variable scores to control for the effects of an extraneous variable? a) one-way ANOVA b) ANCOVA c) MANOVA d) factorial ANOVA

b

Which is an example of Beck's cognitive triad? a) If I don't get an "A" in this class, my life will be ruined. b) That's the story of my life - I never do anything right. c) I saw my wife with a man at a restaurant yesterday - she must be having an affair. d) No one is coming to my party - everyone must think I'm a nerd.

b) That's the story of my life - I never do anything right

Which best addresses the issue of procedural justice in an organisation? a) Did employees consider the outcomes of an organizational decision reasonable and just? b) Was the organizational procedure used to make the decision fair? c) Did employees feel they were treated fairly when informed about the decision? d) Were the employer's reasons for making the decision reasonable and valid?

b) Was the organizational procedure used to make the decision fair?

Current theories conceptualise ADD as the consequence of : a) an inability to focus attention for an extended period of time. b) a marked inability to regulate attention. c) inadequate cognitive flexibility. d) lower-than-normal levels of task-related motivation

b) a marked inability to regulate attention.

The premise underlying goal-setting theory is that: a) participation in goals is necessary to ensure commitment to them b) a person's conscious goals and intentions regulate his or her actual behavior c) a person will work to achieve organizational goals only when they are consistent with his or her self-concept d) the willingness to work toward achieving organizational goals is a function of a person's self-efficacy beliefs

b) a person's conscious goals and intentions regulate his or her actual behavior --acceptance is the critical factor

A moderator variable: a) is responsible for (causes) the relationship between independent and dependent variables. b) affects the direction or strength of the relationship between independent and dependent variables. c) is applied sequentially to participants during the course of the research study. d) is the outcome variable that is predicted in a correlational research study.

b) affects the direction or strength of the relationship between independent and dependent variables.

ADHD is characterised by all of these except: a) At least 6 months duration of symptoms b) age of onset prior to age 10 c) presence of at least 6 symptoms d) significant impairment in at least 2 settings

b) age of onset prior to age 10--age of onset is prior to age 7 (6)

A DSM diag of borderline Intellec func is appro when : a) an IQ of 71 to 84 with significant deficits in adaptive functioning b) an IQ of 71 to 84 without significant deficits in adaptive functioning c) an IQ of 71 to 75, deficits in adaptive functioning, and an onset of symptoms before age 18 d) an IQ of 71 to 75, deficits in adaptive functioning, and an onset of symptoms after age 18

b) an IQ of 71 to 84 without significant deficits in adaptive functioning

On of your clients, a 31-yr old man who was sexually abused as a child tells you that he's been having fantasies about sexual contacts with kids. He says he's not acted on them but he is concerned. You should: a) advise him that you are required to make a report with the authorities advise him that you are required to make a report with the authorities b) attempt to determine the degree of risk that he will act on his fantasies c) tell him that this is not unusual for someone who has been sexually abused and continue to work on this issue in therapy d) contact his wife to see if she has noticed any changes in the behavior of their three children

b) attempt to determine the degree of risk that he will act on his fantasies

In terms of NTs, which of the following is true about Sx and mania? a) both disorders have been linked to excessive levels of or oversensitivity to serotonin b) both disorders have been linked to excessive levels of or oversensitivity to dopamine c) schizophrenia has been linked to dopamine but mania has not d) mania has been linked to serotonin but schizophrenia has not

b) both disorders have been linked to excessive levels of or oversensitivity to dopamine

Kohlberg-moral dvpt is directly related to : a) changes in cognitive skills b) changes in cognitive skills and social perspective-taking c) changes in cognitive skills and socioemotional status d) sociocultural influences and changes

b) changes in cognitive skills and social perspective-taking

If a psychologist finds that clients who pay a greater fee for therapy generally express a greater satisfaction, what is his finding supporting? a) the overjustification hypothesis b) cognitive dissonance theory c) the approach-avoidance conflict d) equity theory

b) cognitive dissonance theory

Adequate "floor" is esp important when a test is used to: a) determine if trainees have mastered basic skills b) confirm a diagnosis of Mental Retardation c) assign students to advanced placement classes d) distinguish between successful and unsuccessful employees

b) confirm a diagnosis of Mental Retardation- floor= test's ability to discriminate among examinees at low end of score range

Central achromatopsia is caused by bilateral lesions in the occipitotemporal region of the brani and is characterised by an inability to: a) name colors b) distinguish between different hues c) associate a color with an object d) perceive variations in brightness

b) distinguish between different hues

Authoritative parenting is best described as: a) high in control and acceptance b) high in control but low in responsiveness c) low in control and high in acceptance d) low in control and low in responsiveness

b) high in control but low in responsiveness

Which type of hallucinations occur when falling asleep and are characteristic of narcolepsy? a) hypnopompic b) hypnagogic c) illusions

b) hypnagogic

A patient with Broca's aphasia would be most likely to exhibit: a) apathy b) intense anxiety and depression c) inappropriate giddiness d) explosive episodes of anger and violence

b) intense anxiety and depression

explicitly connecting current behavior to unconscious processes, must be given repeatedly. leads to catharsis, insight, & working through: a) clarification b)interpretation c)working through d)resistances

b) interpretation

When assessing the reliab and validity of a job selection test, your sample includes current employees who are all doing well on the job. The nature of your sample will most likely have which of the following effects? a) it will inflate the reliability and validity coefficients b) it will deflate the reliability and validity coefficients c) it will inflate the reliability coefficient and deflate the validity coefficient d) it will deflate the reliability coefficient and inflate the validity coefficie

b) it will deflate the reliability and validity coefficients -to restrict the range

Research on the impact of cultural identification on substance abuse among native american youth has found that the risk for abuse is: a) lowest for those who are acculturated into the non-Native American (mainstream) culture b) lowest for those who can adapt to both Native American and non-Native American cultures c) highest for those who strongly identify with the Native American culture and reject the non-Native American culture d) highest for those who do not strongly identify with either the Native American or non-Native American culture

b) lowest for those who can adapt to both Native American and non-Native American cultures

Research comparing the perf of older and younger adults on free recall and recog memory tasks has found that: a) younger adults benefit more than older adults from retrieval cues. b) older adults benefit more than younger adults from retrieval cues. c) younger and older adults benefit to about the same degree from retrieval cues. d) younger and older adults benefit from retrieval cues only when the task is easy to moderately difficult.

b) older adults benefit more than younger adults from retrieval cues.

Dr Dilemma is contacted by an insurance company which wants info about a previous client of hers. The client's fees were paid by the insurance company and they now want info for part of it's regular peer review process. The best course of action would be to: a) provide the insurance company with the information it requests b) provide the insurance company with the information it requests only after ensuring that the company will take steps to protect the client's confidentiality c) provide the insurance company with the information it requests in a way that allows the client's identity to be concealed d) refuse to release any information to the company unless she feels it is in the client's best interests

b) provide the insurance company with the information it requests only after ensuring that the company will take steps to protect the client's confidentiality

Howard's phase model proposes that a client's progress in therapy occurs in 3 predictable phases: a) unfreezing, changing and refreezing b) remoralization, remediation and rehab c) engagement, exploration and evaluation d) contemplation, action, and termination

b) remoralization, remediation and rehab

A primary goal of Minuchin's structural family is to: a) increase differentiation in family members by identifying and restructuring family triangles b) replace rigid and diffuse boundaries with clear boundaries c) balance separation and togetherness d) align boundaries with family systems and subsystems

b) replace rigid and diffuse boundaries with clear boundaries

____ type of Sx is charac by the presence of negative symps or at least 2 attenuated + symps with an absence of prominent delusions, hallucs, catatonic behav or disorgan speech/behav a) undifferentiated b) residual c) hebephrenic d) mixed

b) residual

Kirkpatrick identifies 4 levels of rigor in the eval of training programs. According to his model, which of the following reps the highest level of eval? a) reaction criteria b) results criteria c) behavioral criteria d) learning criteria

b) results criteria

A 12 month old baby who is classified as avoidant is reunited with mother in Strange Situation. The baby will: a) hit or push his mother when she approaches and continue to cry after she picks him up b) run away from his mother when she approaches and fail to cling to her when she picks him up c) seem very agitated and confused and not calm down when his mother picks him up d) seem happy to see his mother but resist her attempts to pick him up

b) run away from his mother when she approaches and fail to cling to her when she picks him up

Which type of PD is characterised by oddities in mannerisms/speech/thinking? a) schizoid b) schizotypal c) antisocial d) histrionic

b) schizotypal

The best conclusion that can be drawn about the stability of IQ test scoers is: a) scores are stable over the lifespan with genetic influences decreasing somewhat over time. b) scores are stable over the lifespan with genetic influences increasing somewhat over time. c) scores are unstable throughout the lifespan with genetic influences decreasing somewhat over time. d) scores are unstable throughout the lifespan with genetic influences increasing somewhat over time.

b) scores are stable over the lifespan with genetic influences increasing somewhat over time.

governed by ego. logical & sequential. functions according to the reality principle a) transference b)secondary process c)primary process d)free association

b) secondary process

The most effective interv for smoking combines nicotine replacement with: a) antidepressants and a self-help manual b) social support and skills training c) covert sensitization and relapse prevention d) stimulus control and relaxation training

b) social support and skills training

A listener is least likely change her attitude as the result of a communicator's message if: a) the listener accidentally overhears the message b) the listener has prior knowledge about the content of the message c) the communicator is arguing against his or her own best interests d) there is a moderate discrepancy between the listener's initial position and the communicator's position

b) the listener has prior knowledge about the content of the message

Which is most useful for explaining the phenomenon of the serial position effect? a) the levels of processing model of memory b) the multi-store (three-box) model of memory c) the distinction between implicit and explicit memory d) the concept of overlearning

b) the multi-store (three-box) model of memory (serial positioning=primacy and recency effect)--this model proposes that there are 3 memory systems=sensory register, short-term, and long-term memory-recent items are in short-term memory and primary items are in long-term memory

In prepubertal children, boys and girls are about equally affected by MDD. In adolescents and adults: a) the rates for males and females remain about equal b) the rate for females is about twice the rate for males c) the rate for females is about four times the rate for males d) the rate for males is about twice the rate for females

b) the rate for females is about twice the rate for males

As predicted by the Taylor-Russell tables, th incremental validity of a selection test that has a validity coeff =.5 will be greatest when a) the selection ratio is .05 and the base rate is .20 b) the selection ratio is .05 and the base rate is .50 c) the selection ratio is .95 and the base rate is .20 d) the selection ratio is .95 and the base rate is .50

b) the selection ratio is .05 and the base rate is .50

Among African Americans, high-context communication is manifested in: a) the use of many words to communicate a message that would require only a few words for a Euro-American b) the use of a few words to communicate a message that would require many words for a Euro-American c) the use of indirect gaze when listening and speaking d) the avoidance of eye contact when listening and speaking

b) the use of a few words to communicate a message that would require many words for a Euro-American (relies more heavily on nonverbal cues)

Weber's law explains why: a) you perceive objects that are close together as a single stimulus or unity b) you can be heard when you whisper in a very quiet room but must shout to be heard in a very loud, noisy room c) you perform better in the presence of others on simple tasks but worse in the presence of others on difficult or complex tasks d) you feel less pain after banging your elbow when you vigorously rub your elbow or apply heat to it

b) you can be heard when you whisper in a very quiet room but must shout to be heard in a very loud, noisy room

collaborative relationship, help client identify & understand his/her style of life & its consequences, reorienting the client's beliefs & goals so that they support a more adaptive lifestyle. a)psychoanalysis b)individual c)person centered d)analytical

b)individual psychotherapy goals & techniques

aggressive drives displaced from parents onto therapist: a) positive transference b) negative transference c) repetition compulsion d)confrontation

b)negative transference

When this design is used the effects of different levels of an independent variable are assessed by administering each level to a different group of subjects and then comparing the status or performance of the groups on the dependent variable:

between group design

According to the logic of ANOVA, if the independent variable has an effect, ______________ variance should be _____________ as/than ______________ variance. a) within-group, the same, between-group b) within-group, greater, between-group c) between-group, greater, within-group d) between-group, less, within-group

c

After several sessions with her therapist, a client starts acting toward the therapist as though he were the client's father. Assuming that the therapist is a practitioner of Gestalt therapy, he is most likely to respond to this transference by. a. ignoring it. b. temporarily assuming the role of the client's father. c. helping the client see the difference between her transference and reality. d. helping the client understand how her past relationship with her father is affecting her current relationships with men.

c

Investigations into the relationship between client race or ethnicity and premature termination from therapy have shown that. a. matching minority clients and therapists in terms of race/ethnicity substantially reduces premature dropout rates. b. premature dropout rates among minority clients arc, in fact, due to the confounding effects of socioeconomic status rather than to race or ethnicity. c. minority clients do have higher premature dropout rates, which seem to be attributable to negative attitudes toward the therapist and/or a perception of therapy as nonbeneficial. d. premature dropout rates among minority clients is largely a myth.

c

Research on the effects of maternal employment suggests that it. a. has positive effects on the cognitive development of all children. b. has negative effects on the cognitive development of all children. c. may have negative effects on the cognitive development of middle-class boys. d. may have negative effects on the cognitive development of middle-class girls.

c

You've been seeing a client for eight months. During that time, he has complained several times about the course of therapy and, each time, you have discussed this matter with him and modified treatment goals. During the last three sessions, he has again mentioned that he is not happy with therapy. As an ethical psychologist, your best course of action would be to. a. realize that his behavior is a manifestation of resistance and ignore it. b. make a clinical decision as to whether or not the client is benefitting from therapy and, if you determine that he is, encourage him to continue in therapy. c. discuss his dissatisfaction with him and the possibility of a referral to another therapist. d. tell him that, as an ethical therapist, you must terminate treatment since he does not feel he is benefitting.

c

Which med is most effective for preamture ejaculation? a) a drug that stimulates dopamine receptors b) a drug that inhibits GABA receptors c) a drug that inhibits the reuptake of serotonin d) a drug that stimulates the reuptake of endorphins

c) a drug that inhibits the reuptake of serotonin

Wolpe-reciprocal inhibition involves replacing: a) an avoidance response with a more adaptive response b) a dysfunctional thought with a more functional one c) an anxiety response with a relaxation response d) an external attribution with an internal attribution

c) an anxiety response with a relaxation response

Common assoc features for many of the Somatoform Disorders are: a) disturbances in memory and concentration. b) phobic and paranoid reactions. c) anxiety and depression. d) pseudologia fantastica and vorbeireden.

c) anxiety and depression.

For patients with Alzheimer's dementia, deficits in semantic memory: a) are apparent prior to deficits in episodic memory b) are not apparent until the final stages of the disorder c) are apparent in the early stages of the disorder but may be more subtle than deficits in episodic memory d) are initially indistinguishable from deficits associated with normal age-related cognitive decline

c) are apparent in the early stages of the disorder but may be more subtle than deficits in episodic memory

In comparison to tricyclic antidepressants, SSRIs: a) are more cardiotoxic b) are less likely to negatively affect sleep and anxiety symptoms c) are less likely to cause cognitive impairments d) are more effective for positive symptoms

c) are less likely to cause cognitive impairments

You administer the MMPI to several clients. One of them obtains a t-score of 100 on the F scale. This means: a) the client has attempted to "fake good" b) you should use the F scale score as a correction factor for other scales c) assuming no scoring errors, you should consider the test results invalid d) the client is insecure, intolerant of unconventional behavior, and has a lack of insight

c) assuming no scoring errors, you should consider the test results invalid = attempt to fake bad and when Fscale is > 100 the results should be considered invalid

To use the Taylor-Russell tables to eval a new selection test, you need info about which of the following? a) predictor cutoff, criterion cutoff, and hit rate b) base rate, hit rate, and selection ratio c) base rate, selection ratio, and validity coefficient d) base rate, hit rate, and predictor cutoff

c) base rate, selection ratio, and validity coefficient-these tables are used to find an estimated hit rate

Identifying alternative behavs to replace targeted undesirable behavs is an integral part of: a) response cost b) implosive therapy c) contingency management d) negative practice

c) contingency management --contingency= conseq of the behav; identifying and pplying punishments for undesirable behavs and applying reinf to desirable ones

The left hemisphere tends to be dominant for which of the following funcs? a) perceiving whole shapes from parts b) perceiving direction and distance c) controlling complex and precise movements d) recognizing emotions in facial expressions

c) controlling complex and precise rmovements

Bipolar I can be diagnosed in the presence of all of these except: a) only mania b) cyclical mania and major depression c) cyclical hypomania and major depression

c) cyclical hypomania and major depression

unconscious operations disavowing the feelings, thoughts, or needs that cause anxiety. a)defense mech b)repression c)denial d) conflict

c) denial

A higher-than-normal level of triglycerides is assoc with ______ and is a potential adverse side effect of the atypical antipsychotics, esp olanzapine and clozapine a) nicotine addiction b) confusion and disorientation c) diabetes and obesity d) decreased appetite and weight loss

c) diabetes and obesity

Kobasa: The _____ was defined as a tendency to involve oneself in the activities in life and having a genuine interest in and curiosity about the surrounding world (activities, things, other people).

commitment disposition

According to Carstensen's socioeconomic selectivity theory, diffs in the preferences of adults for emotionally close vs novel social partners is related to: a) gender differences. b) age differences. c) differences in time perspective. d) differences in basic personality traits

c) differences in time perspective. (time left to live. those who perceive time left as unlimited prefer novel partners)

A researcher would use which technique to classify people into criterion groups based on their scores on 2/more predictors? a) cluster analysis b) multitrait-multimethod matrix c) discriminant function analysis d) structural equation modeling

c) discriminant function analysis

A client-centered therapist would most likely respond to a client's transference by: a) interpreting it b) challenging it c) disregarding it d) reflecting it

c) disregarding it

A common migraine: a) begins with an aura and is usually accompanied by nausea. b) is constant and non-throbbing and may be exacerbated by bright lights. c) does not begin with an aura and may be exacerbated by bending over or lifting. d) is sharp and unilateral and is usually accompanied by autonomic symptoms.

c) does not begin with an aura and may be exacerbated by bending over or lifting

reality principle--defers Id's demands. logical ordered aspect of personality; reason & judgment: a)superego b)id c)ego d)oral

c) ego

returning, repeatedly, to a previously mastered stage of development to cope with life's problems: a)displacement b)projection c)fixation d)sublimation

c) fixation

A transformational leader defines a goal to employees in a way that imbues the goal w/meaning and purpose. This is: a) priming b) escalating commitment c) framing d) positioning

c) framing

When working with an Af-Am client exhibiting healthy cultural paranoia, an Anglo therapist would be best advised to: a) refer the client to an African-American therapist b) use a culturally-sensitive approach that ameliorates the client's paranoia c) help the client bring feelings of suspiciousness, frustration, and antipathy toward whites into conscious awareness d) place the client's behavior within the context of racism and distinguish between normal and abnormal responses within that context

c) help the client bring feelings of suspiciousness, frustration, and antipathy toward whites into conscious awareness

As described by Super, career counseling involves: a) helping the individual transition from the anticipation/preoccupation phase to the implementation phase of career development b) instilling an attitude of self-development in the individual so that he/she can adapt to changing work requirements in the future c) helping the individual successfully accomplish the tasks of his/her current stage of career development

c) helping the individual successfully accomplish the tasks of his/her current stage of career development --Also, assessing the individual's self-concept and then identifying careers that match

Tulving defined semantic memory as: a) consists of internal representations of stimulus-response connections b) is more affected by amnesia than procedural memory and episodic memory c) includes rules for manipulating words and symbols d) is coded temporally

c) includes rules for manipulating words and symbols

Which would not be useful for increasing a test's reliab coefficient? a) increasing the sample variance b) ensuring that the average p value is close to .50 c) increasing the heterogeneity of the content domain d) increasing the number of items from 50 to 100

c) increasing the heterogeneity of the content domain

As described by Krumboltz, career counseling involves: a) assessing the individual's self-concept and then identifying careers that match his/her self-concept b) helping the individual transition from the anticipation/preoccupation phase to the implementation phase of career development c) instilling an attitude of self-development in the individual so that he/she can adapt to changing work requirements in the future d) helping the individual successfully accomplish the tasks of his/her current stage of career development

c) instilling an attitude of self-development in the individual so that he/she can adapt to changing work requirements in the future- the purpose of career counseling is to help the individual dvp attitudes and skills that will enable him to adapt to changing work requirements

A measure of test anx is admin to a sample o 50 test takers. A split half reliab coeff of .8 is calculated from scores. The test is then administered to another group of 50 test takers who are more heterogeneous with regard to level of test anx. The split half reliab coeff for the 2nd group is likely to be: a) between .77 and .83 b) between .74 and .86 c) larger than .80 d) smaller than .80

c) larger than .80 -greater variablility of scores=higher reliab coeff

Left-right disorientaion is most likely caused by a lesion in the a) corpus callosum b) basal forebrain c) left parietal region d) right occipital region

c) left parietal region

People with Korsakoff's exhibit anterograde amnesia, retrograde amnesia, and confab. Research suggests which area is most affected? a) temporal lobes b) hippocampus c) mammillary bodies d) basal ganglia

c) mammillary bodies

Prior to age 5, signs of severe MR include: a) minimal capacity for functioning in sensorimotor areas b) some slowness in learning to walk and talk c) marked delays in motor development with little or no communication skills d) noticeable delays in motor development

c) marked delays in motor development with little or no communication skills

The results of a longitudinal study assessing the relationship btwn age and intelligence is most likley to find that: a) general IQ begins to decline in the early 20s b) crystallized intelligence shows more decrements than fluid intelligence beginning in early adulthood c) measures of reasoning and word fluency begin to show declines in scores sooner than measures of verbal meaning and number d) declines in reasoning, word fluency, verbal meaning, and number usually begin between the ages of 30 and 40

c) measures of reasoning and word fluency begin to show declines in scores sooner than measures of verbal meaning and number

A selection test is better predictor of job success for applicants from higher socioeconomic levels than for applicants from lower socioeconomic levels. In this situation, socioeconomic level is: a) suppressor variable b) dependent variable c) moderator variable d) organismic variable

c) moderator variable

Which of the following is true about suicide rates for ages 10-19 for the years of 2000-2005? a) black males had the highest rate and black females has lowest b) white males had highest rate and asian/pacific islander females had lowest c) native americans/alaskan males had highest rate and black females had lowest d) native american/alaskan males had highest and white females had the lowest

c) native americans/alaskan males had highest rate and black females had lowest

Schein proposed that organisational culture exists on which 3 levels? a) time orientation, space orientation, and activity orientation b) microsystem, mesosystem, and macrosystem c) observable artifacts, espoused values and beliefs, and underlying assumptions d) informal organizational elements, formal organizational elements, and individual characteristics of employees

c) observable artifacts, espoused values and beliefs, and underlying assumptions

For Yalom, seeing a client in both individ therapy and group therapy is inadvisable b/c: a) the client will spend too much time in individual therapy discussing problems that arise during the group b) the client will terminate individual therapy prematurely because of the benefits he/she is deriving from the group c) participation in individual therapy may decrease the client's involvement and participation in the group d) participation in both individual and group therapy is likely to be too anxiety-evoking for most clients

c) participation in individual therapy may decrease the client's involvement and participation in the group

From the perspective of Rational-Emotive Behav therapy: a) irrational beliefs are acquired primarily through social learning processes. b) irrational beliefs are acquired largely through the process of selective reinforcement. c) people are biologically prone to the acquisition of irrational beliefs. d) people adopt irrational beliefs as the result of early traumatic events that are still unresolved.

c) people are biologically prone to the acquisition of irrational beliefs.

With regard to terminating the life of animals in research, the Ethics Code: a) permits this practice only when an animal is suffering. b) permits this practice only in the "most unusual circumstances." c) permits this practice when it is appropriate and done in a way that minimizes pain. d) prohibits this practice.

c) permits this practice when it is appropriate and done in a way that minimizes pain.

characterized by lack of logic, ease of substitution of 1 idea with another, & by immediate discharge of energy. functions according to the pleasure principle: a) transference b)secondary process c)primary process d)free association

c) primary process

An Anglo-Am teacher will be least effective in teaching an Af-Am student if she is in the ________ stage of racial identity dvpt and the student is in the _____ stage a) reintegration; pre-encounter b) pseudo-independent; pre-encounter c) reintegration; immersion d) pseudo-independent; encounter

c) reintegration; immersion

individuals repeat in the present feelings & affects from the past: a) positive transference b) negative transference c) repetition compulsion d)confrontation

c) repetition compulsion

Stage 1 of dementia is characterised by all of these except: a) anomia b) anterograde amnesia c) retrograde amnesia d) deficits in visuospatial skills

c) retrograde amnesia-this doesn't occur until stage 2

Piaget-which of the following is a necessary prereq for the dvpt of reversibility? a) self-awareness b) meta-cognition c) symbolism d) hypothetico-deductive reasoning

c) symbolism

A client you have been seeing in therapy tells you that her former therapist made repeated sexual advances to her. She doesn't want you to tell anyone anything. As an ethical psychologist you should: a) inform the client that you are ethically obligated to make a report to the state licensing board b) convince the client that she should make a report to the appropriate authorities c) tell the client that you will maintain confidentiality d) tell the client that you must file a complaint with APA but that you will not reveal her name

c) tell the client that you will maintain confidentiality

A strategic family therapist is most likely to yse which therapeutic techniqe in dealing with a husband and wife who complain that they are constantly arguing with eachother: a) obtain a detailed family history from each partner b) ask the couple to identify the benefits they obtain from arguing c) tell the couple to argue for at least two hours each evening d) work individually with the partner who is most "differentiated

c) tell the couple to argue for at least two hours each evening

In the DSM, the NOS category is appro in all situations EXCEPT: a) symptoms are below the diagnostic threshold for a specific disorder b) there is uncertainty as to whether the disorder is due to a general medical condition, is substance-induced, or is primary c) the available information is adequate only for a "working diagnosis" d) there is insufficient opportunity to complete data collection but symptoms seem to fit a particular diagnostic class

c) the available information is adequate only for a "working diagnosis"-in this situation the term "provisional" is attached to the specific diagnosis

results in assimilation of insights into personality: a) clarification b)interpretation c)working through d)resistances

c) working through

replaces unacceptable urges with their opposites; a)displacement b)Projection c)Reaction Formation d) Rationalization

c)Reaction Formation

Which NT has been implicated in long-term potentiation and excitotoxicity that underlies several neurodegenerative disorders? a) serotonin b) acetylcholine c) norepinephrine d) glutamate

d) glutamate

In factor analysis, the _________ represents the total amount of variability accounted for in a test (variable) by the identified factors and is a reflection of the amount of variance that the test has in common with other measures included in the factor analysis. In other words, it is a measure of common variance.

communality

Noise that _____has less adverse effects on task performance even when it is not actually terminated. Apparently it is the ______ the noise that is the important factor. Intermittent noises and noises

can be controlled (stopped) sense of control over

beta

cannot know beta UNLESS GIVEN-chance of making type II error unless given; which would mean there is a 99% chance researcher has correctly rejected the null.

A person with associative visual agnosia: a) can name a familiar object she sees but does not know how to use it. b) can name a familiar object when she does so spontaneously but not when she is asked to do so. c) cannot name a familiar object she sees but may recognize it when it is placed in her hand and cannot copy or match a drawing of the object. d) cannot name a familiar object she sees but may know what it is used for and can copy or match a drawing of the object.

cannot name a familiar object she sees but may know what it is used for and can copy or match a drawing of the object.

which the therapist uses techniques such as active listening within a genuine, accepting, empathic environment to facilitate clients' growth, and unconditional positive regard

client centered therapy

The goal of ___________ case consultation is to advice the consultee regarding the client's treatment by indirectly or directly assessing the specific client's condition or situation and then making a recommendation about how the consultee should proceed with tx.

client-centered

involves working with the consultee (teacher/therapist) to develop a plan that will enable the consultee to work more effectively with a particular student/patient

client-centered case consultation

Which technique should be used to identify homogenous groups from items measuring different symptoms? a) cluster analysis b) multitrait-multimethod matrix c) discriminant function analysis d) structural equation modeling

cluster analysis

Which analysis would be used to identify subgroups of patients with MDD in terms of predominant symptoms and help-seeking behavs? a) survival analysis b) trend analysis c) time-series analysis d) cluster analysis

cluster analysis- used to divide people, events or variables into meaningful groups based on data collected on those variables

1) reduce the incidents of health problems by increasing preventative activities 2) improve the care of the ill by educating the public about the nature of their disorders & treatment

community psychology strategy--education

When ysing the Cleary (1968) model, test bias is evaluated by: a) comparing the results of a factor analysis for members of different groups. b) comparing the test's regression lines for members of different groups. c) comparing the test's means for members of different groups. d) comparing the "base rates" for members of different groups.

comparing the test's regression lines for members of different groups

To determine if a test you have developed is valid for making diagnostic decisions, you would be most interested in determininif it has adequate : a) content validity b) divergent validity c) concurrent validity d) differential validity

concurrent validity= criterion-related validity used to establish validity when the purpose is to estimate current status on a criterion (ie. method of diagnosis) predictor= test; criterion= accuracy of diagnosis

In most cases, for classical conditioning to be effective, the ________ stimulus should be presented before the _______ stimulus.

conditioned unconditioned note: this is called forward conditioning

When aversive counterconditioning is being used to eliminate a fetish, the fetish obj is: a) unconditioned stimulus. b) unconditioned response. c) conditioned stimulus. d) conditioned response.

conditioned stimulus (paired with an aversive US)

A 12-year olds boy's mother says her son is very argumentative with adults, doesn't do what he is asked to do, and frequently starts fights with his sister and classmates at school. In addition, in the past year, the boy has skipped school at least a half dozen times, and he ran away from home for two days on three different occasions. The mother has been divorced for threel years, and she notes that these problems started soon after she remarried 14 months jago. Based on these symptoms, the most likely diagnosis for the boy is: a) Child Antisocial Behavior b) Adjustment Disorder with Disturbance of Conduct c) Oppositional Defiant Disorder d) Conduct Disorder

conduct disorder

Ridley's explanation of distrust that often exists btwn African american clients and their therapist, the therapists ethnicity is most important factor for: a) confluent paranoiac b) cultural paranoiac c) non-paranoiac d) someone who is experiencing dissonance

confluent paranoiac-high cultural and func paranoia and nondisclosure is due to a combination of pathology and the effects of racism

Racial/Culturall Raciall Identity Development model: Preference for the dominant culture is characteristic of the ____ stage.

conformity

consequences of racial oppression on African Americans may take several forms. 1) internalized oppression, which can involve system beating, system blaming, total avoidance of whites, &/or denial of the political significance of race. 2) Conceptual incarceration--adopting a white, Anglo-Saxon, Protestant worldview. 3) split self syndrome--characterized by polarizing oneself into "good" & "bad" components; 4) reduce one's ability to recognize &/or use internal & external resources.

consequences of oppression (Landrum & Batts, 1985)

a process in which a human services professional assists a consultee with a work-related (or a caretaking related) problem with a client system in some specific way

consultation

goal is to help administrative level personnel improve their professional functioning so they can be more effective in the future with regard to program development, implementation, & evaluation

consultee centered administrative consultation

A licensed psychologist with epertise in the areas of mental retardation and learning disabilities is hired by a school district to assist a teacher who is having difficulty in working effectively with newly mainstreamed students in his class. The psychologist will work with the teacher an dhave no direct contact with the students. This is a type of: a) client-centered case consultation b) consultee-centrered case consultation c) program-centered case consultation d) consultee-cenetered admin consultation

consultee-centered case consult

To determine the degree of assoc btwn 2 variables when both are nominal you should use? a) contingency b) biserial c) phi d) Spearman

contingency

An elderly man has a stroke involving the middle cerebral artery. His symptoms will most likely include: a) memory impairment and cortical blindness with denial of the disability b) contralateral weakness and sensory loss in the leg, amnesia, and personality change c) contralateral weakness and sensory loss in the arm and face, aphasia, and visual disturbances d) visual agnosia and alexia without aphasia

contralateral weakness and sensory loss in the arm and face, aphasia, and visual disturbances

Kobasa: The ____ was defined as a tendency to believe and act as if one can influence the events taking place around oneself through one's own effort.

control disposition

Your research study involves assessing the effects of 2 IVs on 3DVs. You conduct a MANOVA rather than separate factoral ANOVAs to: a) max exptl variance b) control extraneous variable c) increase statistical power d) control experimentwise error rate

control experimentwise error rate-lower probability of making a type I error

The Papez circuit is involved in which? a) control of emotions b) control of circadian rhythms c) emergence of the secondary sex characeristics d) transfer of visual input to the visual cortex

control of emotions

Holland: Bookkeeper is associated with the ____ theme, which is similar to the realistic theme.

conventional

Someone who prefers unstructured situations that permit a high degree of self-expression and who is sensitive, emotional and indep is likely to score loweson which of Holland's occupational themes? a) realistic b) investigative c) social d) conventional

conventional

Kohlberg's levels of moral development: good boy/nice girl orientation and law and order orientation

conventional morality

To assess a measure's _____ the measure is administered to a group of examinees along with other measures that assess the same and different traits and scores on all measures are correlated.

convergent and discriminant validity,

High scores on MMPI scales 1 and 3 and low scores on scale 2 indicate which disorder? a) malingering b) conversion disorder c) psychosis d) depression

conversion disorder

Which non-intentional disorder is charcterised by symptoms that affect voluntary motor or sensory function than suggest neuro impairment and are caused/maintained by primary/secondary gain? a) somatization disorder b) factitious disorder c) malingering d) conversion disorder

conversion disorder

To reduce a 5 year old's nighttime fear of the dark, the most effect a) coping self-statements and positive imagery. b) in vivo exposure with response prevention. c) stress inoculation. d) systematic desensitization

coping self-statement and positive imagery

Sensate focus involves pairing pleasurable sensations with performance anxiety in order to reduce the latter. In other words, two incompatible responses are paired so that one of the responses will be eliminated. This is a technique called _________.

counterconditioning

When pleading not guilty by reason of insanity, a person relinquishes his or her legal privilege. Therefore, the psychologist should produce the records requested by the ______. Except when privilege has been waived by a client (e.g., when he/she is invoking the insanity defense), a psychologist should first assert the privilege when a client's records are subpoenaed by the court.

court

Which treatment is a type of aversive conditioning conducted in the imagination? a) overcorrection b) implosive therapy c) covert sensitization d) PST

covert sensitisation

In the DSM, substance dependence is characterised by all of these except: a) craving b) tolerance c) withdrawal d) efforts to cut down

craving-not diagnositic criteria

All of these are DSM criteria for diagnosis of substance dependence EXCEPT: a) withdrawal b) tolerance c) craving d) reduced activities

cravings

According to Sue and Zane, cultural knowledge and techniques are useful only insofar as they contribute to the therapist's _________.

credibility

The development of binocular vision is viewed by experts as having a ____ period. Specifically, if an infant is not able to coordinate both eyes by age one, he/she will always have problems with binocular vision; and, if the problem is not corrected by age 5, binocular vision will never develop.

critical

When using a behavioural-anchored rating scale (BARS), scale points are anchored with: a) empirically-derived weights b) job dimensions c) criterion outcomes d) critical incidents

critical incidents

Early ________ studies on the relationship between age and IQ suggested that IQ scores begin to drop in late adolescence or early adulthood. Subsequent _______ studies found, however, that this result was due more to methodology (cohort effects) than to actual IQ score declines.

cross-sectional longitudinal (and cross-sequential)

therapists exhibit when they 1) define everyone's reality according to their own cultural assumptions & stereotypes; 2) disregard cultural differences; 3) ignore evidence that disconfirms their beliefs; 4) rely on techniques & strategies to solve problems; & 5) disregard their own cultural biases.

cultural encapsulation (wren, 1985)

a healthy reaction to racism. shown when a client does not disclose to a white therapist b/c of fear of being hurt or misunderstood.

cultural paranoia

A researcher designs a study to test Vgotsky's theory of cognitive dvpt. Assuming that Vygotsky is right, the researcher will find that cognitive dvpt is most affected by: a) heredity b) culture c) ext reinforcement d) intrinsic motivation

culture

is a genetically-transmitted disorder that affects exocrine gland functioning. a. myasthenia gravis. b. cerebral palsy. c. cystic fibrosis. d. Parkinson's disease.

cystic fibrosis

Research investigating the effects of maternal employment on cognitive development in children has revealed that such employment. a. has beneficial effects for middle-class girls and boys only. b. has beneficial effects for middle-class boys only. c. has statistically insignificant adverse effects on children from all social classes. d. has no substantial effects.

d

Research on sex differences in leadership style suggests that actual differences between male andfemale leaders are far less than what might be expected. The research has also shown that. a. males and females are evaluated similarly by group members regardless of their perceived leadership style. b. males are evaluated more favorably than females by group members regardless of their perceived leadership style. c. although males and females are evaluated similarly by group members when they are perceived to have an autocratic leadership style, males are evaluated more favorably than females when they are perceived to have a participative leadership style d. although males and females are evaluated similarly by group members when they are perceived to have a participative leadership style, males are evaluated more favorably than females when they are perceived to have an autocratic leadership style.

d

The problem-solving and decision-making abilities of a team of employees who have been working together for several months are beginning to decline. Which of the following would be least useful for improving the performance of this team. a. having members come up with solutions alone before discussing them with each other. b. dividing their tasks into subtasks and having subgroups work on those tasks they are best suited for. c. encouraging members of the team to express more dissent and to be more openly critical about the solutions expressed by other members. d. adding members to the group who are known to be low in dominance.

d

The psychoanalyst Adolph Stern provided the first organized clinical description of the borderline patient. Of the ten basic characteristics Stern delineated, which of the following did he consider to be the most primary? a. inhibited aggression. b. fixation. c. emotional dysregulation. d. narcissism.

d

When it comes to surveys, often _________ is not an issue. a) biased sampling b) qualitative design c) selection d) internal validity

d

Which characteristic does not affect the power of a statistical test? a) sample size b) alpha c) magnitude of the population difference d) mean square

d

Which of the following is most true about pregnancy prevention programs for adolescents? a. They are most effective when they are school-based and they contribute to reduced pregnancy by reducing sexual activity. b. They are most effective when they are school-based and they contribute to reduced pregnancy by increasing contraceptive use. c. They are most effective when they are community-based and they contribute to reduced pregnancy by reducing sexual activity. d. They are most effective when they are community-based and they contribute to reduced pregnancy by increasing contraceptive use.

d

KAgan related the basic temperament quality of inhibition (degree of approach or withdrawal in new situations) to: a) prenatal development b) intellectual capacity c) caregiver behaviors d) CNS reactivity

d) CNS reactivity

On the WAIS, the greatest age-related decline in scores is apparent among older adults on the Processing Speed Index which consists of the: a) Similarities, Matrix Reasoning, and Block Design subtests. b) Matrix Reasoning, Symbol Search, and Letter-Number Sequencing subtests. c) Letter-Number Reasoning, Cancellation, and Digit Span subtests. d) Symbol Search, Coding, and Cancellation subtests

d) Symbol Search, Coding, and Cancellation subtests

Multiple motor and vocal tics with age of onset at 20 and duration of 13months is most suggestive of: a) Chronic Motor/Vocal Tics b) Tourette''s Disorder c) Atypical Tic Disorder d) Tic Disorder NOS

d) Tic Disorder NOS

Which is a contingency model of leaderships that distinguishes btwn 5 levels of participative leadership? a) Fiedler's contingency theory b) House's path-goal theory c) Hersey and Blanchard's situational leadership model d) Vroom, Yetton, and Jago's normative model

d) Vroom, Yetton, and Jago's normative model

Patients who have recently starting taking naltrexone (Revia) as a treatment for alcohol dependence ar emost likely to experience which of the following drug side effects? a) dizziness,‭ ‬ataxia,‭ ‬visual disturbances,‭ ‬nausea,‭ ‬and rash b) blurred vision,‭ ‬sexual dysfunction,‭ ‬weight gain,‭ ‬edema,‭ ‬and tremor c) shortness of breath,‭ ‬increased dreaming,‭ ‬nausea,‭ ‬diarrhea,‭ ‬and bradycardia d) abdominal cramping,‭ ‬nausea,‭ ‬insomnia,‭ ‬nervousness,‭ ‬and headache

d) abdominal cramping, nausea, insomnia, nervousness and headache

ADHD has been linked to all of these except: a) lower than normal level of activity in the basal ganglia b) abnormal levels of dopamine c) abnormal levels of serotonin d) abnormalities in the primary motor cortex

d) abnormalities in the primary motor cortex

Nicotine enhances alertness and memory by mimicking _________ at nicotinic receptor sites: a) endorphins b) GABA c) serotonin d) acetylcholine

d) acetylcholine

Helms- the reintegration stage is characterised by: a) a realization that whites have a responsibility for racism b) adoption of a "culture-blind" perspective c) embracing white identity while rejecting racist views of minorities d) adopting a belief in white superiority and minority inferiority

d) adopting a belief in white superiority and minority inferiority

In DSM, withdrawl delirium is assoc with: a) alcohol only b) alcohol; amphetamines; opioids c) alcohol; opioids d) alcohol; sedatives, hypnotics, and anxiolytics

d) alcohol; sedatives, hypnotics, and anxiolytics

Infant tests administered prior to 18months: a) have moderate to moderately high correlations with IQ tests administered during childhood b) are most valid for predicting future IQ for infants who score in the mid-range c) are most valid for predicting future IQ for infants who score in the superior (gifted) range d) are most valid for predicting future IQ for infants who score in the low range

d) are most valid for predicting future IQ for infants who score in the low range

In your 1st therapy session with a new client it's clear that she's exp a crisis that requires immediate attention. With regard to obtaining informed consent, the best strategy is to: a) explain to the client that you are ethically obligated to obtain an informed consent before proceeding but, if necessary, obtain a verbal (rather than signed written) consent. b) allow the client to vent her feelings before beginning the informed consent process. c) determine if the client is capable of giving a truly informed consent before attempting to do so. d) attempt to obtain the client's assent before proceeding and postpone obtaining an informed consent until the crisis has been addressed.

d) attempt to obtain the client's assent before proceeding and postpone obtaining an informed consent until the crisis has been addressed.

Echoic is to iconic as: a) procedural is to declarative. b) temporary is to permanent. c) external is to internal. d) auditory is to visual

d) auditory is to visual

Bandura's bobo doll expt found that: a) boys (but not girls) imitate an aggressive model. b) boys and girls imitate an aggressive model only when the model has been reinforced for acting aggressively. c) boys and girls imitate an aggressive model only when they are told they will receive a reward for doing so. d) boys and girls imitate an aggressive model even when there is no reinforcement for them or for the model for doing so

d) boys and girls imitate an aggressive model even when there is no reinforcement for them or for the model for doing so

The lack of a relationship btwn a student's confidence about her ability to succeed on a task and her actual success on the task reflects probs related to: a) means-end analysis b) equilibration c) resolution d) calibration

d) calibration

3 overlapping stages of stress innoculation training: a) formulation, problem focus, and termination. b) self-monitoring, self-evaluation, and self-reinforcement. c) cognitive modeling, overt instruction, and covert instruction. d) conceptualization, skills acquisition, and application.

d) conceptualization, skills acquisition, and application.

You work in a correctional facility and are asked to eval a prisoner to determine his eligibility for parole. You should: a) conduct the evaluation as requested since it is part of your job b) conduct the evaluation after reminding the prisoner that anything he says can be shared with prison authorities c) refuse to conduct the evaluation unless it has been court-ordered d) conduct the evaluation only if you believe it will serve a useful dispositional function

d) conduct the evaluation only if you believe it will serve a useful dispositional function

In psychodynamic theory when ego cannot give into id's needs b/c of pressure from superego or reality a)defense mech b)repression c)denial d) conflict

d) conflict

On the MMPI, which disorder is indicated by high scores on scales 1 and 3 and low score on scale 2? a) somatization disorder b) factitious disorder c) malingering d) conversion disorder

d) conversion disorder

A psychologist who has been seeing a prison inmate in therapy is asked to evaluate him to determine his readiness for parole. She should: a) agree to do so since she is already familiar with the inmate's readiness for parole b) agree to do so only if the inmate agrees and signs a waiver of confidentiality c) agree to do so only if she has the expertise necessary to make an informed parole recommendation d) decline to conduct the evaluation because doing so would constitute a multiple relationship

d) decline to conduct the evaluation because doing so would constitute a multiple relationship

Imipramine is approved to treat: a) schizophrenia and impulse control disorders b) schizophrenia and mania c) depression and panic disorder d) depression and childhood enuresis

d) depression and childhood enuresis

Whilst developing a training programme, you evaluate its effects on trainee satisf and learning to determine what changes in the programme are needed to achieve max effectiveness. This is an example of: a) needs assessment b) functional analysis c) summative evaluation d) formative evaluation

d) formative evaluation

patient lies on couch & attends to all thoughts & reports them without suppressing or censuring a) transference b)secondary process c)primary process d)free association

d) free association

Stats on the risk of suicide among adolescents from single-parent and dual-parent families show that: a) there is no relationship between family configuration and risk for suicide b) girls (but not boys) from single-parent families are at higher risk than girls from dual-parent families c) boys (but not girls) from single-parent families are at higher risk than boys from dual-parent families d) girls and boys from single-parent families are both at higher risk than girls and boys from dual-parent families

d) girls and boys from single-parent families are both at higher risk than girls and boys from dual-parent families

In elementary school, ability tracking: a) has beneficial effects on the achievement of high, moderate, and low achieving students b) has no effect on the achievement of high, moderate, or low achieving students c) has detrimental effects on the achievement of high, moderate, and low achieving students d) has different effects on achievement for students of different ability levels

d) has different effects on achievement for students of different ability levels (ability tracking=assigning students to groups based on ability levels)

A four-day workweek has been found to: a) increases productivity and has a positive effect on family life. b) increases productivity and has a negative effect on family life. c) decreases productivity and has a negative effect on family life. d) has little effect on productivity and a positive effect on family life.

d) has little effect on productivity and a positive effect on family life.

Research on goal-setting theory suggests that monetary incentives: a) are irrelevant to goal achievement as long as the goals have been accepted by employees b) are irrelevant to goal achievement as long as the goals have been self-set by employees c) have a positive effect only if employees participate in determining the difficulty of the goals d) have a positive effect whether or not employees have been involved in goal-setting

d) have a positive effect whether or not employees have been involved in goal-setting

Cluster analysis is used to: a) assess the length of time to the occurrence of a critical event b) evaluate the causal (predictive) influences of multiple latent factors c) determine, for a sample of people, how many types the sample represents d) identify natural grouping among a collection of observations

d) identify natural grouping among a collection of observations

A 9 year old with panic disorder: a) has been misdiag b/c panic disorder does not occur in preadolescent kids b) is most likely to manifest disorder as crying, freezing and clinging to parents c) most likely to manifest disorder as shortness of breath, chest pain, and heart palpitations d) is most likely to manifest disorder as shortness of breath, chest pain, tachycardia, and school refusal

d) is most likely to manifest disorder as shortness of breath, chest pain, tachycardia, and school refusal

Which is not true about specific phobia, blood-injection injury type? a) its onset is usually during childhood b) it has a strong familial pattern c) it is associated with fainting in reaction to the feared stimulus or event d) it is more common among males than females

d) it is more common among males than females

A disadvantage of the forced-distribution method of rating employees is: a) it is highly susceptible to leniency and central tendency rating biases. b) it makes it difficult to identify employees with very high or low levels of performance. c) it forces the rater to rate an employee in terms of what he/she is likely to do rather than what he/she actually does. d) it may lead to the identification of some above average workers as average or poor performers.

d) it may lead to the identification of some above average workers as average or poor performers. --most workers may be above avg

As a first step in therapy, a family therapist using the techniques described by Minuchin makes individual contact with each family member and adopts the mood communication style and behav of the family members. This is: a) family sculpting b) the therapeutic double-bind c) enactment d) joining

d) joining

A psychologist conducts a custody eval for a divorce proceeding. In terms of making a specific recommendation about custody, they psychologist should: a) avoid making a recommendation and, instead, leave the final decision to the court b) make a recommendation only if the assessment has involved procedures that have been validated for this purpose c) make a recommendation only if he/she was not retained by one of the parents but was retained by the court d) make a recommendation only if it is based on adequate psychological data and consideration of the best interests of the child

d) make a recommendation only if it is based on adequate psychological data and consideration of the best interests of the child

An agonist works by: a) blocking a neurotransmitter's access to the site b) exerting the opposite effect of a neurotransmitter. c) facilitating the action of a neurotransmitter at the site. d) mimicking the effect of a neurotransmitter.

d) mimicking the effect of a neurotransmitter

Deliberately repeating an undesirable behav is an integral part of: a) response cost b) implosive therapy c) contingency management d) negative practice

d) negative practice

Prior to age 5, signs of moderate MR include: a) minimal capacity for functioning in sensorimotor areas b) some slowness in learning to walk and talk c) marked delays in motor development with little or no communication skills d) noticeable delays in motor development

d) noticeable delays in motor development

Which PD is characterised by perfectionism? a) avoidant b) dependent c) histrionic d) obsessive-compulsive

d) obsessive-compulsive

All of these are required for diagnosis of Autism except: a) Impaired social interactions b) impaired communication c) restricted/repetitive behav d) onset prior to age 5

d) onset prior to age 5-onset should be prior to age3

"I have 2 feets" and "I runned fast" are examples of the grammatical error known as: a) telegraphic speech. b) holophrastic speech. c) overextension. d) overregularization.

d) overregularization.

Dr Dither is hired by the court to conduct an eval of a family as part of a child protection matter. Based on his eval and other evidence, the court determines that the child can remaing at home with his family who will be provided with supervision and counseling. 6months later, Dr Dither is contacted by the mother who says she would like to begin therapy with him for issues unrelated to the child protection matter. According to APA, Dr Dither should: a) agree to see the woman in therapy since the child protection matter has been resolved b) agree to see the woman in therapy only if her problem is, in fact, unrelated to the child protection matter c) agree to see the women in therapy since his familiarity with her situation will be an advantage d) refer the woman to a colleague

d) refer the woman to a colleague

In psychometrics, "banding" is a method for: a) increasing reliability b) reducing false positives c) improving incremental validity d) remedying adverse impact

d) remedying adverse impact (test scores within a certain range are treated as equivalent)

Rhett's disorder is characterised by all of these except: a) very small heads b) loss of hand skills c) loss of social interest d) repetitive movements

d) repetitive movements

Charging telephone customers for using directory assistance to obtain phone numbers in order to decr the usage of directory assistance is an application of which? a) Premack Principle b) overcorrection c) negative reinforcement d) response cost

d) response cost-type of negative punishment that involves removing a stiumulus (ie. money) following a behav to decrease that behav

The mother of a 35 year old man says that for 5 weeks he has seemed perplexed, nervous and his tone of voice varies btwn flat and shrill. His speech is sometimes like a "word salad" and he seems to be listening to voices/sounds that she cannot hear. The best diagnosis is: a) schizophrenia, disorganized type b) schizoid personality disorder c) schizoaffective disorder d) schizophreniform disorder

d) schizophreniform disorder - similar to Sx but duration of less than 6 months

Krumboltz- career dvpt and choice are related primarily to: a) needs and drives b) early childhood experiences c) ego identity development d) social learning

d) social learning

A researcher would use which technique to evaluate the cause-and-effect or predictive relationships btwn measured variables and latent factors? a) cluster analysis b) multitrait-multimethod matrix c) discriminant function analysis d) structural equation modeling

d) structural equation modeling

A diagnosis of somatisation disorder includes all of these except: a) at least 4 pain symptoms b) two gastro symptoms c) one sexual symptom d) two pseudoneuro symptoms

d) two pseudoneuro symptoms-only one is required

Fielder's contingency theory of leadership- task-oriented leaders are most effective when the work situation is _________ with regard to the leader's sense of certainty, predictability and control over his employees a) moderate in favorableness b) very high in favorableness c) very low in favorableness d) very high or very low in favorableness

d) very high or very low in favorableness - low LPC leaders (task-oriented) most effective in extreme situations

helping patient see their neurotic behaviors: a) positive transference b) negative transference c) repetition compulsion d)confrontation

d)confrontation

a mature defense. transforms a libidinal urge into socially acceptable interests & activities: a)displacement b)projection c)fixation d)sublimation

d)sublimation

Generally speaking, as an objective test, the Bender-Gestalt is most useful for identifying which of the following: a) damage to the left hemisphere b) damage to the right hemisphere c) bilateral temporal lobe lesions d) diffuse brain damage

damage to the right hemisphere . The Bender-Gestalt tests visual and perceptual skills which are mediated by the right hemisphere (but not at a more detailed specificity)

Which type of memory is the most impaired in dementia? a) declarative b) implicit c) procedural d) episodic

declarative

Beck's cognitive therapy-the info processing of suicidal individuals is characterised by a high degree of hopelessness and: a) equivocation b) dichotomous thinking c) personalisation d) automatic thinking

dichotomous thinking

A 50 year old man with a frontal lobe tumor frequently makes lewd comments, enages in inappro sexual behav, and somets responds to events with unexpected outbursts. Other symptoms he is likely to exhibit: a) concreteness and perseveration. b) distractibility and emotional lability. c) sparse verbal output and lower extremity weakness. d) occasional hallucinations and a "dazed" feeling

distractibility and emotional lability.

Theories of Teidman & O'Hara are examples of _____________ approach to career choice and dvpt a) developmental b) trait-oriented c) reinforcement-based d) personality

dvptl (vocational identity theory)

Gerstmann's syndrome is characterised by: a) dysgraphia, finger agnosia, acalculia, and right-left disorientation b) responding to simple questions with absurd, approximate, or inappropriate answers c) strong oral tendencies, hypersexuality, lack of concentration, and alterations in appetite d) anterograde amnesia, confusion, and confabulation

dysgraphia, finger agnosia, acalculia, and right-left disorientation

Number of correct items on a test, number of calories consumed are examples of what measurement: a)interval b)ordinal c)ratio d) nominal e)ratio

e

This measurement scale has the properties of order and equal intervals as well as the property of an absolute zero point. a)interval b)ordinal c)ratio d) nominal e)ratio

e

A transactional leader is least likely to agree with which: a) employee behavior is goal directed and employees will act rationally to achieve their goals b) contingent rewards are effective for motivating good performance c) the best idea for a boss is to adopt a laissez-faire leadership style d) each employee will function best if he/she is viewed as an individual and is given personal attention

each employee will function best if he/she is viewed as an individual and is given personal attention (this is more consistent with transformational leaders)

Peer pressure peaks during: a) preadolescence(10-13) b) early adolescence (13-15) c) middle adolescence (15-17) d) late adolescence (17-19)

early adolescence

A researcher obtains a statistically significant F-ratio for the interaction between two independent variables. This means that:

effects on one variable are contingent on the levels of the second variable.

Theory about development of conscience (Kochanska and Knaack) suggest that __________ _________ (inhibitory) is an important precursor, and develops between ages 2 and 3.

effortful control note: becomes trait-like by 33 months

In principle component analysis the ______ is the total amount of variability accounted for by each component a) communality b) factor loading c) principal component d) eigenvalue

eigenvalue

The ring and little fingers of the hand represent one dermatome and are innervated by which of the following a) second cervical nerve b) either cervical nerve c) second thoracic nervice d) tenth thoracic nerve

either cervical nerve

The _____ is a model of attitude change and proposes that there are two routes of persuasion - the central and peripheral routes - which differ in terms of the degree to which a persuasive message is elaborated (processed) by the recipient.

elaboration likelihood model

____ identifies two routes to persuasion - central and peripheral

elaboration likelihood model

According to Craik and Lockart, ___________ rehearsal involves coding memories semantically (in terms of meaning) and is the deepest level of processing.

elaborative

The ________ ___________ principle predicts that recall is improved when the same information is available at encoding and at retrieval.

encoding specificity Note: This principle is supported by the fact that recognition memory is better than recall memory (in recognition tasks, there are cues that help. That is, there is greater overlap between the information in the test items and the information contained in the memory trace).

Resaerch suggests that combining work with family roles is most assoc with_______ in women a) stress-related illness b) reduced involvement in family roles c) enhanced self-esteem d) low job commitment

enhanced self-esteem

identifying consultee needs; contracting; & physically & psychologically entering the system. most common time for resistance. resistance can be minimized by publicly clarifying the nature of the consultant's services & by establishing a strong collaborative relationship between the consultant & the consultee at the onset of the consultation

entry

groups reduces intergroup hostility is called

equal status contact hypothesis.

Test Construction 41. Computer-adaptive testing will yield A more accurate results for high scorers on a test. B more accurate results for low scorers on a test. C more accurate results for examinees who score in the middle range of a test. D equally accurate results across all range of scores on a test.

equally accurate results across all range of scores on a test.

no matter where one enters the system, the same end result occurs for the family. it is the patterns of behavior that are crucial to systems therapists, not the topics or controversies

equifinality

According to Piaget, the source of motivation for cognitive dvpt is: a) social acceptance b) parental influence c) equilibration d) collective unconscious

equilibration

_______ involves a combination of assimilation and accommodation and is motivated by a drive for balance or order.

equilibration

Learning Theory and Behavior Therapy 72. Negative reinforcement is essential to: A escape conditioning. B punishment. C aversion conditioning. D phobias.

escape conditioning. The concept of negative reinforcement involves increasing the likelihood of a behavior through the elimination of an aversive stimulus (the negative part).

A third-grade teacher puts a student in the time out chair for 15mins when he's disruptive. After 10mins, he tells her that he is sorry and knows what he did was wrong and that he won't do it again. the teacher lets him rejoin the other students. laer, he disrupts the others again and is again sent to time-out for 15mins. This time he apologises and promises he won't misbehave again after only 8mins. The teacher lets him rejoin his classmates. In this situation, his expression remorse is contolled by: a) positive reinforcement b) avoidance conditioning c) escape conditioning d) higher-order conditioning

escape conditioning=negative reinforcement

Using a collection agency to collect delinquent fees from a former client is: a) unethical and illegal b) unethical but legal c) ethical under certain circumstances d) always ethical

ethical under certain circumstances

phenomena that reflect a universal or culture-general orientation. involves viewing people from different cultures as essentially the same.

etic orientation

When working with a Japanese client in therapy, during the 1st session you decide that your primary goals are to normalise the problems and instill a sense of hope. According to Sue & Zane, these goals are: a) examples of "giving" b) ways of establishing ascribed credibility c) ways of fostering "amae" d) expressions of cultural knowledge

examples of giving

Which of the following approaches in family therapy encourages the therapist to engage in therapeutic triangles with family members during the treatment process? a) extended family systems therapy b) communication/interaction family therapy c) adaptive family therapy d) strategic family therapy

extended family systems therapy

What is the critical component of a dismantling strategy? a) stimulus control b) reciprocal inhibition c) extinction d) gradual exposure

extinction-exposure to the CS without the US

Clozapine is sometimes preferable to chlorpromazine as treatment for Sx b/c it is less likely to produce: a) sedation b) extrapyramidal side effects c) anticholinergic side effects d) bone marrow suppression

extrapyramidal side effects (atypical antipsychotics rarely cause movement disorders)

Job applicants complaing that bio info they are required to provide is unrelated to their ability to do the job and argue that it's an invasion of their privacy. These concerns suggest that the questionnaire lacks: a) construct validity b) face validity c) social validity d) predictive validity

face validity

If the dominant response is the right one, then the presence of others will ____ performance. When the dominant response is incorrect, the presence of others results in ________

facilitate, social inhibition

Which disorder is characterised by intentionally producing or faking symptoms in order to adopt a sick role? a) somatization disorder b) factitious disorder c) malingering d) conversion disorder

factitious disorder

A major advantage of this is that it provides more thorough information about the relationships among variables by allowing an investigator to analyze the main effects of each independent variable as well as the interaction between independent variables:

factorial design

When a study includes two or more independent variables, it is called a a)main effect b)interaction c) multi-effect d)factorial design

factorial design

Agranulocytosis is a potential side effect of clozapine, carbamazepine and other psychiatric drugs. Early symptoms include: a) fever, sore throat, mouth ulcers and lethargy b) nausea, diarrhea, vomiting c) sweating, palpitations, headache, termulousness d) constricted pupils, decreased visual acuity, sweating, constipation

fever, sore throat, mouth ulcers and lethargy

Although traumatic brain injury can produce diffuse brain injury, the _____are the areas of the cortex that are usually most adversely affected.

frontal and temporal lobes

A number of neuropsychologists interested in constructive memory have attempted to identify areas of the brain that are involved in false recollections. Their research has most consistently linked false recollection to lesions in the: a) frontal lobe b) temporal lobe c) occipital lobe d) parietal lobe

frontal lobe

an unhealthy condition that itself is an illness. African American unwilling to disclose to any therapist regardless of race or ethnicity due to general mistrust & suspicion.

functional paranoia

do not distinguish between ethnic and American cultures but have characteristics of both, is referred to as a ______ Phinney and Navarro found no evidence of this pattern in their sample.

fused identity.

A criterion measure can have ____ reliability but___ validity - i.e., it can give consistent results but measure only some aspects of the ultimate criterion.

high low

A _____ occurs when the effects of an independent variable differ at different levels of another independent variable:

interaction

There can't be an __ unless the study has at least two IVs and that when there is one, the _____ effects must be interpreted in light of the ___

interaction, main, interaction

____ which proposes that memory loss is due to inhinderence (i.e., stimuli encountered during the period of activity hinders previously acquired information). a. interference. b. memory trace decay. c. retrieval cue. d. motivated forgetting.

interference

the APA model/Nicotine dependence model suggest that the largest contributor to relapse is: a) external cues b) internal cues c) relational cues d) lack of coping skills

internal cues-depression and anxiety

An anglo-american therapist with internal local of control and internal locus of responsib is most likely to exp probs when working with an african-american client who has: a) extrenal locus of control and internal locus respons b) extrenal locus control and external locus respons c) internal locus control and external locus respons d) internal locus control and internal locus of respons

internal locus control and external locus respons-will challenge therapist's authority and view therapist as part of hte establishment

Holland Technical writer is associated with the ___ theme, which is also similar to the realistic theme.

investigative

a 4 year old who hates beans gets upset when his mother cuts them into small piece b/c he thinks that he now has to eat more beans. A Piagetian would describe this as an example of which of the following? a) object concept b) transduction c) irreversibility d) decentration

irreversibility (contributes to a child's inability to conserve)

Matching a therapy client to his therapist in terms of ethnicity a) is more accurate predictor of treatment length than outcome b) is a more accurate predictor of treatment outcome than length c) is a good predictor of treatment length an doutcome across all groups d) is not a good predictor of treatment length and outcome regardless of ethnic group

is a more accurate predictor of treatment length than outcome

Which of the following are characteristic symps of Huntington's? a) resting tremor; slowed movement; rigidity; and impaired balance and coordination b) jerky, involuntary movements; facial grimaces; restlessness; and general weakness c) muscle weakness; impaired balance and coordination; paresthesias; and nystagmus d) twitching; cramping or stiffness of the muscles; muscle weakness; and slurred speech

jerky, involuntary movements; facial grimaces; restlessness; and general weakness

a _________ evaluation is an assessment of the worth of a particular job for the purpose of determining appropriate wages.

job

A ____ is performed to determine the skills, knowledge, and abilities required by a job. Once these have been identified, selection tests can be chosen or developed.

job analysis

A ____ is conducted specifically to determine the relative worth of jobs in order to set salaries and wages.

job evaluation

When a test has high sensitivity, this means there is: a) low chance of false negatives and high chance of false positives. b) low chance of false negatives and low chance of false positives. c) high chance of false negatives and high chance of false positives. d) high chance of false negatives and low chance of false positives.

low chance of false negatives and high chance false positives-so most people with the disorder will be identified as having the disorder but there will be some people without the disorder who will also be identified as having the disorder

49. The prevalence of mental disorders in people over 65 is: A higher than any other age group B lower than any other age group C higher, and includes cognitive impairment D The same as the under 65 population

lower than any other age group

The result of one independent variable on the dependent variable, disregarding the effects of all other independent variables.

main effect

An ____ effect is likely to modify or invalidate the conclusions made on the basis of the ____ effects alone. a) main, interaction b)interaction, main c) factorial, multiple d)multiple, factorial

main, interaction

situated midway between ethnic and American culture and are part of neither, describes _____. None of the adolescents in Phinney and Devich-Navarro's sample fit this pattern..a) blended biculturalism, b) alternating biculturalism, c) separated. d)marginalization

marginalization

A person with damage to which area of the frontal lobes is most likely to engage in akinesia, mutism an dweakness an dloss of sensation in lower extremities? a) medial frontal area b) anterior cingulate area c) dorsolateral prefronal area d) orbitofrontal area

medial frontal area

Compare medication management alone, behavioral treatment alone, combined med and behavioral, and routine community care in the tx of ADHD.

medication alone or combined medication/behavioral better than behavioral alone or routine community care

Fielder's contingency theory of leadership- high LPC leaders are most effective when the work situation is _________ with regard to the leader's sense of certainty, predictability and control over his employees a) moderate in favorableness b) very high in favorableness c) very low in favorableness d) very high or very low in favorableness

moderate in favorableness

The effectiveness of guided imagery in reducing anxiety is effected by the degree of imaging ability of the client. The imaging ability of the client is a __________ variable.

moderating

A ____ is a variable that alters the strength of the causal relationship. So for instance, psychotherapy may reduce depression more for men than for women, and so we would say that gender ____ the causal effect of psychotherapy (X) on depression (Y).

moderator variable, moderates

late during Piaget's Preoperational Stage children treat social rules as absolute and unchangeable

moral realism

The view that there is no absolute or universal moral law or truth, resulting in a morality determined by cultural factors or personal preference.

moral relativism

A sudden loud noise would elicit which reflex from a one month old infant? a) babkin b) darwinian c) moro d) babinski

moro

A researcher wants to compare the effects of 4 different prevention programs on willingness to use safe sex. Subjs are randomly assigned to 1 of 4 programs and 6 months later they indicate if they engaged in safe sex all of the time, some of the time, or never. Which test is best? a) one-way ANOVA b) two-way ANOVA c) multiple-sample chi-square d) chi-square goodness of fit

multiple sample chi square

When working with an Af-Am family, the best approach is to: a) focus on parent-child subsystems initially b) adopt a multisystems approach c) use a combination of individual, nuclear family, and extended family sessions d) rely on nondirective, nonstructured techniques

multisystems approach

The ______ is used to organize the data collected when assessing a measure's convergent and discriminant (divergent) validity. As its name implies, it involves administering different measures that assess different traits.

multitrait-multimethod matrix

Which of the following are characteristic symps of multiple sclerosis? a) resting tremor; slowed movement; rigidity; and impaired balance and coordination b) jerky, involuntary movements; facial grimaces; restlessness; and general weakness c) muscle weakness; impaired balance and coordination; paresthesias; and nystagmus d) twitching; cramping or stiffness of the muscles; muscle weakness; and slurred speech

muscle weakness; impaired balance and coordination; paresthesias; and nystagmus

A ____ is a first step in designing training programs.

needs assessment (needs analysis)

Escape conditioning is a type of _________ reinforcement.

negative

Parents give their son time-out each time he torments his little sister. They find that, over time, the child needs less and less time in the time-out to calm down and apologise. His behav is being controlled by: a) negative punishment b) negative reinforcement c) stimulus control d) stimulus fading

negative punishment

From the perspective of Freudian theory, a phobia is a manifestation of _____, which is aroused by a perception of danger from the ___.

neurotic anxiety, instincts

a disturbance in the boundary between the self & the external environment, which interferes with the person's ability to satisfy his/her needs & maintain homeostasis.

neurotic behavior stems from...

Score transformations that alter the rank order and relative size of the distance btwn scores are referred to as: ___________ transformations a) llinear b) nonlinear c) leptokurtic d) platykurtic

nonlinear

If the harassment is _____and the person stops when asked, it is unlikely that the act would be found to constitute sexual harassment.

not severe

A 50 year old man with a temporal lobe tumor is likely to exhibit: a) concreteness and perseveration. b) distractibility and emotional lability. c) sparse verbal output and lower extremity weakness. d) occasional hallucinations and a "dazed" feeling

occasional hallucinations and a "dazed" feeling

The _________ is to conscious perception of odors are the __________ is to olfactory memory a) insular cortex; septum b) orbitofrontal cortex; amygdala c) somatosensory cortex; thalamus d) cingulate cortex; medulla

orbitofrontal cortex; amygdala

The _____ region is involved in inhibition. Damage to this area may produce disinhibition, impaired social insight, and inappropriate affect.

orbitofrontal region

A score on which of the Rorschach variables is most useful for estimating his level of intelligence? a) organizational activity b) perceptual thinking c) WSum6 d) S-CON

organisational activity

An ____ (as opposed to a manipulated variable), is one that can be measured but cannot be manipulated or controlled by the researcher. For example, in a study assessing the relationship between gender and beer consumption, both variables are ___ since the researcher cannot control either the gender or the beer consumption of the subjects.

organismic variable, organismic

Learning Theory and Behavior Therapy 34. The discrimination hypothesis, frustration theory, and sequential theory are alternative explanations for the reason why A organisms exhibit experimental neurosis when presented with difficult discriminations during classical conditioning trials. B organisms do not respond to punishment when it is initially presented at low intensity and then gradually increased in intensity. C organisms respond for a longer period of time during extinction trials when they were previously reinforced on an intermittent schedule than when they were reinforced on a continuous schedule. D organisms often exhibit a temporary increase in one reinforced behavior when reinforcement for another previously-reinforced behavior is withdrawn.

organisms respond for a longer period of time during extinction trials when they were previously reinforced on an intermittent schedule than when they were reinforced on a continuous schedule. Now you know, though -- these theories are all explanations for the "partial reinforcement effect."

Maslow's five needs, in order, are:

physiological, safety, social, esteem, and self-actualization.

First level of Piaget morality states that people will do something if the consequences for it are happy

premoral

Piaget proposed a sequence of moral development involving three stages:

premoral, moral realism, and moral relativism.

An assumption underlying the notion of groupthink is that: a) pressures toward uniformity limit task effectiveness b) increasing cohesiveness results in more extreme decisions c) excessive pressures toward conformity produce reactance d) lack of familiarity with task demands leads to manitenance of the status quo

pressures toward uniformity limit task effectiveness -decrease in a willingness to consider divergent points of view

Flu-like symptoms are characteristic of which stage in HIV.

primary HIV infection stage (initial stage)

The three stages of HIV infection (at least according to many experts) are: ______

primary HIV infection, clinically asymptomatic stage, and symptomatic HIV infection.

reach people in an acute state of stress & provide them with enough support to prevent them from becoming the "chronic's of the future."

primary goal of crisis intervention (Phares, 1988)

criterion-referenced scores

provide no information about how an individual's score compares with others' scores e.g., percent correct and raw score

When working with older patients, a therapist should keep in mind all EXCEPT: a) older people are more different from one another than younger people on a variety of measures b) "slowing down" is a predictable correlate of the normal aging process c) the correlation between physiological and psychological characteristics increases with age d) psychotherapy, especially insight-oriented psychotherapy, is relatively ineffective with the geriatric population

psychotherapy, especially insight-oriented psychotherapy, is relatively ineffective with the geriatric population

common assumption in both parametric and nonparametric test

random select

A latino client suffering from depression is most likely to complain of: a) being heratbroken b) recurrent headaches c) exp an imbalance d) feelings of guilt

recurrent headaches

studies investigating the effects of culturally responsive counseling suggest that it: a) reduces premature termination but has little effect on depth of self-disclosure or satisfaction with counseling b) enhances depth of self-disclosure but has little effect on premature termination or satisfaction with counseling c) reduces premature termination and enhances depth of self-disclosure and satisfaction with counseling d) has little effect on premature termination, depth of self-disclosure, or satisfaction with counseling

reduces premature termination and enhances depth of self-disclosure and satisfaction with counseling

single group very depressed subjects- treated 12 sessions with CBT. greatest threat to validity

regression to the mean-b/c subjects initial score likely to be in extreme range. when retested subjects make look like they improved when in reality is simply result of regressions

Krumboltz (1979) theory is an example of _____________ approach to career choice and dvpt a) developmental b) trait-oriented c) reinforcement-based d) personality

reinforcement-based

Which term is least descriptive of Lockean tradition underlying the Western approach to individual psychotherapy: a) linear-cause effect b) reductionistic c) individualistic d) relativistic

relativistism is characteristic of non-western approaches. Lockean tradition reflects an absolutist approach

Stage 2 of dementia is characterised by all of these except: a) retrograde amnesia b) rigidity c) flat mood d) ideomotor apraxia

rigidity-doesn't occur until stage 3 ; (Ideomotor apraxia is the inability to translate idea into movement)`

As defined by Lazarus's cog appraisal theor, __________ refers to a person's evaluation of the resources he/she has to cope with a situation that has been ident asstressful: a) primary appraisal b) secondary appraisal c) tertiary appraisal d) re-appraisal

secondary appraisal

A previous client calls to request an appt b/c he has become v depressed and is feleing suicidal. In the past few months you have been reducing your practice for retirement. You should: a) see the man until the crisis has passes and then give him appro referrals b) explain to the man that you are about to retire and refer him to a colleague c) provide the man with several referrals and ask him to let you know if non work out b) provide the man with several referrals and tell him you will consult with the new therapist

see the man until the crisis has passed and then provide with appro referrals

Damage to the postcentral gyrus is most likely to impair which of the following: a) sense of touch b) motor coordination c) vision d) executive cognitive functions

sense of touch

A ____ period is longer in duration and more flexible than a ____ period. Language, for example, is believed to have a ___period: Once the optimal period for being exposed to language has passed, the individual may still be able to develop language skills (although it may be more difficult to do so).

sensitive, critical, sensitive

A predictor's _____ refers to the proportion of individuals in the validation sample who have the characteristic measured by the predictor and were accurately identified by the predictor as having that characteristic. It provides an index of the predictor's ability to identify true positives. It is calculated by dividing the number of true positives by the number of true positives plus false negatives. a) positive predictive value b) negative predictive value c) sensitivity d) specificity

sensitivity

A psychologist dvps a diagnostic test to identify people who have injection phobia. In this situation, the test's _________ refers to how good it is at identifying people who have injection phobia from the pool of people who do actually have it a) specificity b) sensitivity c) positive predictive value d) negative predictive value

sensitivity

What are the 4 stages of Troiden's gay/lesbian identity development model?

sensitization identity confusion identity assumption identity integration

Aspect of memory capable of storing a verylarge amount of info for a very brief period of time is_____ a) short term memory b) sensory c) primary d) secondary

sensory

The corpus collosum is responsible for transfer of _________ information between brain hemispheres

sensory (and some motor)

The _______ _________ effect refers to the tendency to recall items in the beginning and end of a list better than items in the middle of the list, especially when recall occurs immediately after rehearsing the list.

serial position Note: One explanation for the serial position effect is that items in the beginning of the list have been stored in long-term memory, while those in the end of the list are still present in short-term memory.

Which systems are linked to social phobias? a) serotonergic only b) serotonergic dopaminergic c) cholinergice only d) cholinergic and glutamatergic

serotonergic dopaminergic

McClelland described _____ as power that is used to benefit others and proposed that it contributes to management effectiveness by helping managers achieve organizational goals through the actions of subordinates.

socialized power

Which non-intentional disorder is characterised by at least 8 physical symptoms? a) somatization disorder b) factitious disorder c) malingering d) conversion disorder

somatisation disorder

Touch is controlled by the _____, which is located in the ______.

somatosensory cortex, parietal lobe

standard error of the mean formula

standard error = population standard deviation divided by the square root of sample size

test's reliability coefficient = 0

standard error of measurement equal to the standard deviation of the test score

Contrast the state licensing board's roles vs APA's roles in monitoring psychologists.

state licensing board - entry level competence APA - monitor ethical principles

inherent power differential, attempts to minimize by promoting "power with" via appropriate self-disclosures; demystifying the therapy process; & encouraging clients to set their own goals & evaluate the progress of therapy

striving for an egalitarian Relationship

Kluver-Bucy syndrome is characterised by: a) dysgraphia, finger agnosia, acalculia, and right-left disorientation b) responding to simple questions with absurd, approximate, or inappropriate answers c) strong oral tendencies, hypersexuality, lack of concentration, and alterations in appetite d) anterograde amnesia, confusion, and confabulation

strong oral tendencies, hypersexuality, lack of concentration, and alterations in appetite

Which of the following is not true about LD? a) children with LD usually cont to exp problems into adolescence and adulthood b) children with LD have avg to above avg intelligence c) studies have ruled out genetic components in etiology of these disorders d) these disorders are about 2-4x more common in males than females

studies have NOT ruled out genetic components in etiology of these disorders

1) relieve symptoms; 2) restore person to his/her previous level of functioning; 3) id factors that precipitated the crisis; 4) id & apply remedial measures; 5) connect current stresses with past experiences; 6) develop adaptive coping skills to be used in the future.

subgoals of crisis intervention (Rapoport, 1970)

Underlying the technique known as stress inoculatoin is the assumption that : a) exposure to stressful situations extinguishes the stress reaction to habituation b) exposure to stressful situations leads to catharsis which results in reduced reactivity c) successful coping with stress strengthens the ability to cope with stress in the future d) expecting exposure to stressful stimuli reduces their impact

successful coping with stress strengthens the ability to cope with stress in the future

For patients who develop tardive dyskinesia as a result of long-term neurolepic use: a) symptoms are always irreversible b) symptoms are alleviated with a dopamine agonist in most cases c) symptoms may eventually improve to some degree following neuroleptic withdrawal d) symptoms worsen over time following neuroleptic withdrawal even when withdrawal is gradual

symptoms may eventually improve to some degree following neuroleptic withdrawal

Probably the best way to reduce aggressioin kids is to: a) have them engage in strenuous physical activity b) build empathy toward potential victims c) teach alternative, nonaggressive responses d) model prosocial behaviors

teach alternative, nonaggressive responses

Dave D has OCD and he has just started invivo exposure. He reports that he was able o avoid engaging in rituals for 3 days but then started checking again on the 4th day. The therapist will most likely: a) tell Dave to have his wife check the doors and windows for him until the next therapy session. b) tell Dave to check the doors and windows for only 15 minutes each day until the next therapy session. c) tell Dave that lapses are to be expected and remind him of the importance of striving for complete abstinence from rituals. d) tell Dave that lapses are to be expected and instruct him to check the doors and windows for only 30 minutes each day for the next two days, 15 minutes each day for the following two days, and then abstain from all checking the following two days.

tell Dave that lapses are to be expected and remind him of the importance of striving for complete abstinence from rituals.

The _____ are involved in learning and memory, and damage to this along with certain adjacent structures can produce amnesia.

temporal lobes

______ is to prevent a problem from becoming chronic or to prevent relapse.

tertiary preventions

partial correlation

test for relationship b/w two variables while controlling for a third

Symbolic racism is rooted in: a) authoritarianism. b) realistic self-interest. c) the belief that members of minority groups violate traditional American values. d) limited social contact with members of minority groups or contact only under aversive conditions.

the belief that members of minority groups violate traditional American values.

If a 4 year olf child lies to his parents: a) the parents should not be alarmed since, at this age, children do not know the difference between lying and truth-telling. b) the child is doing so in order to avoid punishment since, at this age, that is the only reason why children lie. c) the child is probably exhibiting normal behavior since, at this age, children lie for a number of reasons. d) the parents should be concerned since, at this age, lying is very rare and is usually an early sign of pathology.

the child is probably exhibiting normal behavior since, at this age, children lie for a number of reasons.

In Gestalt, an optimal contact boundary is semipermeable and coterminous with the actual self. Introjection is one type of boundary disturbance and occurs when: a) the contact boundary is located far in the environment b) the contact boundary is located deep within the self c) the contact boundary is turned back against the self d) the contact boundary is extremely rigid

the contact bounary is located deep within hte self

Studies on concurrent schedules of reinf have found that an organism's relative freq of responding to one alternative corresponds to the relative frew of reinf for responses to that alternative. This is: a) the law of effect b) Rescorla-Wagner theory c) the Premack principle d) the matching law

the matching law-proportion of responses matches proportion of reinf

Structural family therapy entails 3 overlappin steps/stages. Which of the followng techniques is/are most useful during the 1st step: a) reframing and relabeling b) tracking and mimesis c) constructing a family map d) enactment

tracking (ident and using family's values) and mimesis (adopting family's affective and behav style)-used to join the family system. Joining is the initial step in structural family therapy

In his research, a cognitive psychologist uses paired-associate and serial learning tasks. Apparently, the psychologist is interested in: a) verbal learning b) selective attention c) procedural memory d) iconic memory

verbal learning

Research comparing the effects of the individual components of Panic Control Treatment (PCT) for Panic Disorder with Agoraphobia has found that, for alleviating the symptoms of Agoraphobia, in____ is the most effective component.

vivo exposure

When using flooding (in vivo) to treat a specific phobia, which intstrument would be most useful for monitoring the client's progress? a) SUDS b) SCID c) CAPS d) BDI

a) SUDS-client rates anx level during exposure

Sherif's social judgement theory implies that, before trying to persuage someone to your point of view, you should consider his: a) categories of judgment b) locus of control c) category-based expectancies d) level of self-monitoring

a) categories of judgment -ie. latitudes of acceptance, non-commitment and rejection--a person is most likely to be persuaded when the message is within his latitude of acceptance

The type of question used by family therapists to explore interconnectedness of family members and the recurrent patterns in their relationships is: a) circular b) lineal c) strategic d) reflexive

a) circular

In factor analysis the ______ is the proportion of a test's total variance that is accounted for by the common identified factors a) communality b) factor loading c) principal component d) eigenvalue

a) communality--total amount of variability in test scores that has been explained by the factor analysis

IQ tests are to measures of creativity as: a) convergent thinking is to divergent thinking b) divergent thinking is to convergent thinking c) disjunctive thinking is to conjunctive thinking d) conjunctive thinking is to disjunctive thinking

a) convergent thinking (deriving a single answer) is to divergent thinking (generate as many ideas as possible)

To determine the impact of a treatment on the patient's qual of life, you would conduct a: a) cost utility b) cost effectiveness c) cost analysis d) cost benefit

a) cost utility--involves attrib monetary values to diff qual of life indices

Qual assurance is least concerned with: a) cost-effectiveness of services b) availability of services c) client satisfaction with services d) the outcomes of services

a) cost-effectiveness of services =target of utilisation & reducing costs

Depr has been linked to: a) decreased REM latency and increased REM density. b) decreased REM latency and decreased REM density. c) increased REM latency and increased REM density. d) increased REM latency and decreased REM density.

a) decreased REM latency ( earlier onset of REM) and increased REM density (increased freq of rapid eye movements)

A test is administered to 200 examinees and the dstribrution of raw scores are converted to percentile ranks. The number of examinees who obtain % ranks btwn 91-95 will be: a) equal to the number of examinees who obtain percentile ranks between 51 and 55 b) less than the number of examinees who obtain percentile ranks between 51 and 55 c) more than the number of examinees who obtain percentile ranks between 51 and 55 d) cannot be determined

a) equal to the number of examinees who obtain percentile ranks between 51 and 55 (5% in each)

Which statement is most consistent with Taylor's principles of scientific mangement? a) Managers should adopt a "hands off" approach and allow workers to make decisions about how work should be accomplished. b) Managers and workers should assume responsibility for their own share of their work. c) Managers must constantly monitor workers to ensure that their work continues to meet organizational goals. d) Managers should adapt their leadership style to the motivation and ability level of each worker.

b) Managers and workers should assume responsibility for their own share of their work.--dividing work and responsibility btwn managers and workers-performing work each best suited for

_____seizures being in and are often restricted to the temporal lobe and involve impaired consciousness, and automatisms such as lip smaking, chewing, etc a) Grand mal b) Complex partial c) Myoclonic d) Absence

b) Complex partial --complex b/c they impaire consciousness and partial b/c begin in limited area of the brain

Research investigating the effectiveness of neurofeedback as a treatment for ADHD has generally found that: a)It is not an effective approach for most individuals with this disorder. b) It is an effective approach and exerts its strongest impact on symptoms of inattention and impulsivity. c) It is an effective approach and exerts its strongest impact on symptoms of hyperactivity. d) It is an effective approach only when used in conjunction with a central nervous system stimulant.

b) It is an effective approach and exerts its strongest impact on symptoms of inattention and impulsivity.

According to _________ , depression is attributable to deficits in self-monitoring, self-eval, and self-reinf: a) Bandura's self-efficacy model b) Rehm's self-control theory c) Seligman's learned helplessness model d) Swann's self-verification theory

b) Rehm's self-control theory -individs with depression selectively attend to negative events and to the immediate consequences of their behav; make inaccurate internal attribs about their behavs, and have low rates of self-reinf

Bateson et al identified dbl-bind communication as an etiological factor in which disorder: a) Anorexia b) Schizophrenia c) Antisocial Personality Disorder d) Narcissistic Personality Disorder

b) Schizophrenia (dbl-bind= when family member receives contradictory messages from another family member and is unable to comment on them/escape them-ie by responding with similarly distorted messages)

Which is an example of cultural encapsulation: a) a client belonging to a minority group cannot separate problems due to individual factors from those related to oppression b) a white male therapist doesn't recognize that he interprets clients' problems from his own cultural worldview c) a young woman's conflict is related to her inability to separate from the values, attitudes, and expectations of her immigrant parents d) a therapist tends to view all problems of minority clients as due to factors inherent to the client's cultural background

b) a white male therapist doesn't recognize that he interprets clients' problems from his own cultural worldview (encapsulated within his own dominant culture)

Research into SADs has found that psychotherapy is: a) substantially less effective than antidepressant drugs. b) an effective treatment and produces results comparable to those produced by antidepressant drugs. c) effective only when administered in conjunction with a low- to moderate-dose of an antidepressant drug. d) effective only for individuals who have atypical symptoms.

b) an effective treatment and produces results comparable to those produced by antidepressant drugs.

According to Sternberg's triarchic theory, intelligence consists of which 3 facets? a) general, broad, and specialized b) analytical, creative, and practical c) attention, processing, and planning d) general, fluid, and crystallized

b) analytical, creative, and practical

To determine the treatment that produces the best benefits at the least cost, you would conduct a: a) cost utility b) cost effectiveness c) cost analysis d) cost benefit

b) cost effectiveness

The WHO's study on Sx compared patients w/Sx from non-western dvping countries and those from western industrialised countries. This research found that patients from dvping and industrialised countries differ in terms of: a) age and gender ratios b) course and outcomes c) symptom profiles d) gender and prognosis

b) course and outcomes

To eval the impact of an educational program on each student's mastery of the info presented, which would be most useful: a) norm-referenced scores b) criterion-referenced scores c) standard scores d) ipsative scores

b) criterion-referenced scores --tells you how well the examinee did in absolute terms

The rate of Depr for married women a) is lower than the rates for unmarried women and unmarried men b) is lower than the rate for married men c) is higher than the rates for unmarried women and married men d) is the same as the rate for unmarried women

c) is higher than the rates for unmarried women and married men --married women at increased risk than single, divorced or widowed (opposite is true for men)

The correction for attenuation formula is useful for estimating a validity coeff when: a) sources of systematic error have been controlled b) sources of criterion contamination have not been adequately eliminated c) measurement error has affected the magnitude of the coefficient d) sampling error has apparently biased the magnitude of the coefficient

c) measurement error has affected the magnitude of the coefficient --The correction for attenuation formula is used to esitmate test's validity coeff when the reliability coeff for the predictor/criterion is increased to1.0 so that measurement error has been entirely removed so it is estimating the validity coeff when measure error has been eliminated

The MMSE is most often administered to: a) children as a measure of intelligence b) adolescents and adults as a measure of brain dysfunction c) older adults as a measure of cognitive functioning d) older adults as a measure of quality of life

c) older adults as a measure of cognitive functioning --often used to assess for dementia although it wasn't developed to do that

To determine the relative costs and benefits of a treatment, you would conduct a: a) cost utility b) cost effectiveness c) cost analysis d) cost benefit

d) cost benefit

An advantage of using treatment manuals in clinical practice is that they: a) ensure that the clinician incorporates effective nonspecific factors into an intervention. b) reduce the need for training and supervision in the selection and application of empirically supported treatments. c) make it easier for a clinician to adapt a treatment to the individual needs of a client. d) provide the clinician with concrete examples that illustrate the appropriate application of an intervention.

d) provide the clinician with concrete examples that illustrate the appropriate application of an intervention.

A Gestalt therapist would most likely be interested in which of the following: a) the client's sense of responsibility and needs for power, freedom, and belongingness b) the client's cognitive distortions (e.g., selective inattention, misperceptions) c) the nature of the client's ego state pathology d) the client's level of "response-ability"

d) the client's level of "response-ability" =ability to act, decide, choose and otherwise respond

Troiden's model of gay and lesbian identity dvpt describes it as an ongoing process involving 4 stages and predicts that: a) the onset of identity confusion is somewhat earlier for males but the onset of identity assumption and identity commitment is somewhat earlier for females b) the onset of identity confusion is somewhat earlier for females, but the onset of identity assumption and identity commitment is somewhat earlier for males c) the onset of identity confusion, identity assumption, and identity commitment is somewhat earlier for females d) the onset of identity confusion, identity assumption, and identity commitment is somewhat earlier for males

d) the onset of identity confusion, identity assumption, and identity commitment is somewhat earlier for males

Autocorrelation would be a problem when an investigator uses which research design? a) between group b) Latin square c) Solomon four-group d) time-series

d) time-series --involves non-independent observations--autocorrelation is a problem whenever obsv are not independent

Bernewas a co-founder of: a) Gestalt therapy b) reality therapy. c) solution-focused brief therapy. d) transactional analysis.

d) transactional analysis.

In factor analysis the ______ is the correlation btwn a single test and an identified factor a) communality b) factor loading c) principal component d) eigenvalue

factor loading

When calc the F-ratio for ANOVA, which provides a measure of both treatment effects and error? a) MST b) MSW c) MSB d) MSA

MST= mean square total= combination of MSB(variability due to treatment effects plus error) and MSW(variability that is only due to error)

When calc the F-ratio for ANOVA, which provides an estimate of variablitiy due to the effects of error only? a) MST b) MSW c) MSB d) MSA

MSW= mean squared within=pooled measure of variability within each treatmetn group. Provides an estimate of variablility due to error only

Hewig compared the gaze patterns of heteros men and women as they looked at pics of the opposite sex and found that: a) females and males both gazed at the face of a member of the opposite sex for the longest period of time. b) females gazed at the face of a male for the longest period of time, while males gazed at the breasts of a female for the longest period of time. c) females gazed at the hip/buttocks region of a male for the longest period of time, while males gazed at the face of a female for the longest period of time. d) females and males both gazed at the hip/buttocks region of a member of the opposite sex for the longest period of time.

a) females and males both gazed at the face of a member of the opposite sex for the longest period of time. --after an initial face scan, men looked earlier and longer at the female's breasts while women looked earlier and longer at male's legs

Test-retest reliab would be most appro for determining the reliab of a test if it was designed to assess: a) general intelligence b) mood c) state anxiety d) reading level

a) general intelligence -least likely to change over time

Children are most likely to be resilient when their parents are: a) high in warmth, structure, and expectations b) high in warmth and structure but low in expectations c) moderate in warmth, structure, and expectations d) moderate in warmth but low in structure and expectations

a) high in warmth, structure, and expectations =Authoritative parenting

The risk of suicide increases in adolescents when an affective disorder is accompanied by: a) is accompanied by ADHD, Conduct Disorder, or Substance Abuse b) is accompanied by ADHD, an Anxiety Disorder, or an Eating Disorder c) is accompanied by Substance Dependence or an Impulse Control Disorder d) involves "double depression

a) is accompanied by ADHD, Conduct Disorder, or Substance Abuse (Antisocial is also high)

Lawler's model of facet satisfaction : a) is similar to equity theory and assumes that job satisfaction is affected by comparisons of one's own inputs and outcomes to the inputs and outcomes of others b) is similar to need hierarchy theory and assumes that job satisfaction is related to the fulfillment of one's most prepotent needs c) is similar to two factor theory and views satisfaction and dissatisfaction as independent factors d) is similar to goal setting theory and proposes that participation in the setting of work objectives is crucial for job satisfaction

a) is similar to equity theory and assumes that job satisfaction is affected by comparisons of one's own inputs and outcomes to the inputs and outcomes of others

Dissociative amnesia is associated with ______ amnesia which occurs when the individual is able unable to recall all of the events that occurred during a defined period of time a) localized b) selective c) systematized d) transient

a) localized

From the perspec of the situational leadership model, the telling leadership style is most effective when an employee is : a) low in ability and low in willingness b) low in ability and high in willingness c) high in ability and low in willingness d) high in ability and high in willingness

a) low in ability and low in willingness

Among the groups listed, admission rates for psychiatric hospitalisation is highest for which: a) never married men b) married men c) divorced men d) widowers

a) never married men

A reality therapist would most likely be interested in which of the following: a) the client's sense of responsibility and needs for power, freedom, and belongingness b) the client's cognitive distortions (e.g., selective inattention, misperceptions) c) the nature of the client's ego state pathology d) the client's level of "response-ability"

a) the client's sense of responsibility and needs for power, freedom, and belongingness ---behav motivated by biological and psychological needs the goal of therapy is the ability to fulfill one's needs without depriving others of the ability to do the same

A new job selection test with a validity coeff of 0.3 will most likely improve decision-making accuracy when: a) the job applicant pool is very large b) the job applicant pool is very small c) the number of successful employees hired without the new test is very large d) the number of successful employees hired without the new test is very small

a) the job applicant pool is very large -a predictor will increase decision-making accuracy when there are many applicants

QWhich of the following is not one of the abilities that Piaget described as being charac of the concrete operational stage of dvpt? a) transductive reasoning b) reversibility c) transitive inference d) decentration

a) transductive reasoning =magical thinking-charac of the preoperational stage

A client reveals that e robbed a convenience store 6months ago. He expresses remorse and says that he notices you make notes while he's talking. He asks you not to keep any written record of the robbery. You should: a) agree to exclude information about the robbery from his file b) determine whether the robbery is clinically relevant before deciding whether or not to include information about it in his file c) inform the client that you are legally required to report the matter to the authorities before you actually do so d) reassure the client that anything he tells you is confidential and will not be revealed without his consent

b) determine whether the robbery is clinically relevant before deciding whether or not to include information about it in his file

Minuchin-scapegoating and overprotection of a child by mother and father are both forms of: a) triangulation b) detouring c) a stable coalition d) marital skew

b) detouring --tension btwn husband and wife is reduced by focusing attention on the child

Toward the end of middle childhood, relationships btwn siblings become more: a) distant b) egalitarian c) conflictual d) complementary

b) egalitarian

If a psychologist is requested by a defense attorney to eval and testify about a defendant for the prupose of establishing the defense insanity, the psychologist should: a) obtain an order from the court before agreeing to evaluate the defendant b) evaluate the defendant and testify as requested but inform the defendant of the purpose of the evaluation and of the limits of confidentiality before doing so c) evaluate the defendant as requested and not be concerned about informing the defendant of the limits of confidentiality since privilege is waived in this situation d) refuse to evaluate the defendant because psychologists are not qualified to determine if a person is "insane"

b) evaluate the defendant and testify as requested but inform the defendant of the purpose of the evaluation and of the limits of confidentiality before doing so

A child who has just progresses into Kohlberd's conventional level of marl dvpt has been caught stealing from other children. Her parents should discourage the stealing by: a) tell her that she will receive a present at the end of each week that she does not steal b) help her understand how the children she steals from feel about having their things taken c) tell her that stealing is against the law d) have her personally return the stolen items to their owners

b) help her understand how the children she steals from feel about having their things taken -b/c she is able to put herself in the palce of others

Herek proposed that the term sexual prejudice should be used as a substitute for: a) heterosexual bias b) homophobia c) homosexism d) sexism

b) homophobia

15-20% people in the advanced stages of HIV dvp AIDS related dementia. Common early signs are: a) irritability, fidgeting, disorganized speech, and lack of initiative b) lapses in concentration, forgetfulness, difficulty walking, and social withdrawal c) distractibility, behavioral disinhibition, reduced speech production, and neglect of personal hygiene d) fatigue, anxiety, concentration problems, and sleep disturbances

b) lapses in concentration, forgetfulness, difficulty walking, and social withdrawal

From the perspec of the situational leadership model, the selling leadership style is most effective when an employee is : a) low in ability and low in willingness b) low in ability and high in willingness c) high in ability and low in willingness d) high in ability and high in willingness

b) low in ability and high in willingness

Research investigating the relationship btwn interest test scores and future occupational choice suggests that these tests have the highest predictive validity for: a) lower-class people b) middle-class people c) upper-class people d) lower-and upper-class people

b) middle-class people --most likely to have the opportunity to choose jobs that coincide with their interests

A researcher interested in the use of guided imagery on anx reduction discovers that imaging ability explains why guided imagery has + effects for only some individuals-ie. those who have good imaging ability benefit from guided imagery but those with poor imaging ability don't. Imaging ability is: a) mediating variable b) moderator variable c) feedback variable d) outcome variable

b) moderator variable -it moderates the strength of the relationship btwn the IV and the DVs (ie. btwn guided imagery and anx reduction)

Nonsexist therapy, in comparison to feminist therapy, places greater emphasis on: a) the impact of sex roles and sexism b) modification of personal behavior c) treating men and women the same d) the egalitarian nature of the therapeutic relationship

b) modification of personal behavior (a & c are characteristic of both types of therapy)

Which technique should be used to eval convergent and divergent validity? a) cluster analysis b) multitrait-multimethod matrix c) discriminant function analysis d) structural equation modeling

b) multitrait-multimethod matrix

When working with an Af-Am client, it is important to recog that their past exps in racist society are likely to have fostered feelings of powerlessness. Conseq, a good strategy for a therapist is to adopt a: a) non-directive, less active approach b) problem-solving, time-limited approach c) color-blind approach d) neutral, educative approach

b) problem-solving, time-limited approach ; goal-directed therapies are most effective with members of most minority groups

Glasser was a co-founder of: a) Gestalt therapy b) reality therapy. c) solution-focused brief therapy. d) transactional analysis.

b) reality therapy.

Secondary reinforcers are: a) stimuli that increase a behavior through their accidental pairing with that behavior. b) stimuli that gain their reinforcing value through their association with an unconditioned reinforcer. c) high-frequency behaviors that are used to reinforce low-frequency behaviors. d) reinforcers that are removed (rather than applied) following a behavior.

b) stimuli that gain their reinforcing value through their association with an unconditioned reinforcer. (ie. money)

Rett's disorder: a) is approximately 4 to 5 times more common in males than in females b) is usually not apparent until the child reaches five or six years of age c) involves severe impairments in expressive and receptive language d) involves perceptual distortions and eccentricities of behavior

c) involves severe impairments in expressive and receptive language

During the 3rd therapy session, a client reveals to you that he has been seeing another psychologist for the past 6 months. The client claims that this therapist doesn't seem to be doing any good and he feels you will be better able to help him. You should: a) call the psychologist and inform her of the situation but only with the client's consent b) tell the client you will continue therapy with him but discuss the implications of the situation c) tell the client you cannot continue therapy with him unless he terminates therapy with the other psychologist d) continue seeing the client but suggest that it would be best if he terminates therapy with the other psychologist

b) tell the client you will continue therapy with him but discuss the implications of the situation

Incremental validity refers to the gain in hiring effective employees by using a predictor as opposed to not using it. The most imp contributor to incremental validity is its criterion-related validity coefficient: Higher coefficient means greater validity. However, the incremental validity of a predictor is also affected by other factors and the predictor will have the highest incremental validity when : a) the selection ratio is moderate and the base rate is high b) the selection ratio is low and the base rate is moderate c) the selection ratio is high and the base rate is moderate d) the selection ratio is moderate and the base rate is low

b) the selection ratio is low (there are lots of applicants) and the base rate is moderate (the current selection technique is moderately accurate)

For a Bowenian therapist, when a family exhibits a high degree of fusion but one member is more differentiated than others, you should: a) work with the least differentiated family members b) work with the most differentiated family member c) work with all family members as a group d) work with all family members in individual therapy until they reach a similar level of differentiation

b) work with the most differentiated family member --that member's + change motivates others to move toward greater differentiation

A researcher is studying the ability of students to remember lists of related word pairs. Prior to studying the to-be-rememb list, the exptl group is presented with a list of opposite word pairs. The control group completes multiplication problems. What results should the researcher expect when the 2 groups are subsequently asked to recall the word pairs? a) Due to retroactive interference, the control group will recall more word pairs. b) Due to retroactive interference, the experimental group will recall more word pairs. c) Due to proactive interference, the experimental group will recall fewer word pairs. d) Due to proactive interference, the control group will recall fewer word pairs.

c) Due to proactive interference, the experimental group will recall fewer word pairs. (what occured before interfers)

the parallel distribution processing (PDP) model describes memory in terms of : a) different levels of encoding. b) multiple discontinuous stages. c) a network of highly interconnected units. d) separate verbal and nonverbal coding systems.

c) a network of highly interconnected units.

You receive a request from the Thics Committe asking for info about a former client who has filed a complaint against her current therapist. You stopped seeing her 7 years ago. You should: a) not comply with the Committee's request initially and claim privilege on behalf of your client b) cooperate with the Committee's request by sending them the information you believe is relevant to the case c) cooperate with the Committee's request by sending them the information you believe is relevant to the case after confirming that the client has signed a release d) inform the Committee that the information you have about the client is obsolete and that you're ethically obligated not to forward it to them

c) cooperate with the Committee's request by sending them the information you believe is relevant to the case after confirming that the client has signed a release --must cooperate fully but also address any confiden issues

When using the multitrait-multimethod matrix: a) different examinees are tested at the same time with multiple measures and their scores on the measures are correlated b) the same examinees are tested at different times with multiple measures and their scores on the measures are correlated c) different examinees are tested on multiple (same and different) traits using multiple measures and their scores on the measures are correlated d) the same examinees are tested on multiple (same and different) traits using multiple measures and their scores on the measures are correlated

c) different examinees are tested on multiple (same and different) traits using multiple measures and their scores on the measures are correlated

Which technique should be used to classify people into criterion groups based on their scores on 2/more predictors? a) cluster analysis b) multitrait-multimethod matrix c) discriminant function analysis d) structural equation modeling

c) discriminant function analysis

Following a stroke, a woman has difficulty articulating words and her speech is slow, garbled and difficult to underst. These symps are charac of : a) dysphasia b) dyspraxia c) dysarthria d) dysprosody

c) dysarthria =speech rather than language disorder

Sophie's general stage theory of lesbian identity dvpt has which 4 stages? a) identity confusion, identity comparison, identity acceptance, and identity pride b) sensitization, identity confusion, identity assumption, and identity integration c) first awareness, testing and exploration, identity acceptance, and identity integration d) pre-encounter, encounter, immersion-emersion, and internalization

c) first awareness, testing and exploration, identity acceptance, and identity integration

From the perspec of the situational leadership model, the participating leadership style is most effective when an employee is : a) low in ability and low in willingness b) low in ability and high in willingness c) high in ability and low in willingness d) high in ability and high in willingness

c) high in ability and low in willingness

Dvptl research has linked anxiety in an expectant mother to which of the following? a) premature birth b) childhood autism c) increased crying in the newborn d) decreased activity levels in the newborn

c) increased crying in the newborn

To be consistent with the Fed Educational Rights and the Privacy Act, a school psychologist must keep in mind that: a) it is necessary to obtain the signed consent of a student's parents before releasing any information from the student's file to officials of another school where the student is planning to enroll b) student records, including identifying information, must be released to designated federal and state educational authorities if needed in connection with the evaluation of federally-sponsored educational programs c) parents have the right not only to inspect and review their children's school records but also to challenge the contents of records d) all of the above

c) parents have the right not only to inspect and review their children's school records but also to challenge the contents of records

Someone taking an SSRI with an MAOI could dvp __________ characterised by agitation, confusion, tremor, unsteady gait, sweating and chills a) a hypertensive crisis b) a cholinergic crisis c) serotonin syndrome d) sympathomimetic syndrome

c) serotonin syndrome

Gilligan-adolescent girls need exps that help them: a) plan for their future in the adult world. b) separate successfully from their families. c) stay connected to themselves and others. d) explore alternative identities.

c) stay connected to themselves and others. --during adolescence, girls exp a relational crisis that involves psychological separation from selves, others and the world due to ext pressures to fit cultural stereotypes. They need to be provided with exps that help them resist disconnection

Bob perceives the sound of a piano as blue, the sound of a violin as green, and the sound of a trumpet as purple. This condition is referred to as: a) achromatopsia b) paresthesia c) synesthesia d) simultagnosia

c) synesthesia

When a psychologist is asked to assess an individ's mental capacity for legal reasons, the most important factor to eval is : a) the person's level of intelligence. b) the person's risk for harm to self or others, c) the "person-environment fit." d) the physical health status of the person.

c) the "person-environment fit." --do this person's abilities match the demands of the situation he is facing?

An 11-year-old loves playing the piano and does so for at least one hour a day. His parents have begun to reward him for doing so by giving him a dollar for each 30 minutes that he practices and find that this increases the amount of time he spends playing the piano each day. Subsequently, the child's parents decide to discontinue the reward. Research by Lepper, Greene, and Nisbett (1973) suggests that, once the parents discontinue the reward, which of the following willl occur: a) the child will continue playing the piano at the increased rate b) the child will go back to playing the piano at the pre-reward rate c) the child will play the piano at a rate less than the pre-reward rate d) the child will initially play the piano at a reduced rate but will eventually go back to the pre-reward rate

c) the child will play the piano at a rate less than the pre-reward rate =overjustification hypothesis-providing an extrinsic reward will reduce intrinsic motivation

According to Fielder's contingency model, which of the following is most important for maximising a supervisor's ability to effectively lead his employees a) each employee is highly motivated b) each employee is very proficient at his/her job c) the leader has good relationships with his/her employees d) the leader is highly experienced

c) the leader has good relationships with his/her employees ; leader-member relationships are most critical

A transactional analyst would most likely be interested in which of the following: a) the client's sense of responsibility and needs for power, freedom, and belongingness b) the client's cognitive distortions (e.g., selective inattention, misperceptions) c) the nature of the client's ego state pathology d) the client's level of "response-ability"

c) the nature of the client's ego state pathology


Conjuntos de estudio relacionados

Combined life and health insurance final exam

View Set

Fundamentals Evolve practice questions

View Set

Biodiversity Unit 3 Review: Part 4

View Set

HESI Case Study: Myasthenia Gravis

View Set

Chapter 11 Mendel and the Gene Idea

View Set

Patho Chapter 12: Disorders of White Blood Cells

View Set